Relations and Functions - EduGorilla Study Material

69
| JUNE ‘17 12 CLASS XI Series 2 Relations and Functions X If A and B be two non-empty sets, then cartesian product of sets is A × B = {(x i , y i ) : x i A, y i B} X (x, y) = (p, q) x = p, y = q X A × B = B × A A = B X If n(A) = p, n(B) = q, then n(A × B) = pq X A × A × A = {(a, b, c) : a, b, c A} is called ordered triplet. X A × B = f A = f or B = f X Subset of X × Y is called a relation from X to Y. X If n(X) = p and n(Y) = q then the total number of relations from X to Y is 2 pq . X e set of all first elements of the ordered pairs in relation R from set X to set Y is called the domain of the relation R. X e set of all second elements of the ordered pairs in relation R from set X to Y is called the range of the relation R. X The whole set B is called the co-domain of the relation R. h Range Co-domain X If R = {(a, b) : a, b R}, then R –1 = {(b, a) : b, a R} X A subset f of X × Y is called a function (or map or mapping) from X to Y iff for each x X, there exists a unique y Y such that (x, y) f. It is written as f : X Y. h Set X is called domain and Set Y is called co-domain of the function f. h e set of elements of Y, which are assigned to the elements of X is called range of f. X Algebra of real functions If f : X R and g : X R, then h (f + g) (x) = f(x) + g(x), x X h (f – g) (x) = f(x) g(x), x X h (af )(x) = a f (x), x X h (f g)(x) = f(x)g(x), x X h f g x fx gx gx x X = ≠0 () () () , () , X Function Relation Cartesian Product X If A and B be two non-empty sets having n elements in common, then A × B and B × A have n 2 elements in common. IMPORTANT FORMULAE

Transcript of Relations and Functions - EduGorilla Study Material

| JUNE ‘1712

CLASS XI Series 2

Relations and Functions

X If A and B be two non-empty sets, then cartesian product of sets is A × B = {(xi , yi) : xi ∈A, yi ∈B}

X (x, y) = (p, q) ⇔ x = p, y = q

X A × B = B × A ⇒ A = B

X If n(A) = p, n(B) = q, then n(A × B) = pq

X A × A × A = {(a, b, c) : a, b, c ∈ A} is called ordered triplet.

X A × B = f ⇔ A = f or B = f

X Subset of X × Y is called a relation from X to Y.

X If n(X) = p and n(Y) = q then the total number of

relations from X to Y is 2pq

.

X �e set of all �rst elements of the ordered pairs in relation R from set X to set Y is called the domain of the relation R.

X �e set of all second elements of the ordered pairs in relation R from set X to Y is called the range of the relation R.

X The whole set B is called the co-domain of the relation R.

h Range ⊆ Co-domain

X If R = {(a, b) : a, b ∈ R}, then

R–1 = {(b, a) : b, a ∈ R}

X A subset f of X × Y is called a function (or map or mapping) from X to Y iff for each x ∈ X, there exists a unique y ∈Y such that (x, y) ∈ f. It is written as f : X → Y.

h Set X is called domain and Set Y is called co-domain of the function f.

h �e set of elements of Y, which are assigned to the elements of X is called range of f.

X Algebra of real functionsIf f : X → R and g : X → R, then

h (f + g) (x) = f(x) + g(x), ∀ x ∈X

h (f – g) (x) = f(x) – g(x), ∀ x ∈X

h (af )(x) = a f (x), ∀ x ∈X

h (f g)(x) = f(x)g(x), ∀ x ∈ X

hfg

xf xg x

g x x X

= ≠ 0 ∈( )( )( )

, ( ) ,

X Function ⊆ Relation ⊆ Cartesian Product

X If A and B be two non-empty sets having n elements in common, then A × B and B × A have n2 elements in common.

IMPORTANT FORMULAE

| JUNE ‘1714

WORK IT OUT

VERY SHORT ANSWER TYPE

1. Let A = {1, 2} and B = {3, 4, 5}. Find(i) B × A (ii) A × A × A

2. Let P = {x, y, z} and Q = {3, 4}. Find the number ofrelations from P to Q.

3. Let f be the exponential function and g be thelogarithmic function. Find (fg)(1).

4. Let f(x) = x2 and g(x) = (3x + 2) be two real functions. �en, �nd (f + g)(x).

5. Let g = {(1, 2), (2, 5), (3, 8), (4, 10), (5, 12), (6, 12)}Is g a function? If yes, �nd its domain and range. Ifno, give reason.

SHORT ANSWER TYPE

6. Let f : Z → Z, g : Z → Z be functions de�ned byf = {n, n2) : n ∈ Z} and g = {(n, |n|2) : n ∈ Z}.Show that f = g.

7. �e function F(x) = 95

32x + is the formula to

convert x° C to Fahrenheit units. Find(i) F(0) (ii) F(–10)(iii) the value of x when F(x) = 212.

8. If f(x) = log ,11

+−

xx

prove that f xx

f x21

22+

= ( ).

9. Let R be a relation on the set of natural numbersN de�ned by xRY ⇔ x + 2y = 41; ∀ x, y ∈ N. Findthe domain and range of R.

10. If P = {a, b} and Q = {x, y, z}, then show thatP × Q ≠ Q × P.

LONG ANSWER TYPE - I

11. Given f (x) = 11( )− x

, g(x) = f {f (x)} and

h(x) = f [f {f (x)}]. �en �nd the value of f (x) ⋅ g(x) ⋅ h(x).

12. Find the domain of the function

yx

x=−

+ +11

210log ( )

13. If A ⊆ B and C ⊆ D, prove that A × C ⊆ B × D.

14. Let f be a real valued function de�ned by

f x e ee e

x x

x x( ) ,| |

| |= −+

−then �nd the range of f.

15. Consider the following :(i) f : R → R : f (x) = logex

(ii) g : R → R : g(x) = x

(iii) h : A → R : h(x) =−

142x

, where A = R – {–2, 2}

Which of them are functions? Also �nd their range, if they are functions.

LONG ANSWER TYPE - II

16. Find the domain and range of the real valued

function f (x) given by f (x) = 44

−−

xx

.

17. If f : R → R is de�ned by f (x) = x3 + 1 and g : R → Ris de�ned by g(x) = x + 1, then �nd(i) f + g (ii) f – g (iii) f ⋅ g

(iv) fg

(v) af (a ∈ R)

18. Find the domain and range of the function

f (x) =−

12 3sin x

19. Let A = {x ∈ N : x2 – 5x + 6 = 0}, B = {x ∈ Z : 0 ≤ x < 2}and C = {x ∈ N : x < 3}, then verify that:(i) A × (B ∪ C) = (A × B) ∪ (A × C)(ii) A × (B ∩ C) = (A × B) ∩ (A × C)

20. Find the domain of the function

f(x) = (log ( ))227 6x x x+ − −

SOLUTIONS

1. (i) B × A = {3, 4, 5} × {1, 2}= {(3, 1), (3, 2), (4, 1), (4, 2), (5, 1), (5, 2)}(ii) A × A × A = {1, 2} × {1, 2} × {1, 2}= {(1, 1, 1), (1, 1, 2), (1, 2, 1), (1, 2, 2), (2, 1, 1), (2, 1, 2), (2, 2, 1), (2, 2, 2)}2. Given, P = {x, y, z} and Q = {3, 4}\ n(P) = 3 and n(Q) = 2\ n(P × Q) = 3⋅2 = 6Total number of relations from P to Q = number of subsets of P × Q = 26 = 64.3. We have, f : R → R given by f(x) = ex and g : R+ → Rgiven by g(x) = logexDomain (f ) ∩ Domain (g) = R ∩ R+ = R+.\ fg : R+ → R is given by (fg)(x) = f(x) g(x) = ex·logex.Now, (fg)(1) = f(1)g(1) = e1 × loge1 = e × 0 = 0.4. We have (f + g)(x) = f(x) + g(x) = x2 + (3x + 2)5. Yes, dom ( g) = {1, 2, 3, 4, 5, 6},

range ( g) = {2, 5, 8, 10, 12}

| JUNE ‘17 15

6. Given, domain of f = Z and domain of g = ZHence, domain f = domain g = Z …(1)Also, f(n) = n2, for all n ∈ Z and g(n) = |n|2 = n2 for all n ∈ ZHence, f(n) = g(n) for all n ∈ Z …(2)From (1) and (2), we have f = g.

7. F(x) = 95

32x + (given)

(i) F( )0 9 05

32 32= × +

= ⇒ F(0) = 32

(ii) F( ) ( )− = × − +{ } =10 9 105

32 14 ⇒ F(–10) = 14

(iii) F(x) = 212 ⇔ 95

32 212x + =

⇔ 9x = (5 × 180) ⇔ x = 100

8. Given, f x xx

( ) log= +−

11

\ f xx

xxxx

x xx

21

1 21

1 21

1 212

2

2

2

2+

=+

+

−+

= + ++

log log−−

2x

= +−

= +−

=log log ( ).11

2 11

22x

xxx

f x

9. We have, y = 412− ∈x N

Clearly x = 1, 3, 5, 7, …, 39\ Domain R = {x : (x, y) ∈ R; x + 2y = 41}

= {1, 3, 5, 7, …, 39} = set of odd natural numbers less than 40.Now, y can be only those natural numbers for which x ∈ N i.e., x = 41 – 2y ∈ N.Clearly, y = 1, 2, 3, …, 20.\ Range of R = {y : x + 2y = 41} = {1, 2, 3, …, 20}

= set of natural numbers less than 21.10. We have, P = {a, b} and Q = {x, y, z}Now, P × Q = {(a, x), (a, y), (a, z), (b, x), (b, y), (b, z)}Q × P = {(x, a), (x, b), (y, a), (y, b), (z, a), (z, b)}\ P × Q ≠ Q × P.11. Given, g(x) = f {f (x)}

= fx

x

xx

11

1

1 11

1−

=−

= − −

and h(x) = f [f {f (x)}] = f {g(x)} = − −

f x

x1

=+ −

=1

1 1 xx

x

\ f(x) ⋅ g(x) ⋅ h(x) = 11

1 1−

− −

⋅ = −x

xx

x

12. For y to be de�ned(i) log10 (1 – x) must be de�ned ⇒ 1 – x > 0 ⇒ x < 1(ii) log10 (1 – x) ≠ 0 ⇒ 1 – x ≠ 100 ⇒ 1 – x ≠ 1 ⇒ x ≠ 0(iii) x + 2 ≥ 0 ⇒ x ≥ –2From (i), (ii) and (iii), we get –2 ≤ x < 1 and x ≠ 0\ –2 ≤ x < 0 or 0 < x < 1Hence domain = [–2, 0) ∪ (0, 1).13. Let (a, b) be an arbitrary element of A × C. �en,(a, b) ∈ A × C⇒ a ∈ A and b ∈ C⇒ a ∈ B and b ∈ D [ A ⊆ B and C ⊆ D]⇒ (a, b) ∈ B × D�us, (a, b) ∈ A × C⇒ (a, b) ∈ B × D for all (a, b) ∈ (A × C).\ A × C ⊆ B × D

14. Let y = f x e ee e

x x

x x( )| |

| |= −+

If x ≥ 0 then y ee

x

x= −2

21

2⇒ e

yx2 1

1 21=

−≥ ( x ≥ 0)

⇒ 11 2

1 0−

− ≥y

⇒y

y1 20

−≥ or

yy2 1

0−

\ 0 12

≤ <y ⇒ y ∈

0 12

,

15. f and g are not functions as they are not de�ned fornegative values of x. But h is a function.

\ For range of h, Let y = h(x) = 1

42x −

⇒ x2 – 4 = 1y

⇒ xy

2 4 1= + ⇒ xyy

=+4 1

Hence, range of h = −∞ −

∪ ∞, ( , )14

0

ANSWER KEYMPP-2 CLASS XII

1. (c) 2. (b) 3. (b) 4. (d) 5. (c)6. (c) 7. (a,b) 8. (b,c) 9. (b,c) 10. (b)11. (a,b) 12. (a) 13. (a,b,d) 14. (d) 15. (b)16. (d) 17. (2) 18. (3) 19. (2) 20. (7)

| JUNE ‘1716

16. We have, f x xx

( ) .= −−

44

Domain of f : We observe that f (x) is de�ned for all x except at x = 4. At x = 4, f (x) takes the indeterminate

form 00

. �erefore, Domain (f ) = R – {4}.

Range of f : For any x ∈ Domain (f ) i.e. for any x ≠ 4, we have

f x xx

xx

( ) ( ) .= −−

= − −−

= −44

44

1

\ Range (f ) = {–1}.17. (i) f + g : R → R is de�ned by(f + g) (x) = f (x) + g(x) = x3 + 1 + x + 1 = x3 + x + 2(ii) f – g : R → R is de�ned by(f – g) (x) = f (x) – g(x) = x3 + 1 – x – 1 = x3 – x(iii) f ⋅ g : R → R is de�ned by(fg) (x) = f (x) g(x) = (x3 + 1) (x + 1) = x4 + x3 + x + 1

(iv) fg

: R – {– 1} → R is de�ned by

fg

xf xg x

xx

x x xx

x x

= = ++

= + − ++

= − +( )( )( )

( )( )3 221

11 1

11

(v) af : R → R is de�ned by(af ) (x) = a f (x) = a (x3 + 1) = a x3 + a

18. We have, f (x) =−

12 3sin x

Domain of f : We know that –1 ≤ sin 3x ≤ 1 for all x ∈ R⇒ –1 ≤ – sin 3x ≤ 1 for all x ∈ R⇒ 1 ≤ 2 – sin 3x ≤ 3 for all x ∈ R⇒ 2 – sin 3x ≠ 0 for any x ∈ R

⇒ f (x) =−

12 3sin x

is de�ned for all x ∈ R

Hence, domain ( f) = R.Range of f : 1 ≤ 2 – sin 3x ≤ 3 for all x ∈ R

⇒ 13

12 3

1≤−

≤sin x

for all x ∈ R

⇒ 13

1≤ ≤f x( ) for all x ∈ R

⇒ f (x) ∈ [1/3, 1]Hence, range ( f ) = [1/3, 1]

19. We haveA = {x ∈ N : x2 – 5x + 6 = 0} = {2, 3};B = {x ∈ Z : 0 ≤ x < 2} = {0, 1} and

C = {x ∈ N : x < 3} = {1, 2}\ A = {2, 3}, B = {0, 1} and C = {1, 2}(i) (B ∪ C) = {0, 1, 2}\ A × (B ∪ C) = {2, 3} × {0, 1, 2}= {(2, 0), (2, 1), (2, 2), (3, 0), (3, 1), (3, 2)}(A × B) = {2, 3} × {0, 1} = {(2, 0), (2, 1), (3, 0), (3, 1)}(A × C) = {2, 3} × {1, 2} = {(2, 1), (2, 2), (3, 1), (3, 2)}\ (A × B) ∪ (A × C) = {(2, 0), (2, 1), (2, 2), (3, 0), (3, 1), (3, 2)}Hence, A × (B ∪ C) = (A × B) ∪ (A × C)(ii) (B ∩ C) = {0, 1} = {1}\ A × (B ∩ C) = {2, 3} × {1} = {(2, 1), (3, 1)}And,(A × B) ∩ (A × C) = {(2, 1), (3, 1)}Hence, A × (B ∩ C) = (A × B) ∩ (A × C)

20. We have, f(x) = (log ( ))227 6x x x+ − −

= + − −(log ( )) ( )( )2 1 6x x x

For f (x) to be de�ned.(i) (log2(x)) ≥ 0 ⇒ x ≥ 20 ⇒ x ≥ 1(ii) (1 – x) (x – 6) ≥ 0 ⇒ 1 ≤ x ≤ 6�erefore domain of f = [1, 6].

| JUNE ‘1718

Only One Option Correct Type

1. 1

8 81

8 8

8

+ +

− +

i

i

sin cos

sin cos

π π

π πequals

(a) 28 (b) 0(c) – 1 (d) 1

2. If a, b, c are in G.P., then the equationsax2 + 2bx + c = 0, dx2 + 2ex + f = 0 have a common

root if da

eb

fc

, , are in

(a) A.P. (b) G.P.(c) H.P. (d) none of these

3. For positive integers n1 and n2 the value of theexpression ( )1 1+ i n + ( )1 3 1+ i n + ( )1 5 2+ i n + ( )1 7 2+ i n

where i = −1 is a real number i�(a) n1 = n2 (b) n2 = n2 – 1(c) n1 = n2 + 1 (d) ∀ n1 and n2

4. If x, y, z are distinct positive reals such thatlog log logxy z

yz x

zx y−

=−

=−

, then value of xx yy zz is

(a) 1 (b) 0(c) – 1 (d) none of these

5. If the roots of the equation bx2 + cx + a = 0be imaginary, then for all real values of x, theexpression 3b2x2 + 6bcx + 2c2 is

(a) less than 4ab (b) greater than –4ab(c) less than –4ab (d) greater than 4ab

6. Solve for x :log2x + 3 (6x2 + 23x + 21) + log3x + 7 (4x2 + 12x + 9) = 4(a) –4 (b) – 2

(c) − 14

(d) All of these

One or More Than One Option(s) Correct Type

7. ABCD is a rhombus, its diagonals AC and BDintersect at the point R where BD = 2AC. Its pointsD and R represent the complex numbers 1 + iand 2 – i respectively, then the complex numberrepresented by A is(a) (3, – 1/2) or (1, – 1/2)(b) (3, – 1/2) or (1, – 3/2)(c) (– 1/2, – 3/2) or (– 3/2, – 1/2)(d) None of these

8. (1 + i)5+ (1 – i)5 =(a) – 8 (b) 8

(c) −2 54

7 2/ cos π (d) −2 54

7 2/ cos π

9. If a and are non-real cube roots of unity andx = a + b, y = aa + b , z = a + ba, then(a) x + y + z = 1(b) x + y + z = 0(c) x3 + y3 + z3 = 3(a3 + b3)(d) none of these

10. �e equation whose roots are a + and a + ,if a are the roots of the equation ax2 + bx + c = 0and , are roots of the equation a x2 + b x + c = 0,is

Total Marks : 80 Time Taken : 60 Min.

This specially designed column enables students to self analyse their extent of understanding of specified chapters. Give yourself four

marks for correct answer and deduct one mark for wrong answer.

Self check table given at the end will help you to check your readiness.

Class XI

Complex Numbers and Quadratic Equations

| JUNE ‘17 19

(a) aa x2 + bb x + cc = 0 (b) ax2 + (a + a + b )x + bc = 0(c) aa x2 – bb x + cc = 0 (d) none of these

11. If A and B are the points (3, –1) and (2, 1) respectively, then the locus of the points P(z), z = x + yi, x, y ∈R, such that |z – 3 + i| = |z – 2 – i| is(a) a circle containing A and B (b) P is equidistant from A and B(c) right bisector of segment joining A and B(d) none of these

12. �e value of x : |x2 + 2x – 8| + x – 2 = 0 is(a) –5 (b) –2(c) 2 (d) –3

13. If a ∈C be such that |a| = 1, then the equation

11

4+−

iziz

= a has all the roots

(a) real and distinct (b) non-real(c) two real and two non-real (d) none of these

Comprehension Type

Let a be the roots of the equation 6x2 + 6px + p2 = 0, where p is a real number.

14. If both a and are greater than 2, then(a) p < – 4 (b) p < −2 6

(c) p < − −6 2 6 (d) none of these

15. �e equation whose roots are (a + )2 and (a – )2

is(a) 3x2 + 4p2x + p4 = 0 (b) 3x2 – 4p2x + p4 = 0(c) 3x2 – 4p2x – p4 = 0 (d) none of these

Matrix Match Type

16. Match the following :

Column I Column IIP. If a and b are posit ive

numbers and log a b+2

= +12

(log log )a b ab

then

is equal to

1. 3 3

Q. Let A(2 + 0i), B(–1 + 3i ) and C (–1– 3i ) b e t he vertices of ABC. Then, 6 sin A is equal to

2.

R. If one root of the equation (x – 1)(7 – x) = is three times the other, then =

3.

S. Conjugate of the complex number

− −72

3 32

i is

4. –2 + 3

P Q R S(a) 4 2 3 1(b) 3 4 1 2(c) 1 4 3 2(d) 2 1 3 4

Integer Answer Type

17. �e number of values of x in the interval [0, 3 ] satisfying the equation 2sin2x + 5sinx –3 = 0 is

18. �e least integral value of k for which(k– 2)x2 + 8x + k + 4 > 0 for all x ∈R, is

19. If a, are non-real cube roots of unity then (1 + a) (1 + ) (1 + a2) (1 + 2) (1 + a4) (1 + 4)....upto 2n factors is equal to

20. If a be non real cube root of unity, then α equals

Keys are published in this issue. Search now! J

Check your score! If your score is> 90% EXCELLENT WORK ! You are well prepared to take the challenge of final exam.

90-75% GOOD WORK ! You can score good in the final exam.

74-60% SATISFACTORY ! You need to score more next time.

< 60% NOT SATISFACTORY! Revise thoroughly and strengthen your concepts.

No. of questions attempted ……

No. of questions correct ……

Marks scored in percentage ……

| JUNE ‘1720

CBSE

CLASS XII Series 2

Inverse Trigonometric Functions

X Functions Domain Range

sin–1x [–1, 1]−

π π2 2

,

cos–1x [–1, 1] [0, ]

tan–1x (– , )−

π π2 2

,

IMPORTANT FORMULAE

PROPERTIES OF INVERSE TRIGONOMETRIC FUNCTIONS

1. sin–1(sin x) = x ∀ x ∈ −

π π2 2

, 2. sin(sin–1x) = x, ∀ x ∈[–1, 1]

cos–1(cos x) = x, ∀ x ∈ [0, ] cos(cos–1x) = x, ∀ x ∈ [–1, 1]

tan–1(tan x) = x, ∀ x ∈ −

π π2 2

, tan(tan–1x) = x, ∀ x ∈R

cot–1(cot x) = x, ∀ x ∈ (0, ) cot(cot–1x) = x, ∀ x ∈ R

sec–1(sec x) = x, ∀ x ∈ [0, ] – π2{ } sec(sec–1x) = x, ∀ x ∈ R –(–1, 1)

cosec–1(cosec x) = x, ∀ x ∈ −

−π π2 2

0, { } cosec(cosec–1x) = x, ∀ x ∈ R – (– 1, 1)

3. sin–1(–x) = – sin–1x, ∀ x ∈[–1, 1] cot–1(–x) = – cot–1x, ∀ x ∈R

cos–1(–x) = – cos–1x, ∀ x ∈[–1, 1] sec–1(–x) = – sec–1x, ∀ x ∈ (– , –1] ∪ [1, )

tan–1(–x) = –tan–1x, ∀ x ∈R cosec–1(–x) = –cosec–1x, ∀ x ∈ (– , –1] ∪ [1, )

Functions Domain Rangecot–1x (– , ) (0, )

cosec–1x (– , –1] ∪ [1, )−

{ }π π2 2

0,

sec–1x (– , –1] ∪ [1, ) [ , ]02

ππ

− { }

| JUNE ‘17 21

4. sin–1(1/x) = cosec–1x, ∀ x ∈ (– , –1] ∪ [1, ) 5. sin–1x + cos–1x = /2, ∀ x ∈[–1, 1]

cos–1(1/x) = sec–1x, ∀ x ∈ (– , –1] ∪ [1, ) tan–1x + cot–1x = /2, ∀ x ∈R

tan ( / ) cot ,cot ,

−−

−= >− + <

1

1

11 00

x x xx x

forforπ sec–1x + cosec–1x = /2, ∀ x ∈ (– , –1] ∪ [1, )

6.

tan–1x + tan–1y =

+−

> > <

++

tan , , ,

tan ,

1

1

10 0 1

1

x yxy

x y xy

x yxy

x

if

ifπ >> > >

− ++

< < >

0 0 1

10 0 11

,

tan , ,

y xy

x yxy

x y xy

and

if andπ

tan–1x – tan–1y =

−+

> < > −

+−

+

tan , , ,

tan ,

1

1

10 0 1

1

x yxy

x y xy

x yxy

if

ifπ xx y xy

x yxy

x y xy

> < < −

− +−

+

< > < −

0 0 1

10 0 11

,

tan , ,

and

if andπ

7.

sin sin

sin , ,

− −

+ =

− + −{ } − ≤ ≤ + ≤

<1 1

1 2 2 2 21 1 1 1 1

x y

x y y x x y x y

xy

if and

if 00 11 1 0 1 1

2 2

1 2 2 2 2and

or

if andx y

x y y x x y x y+ >

− − + −{ } < ≤ + >

− −

−π

π

sin , ,

siin , ,− − + −{ } − ≤ < + ≥

1 2 2 2 21 1 1 0 1x y y x x y x yif and

sin sin

sin , ,

− −

− =

− − −{ } − ≤ ≤ + ≤

>1 1

1 2 2 2 21 1 1 1 1

x y

x y y x x y x y

xy

if and

if 00 11 1 0 1 1 0

2 2

1 2 2 2 2

andif and

orx y

x y y x x y x y+ >

− − − −{ } < ≤ − ≤ ≤ + >−π sin , , 11

1 1 1 0 0 1 11 2 2 2 2− − − − −{ } − ≤ < < ≤ + >

−π sin , ,x y y x x y x yif and

8.

cos coscos , ,

cos

− −

−+ =

− − −{ } − ≤ ≤ + ≥

1 1

1 2 21 1 1 1 0

2x y

xy x y x y x yif and

π 11 2 21 1 1 1 0xy x y x y x y− − −{ } − ≤ ≤ + ≤

, ,if and

cos coscos , ,

cos

− −

−− =

+ − −{ } − ≤ ≤ ≤

− +

1 1

1 2 2

1

1 1 1 1x y

xy x y x y x y

xy

if and

11 1 1 0 0 12 2− −{ } − ≤ ≤ < ≤ ≥

x y y x x y, ,if and

| JUNE ‘1722

9.

2

2 1 12

12

2 1 12

11

1 2

1 2sin

sin ( ) ,

sin ( ) ,−

−=

− − ≤ ≤

− − ≤ ≤

x

x x x

x x x

if

ifπ

π −− − − ≤ ≤ −

−sin ( ) ,1 22 1 1 12

x x xif

3

3 4 12

12

3 4 12

11

1 3

1 3sin

sin ( ) ,

sin ( ) ,−

−=

− − ≤ ≤

− − < ≤

x

x x x

x x x

if

ifπ

π −− − − ≤ < −

−sin ( ) ,1 33 4 1 12

x x xif

10.

22 1 0 1

2 2 1 1 01

1 2

1 2cos

cos ( ),

cos ( ),−

−=

− ≤ ≤

− − − ≤ ≤

x

x x

x x

if

ifπ

3

4 3 12

1

2 4 3 12

12

2

1

1 3

1 3cos

cos ( ) ,

cos ( ) ,−

−=

− ≤ ≤

− − − ≤ ≤x

x x x

x x x

if

ifπ

ππ + − − ≤ ≤ −

−cos ( ) ,1 34 3 1 12

x x xif

11.

2

21

1 1

21

1

12

12tan

tan ,

tan ,−

−=

− < <

+−

>x

xx

x

xx

x

if

ifπ 11

21

112− +

< −

−π tan ,xx

xif

3

31 3

13

13

31 3

1

13

2

13

2tan

tan ,

tan−

−=

−−

− < <

+ −−

x

x xx

x

x xx

if

π

>

− + −−

< −

,

tan ,

if

if

x

x xx

x

13

31 3

13

13

12.

2

21

1 1

21

1

12

12tan

sin ,

sin ,−

−=

+

− ≤ ≤

−+

>x

xx

x

xx

x

if

ifπ 11

21

112− −

+

< −

−π sin ,xx

xif

2

11

0

11

1

12

2

12

2

tancos ,

cos ,

=

−+

≤ < ∞

− −+

x

xx

x

xx

if

if −− ∞ < ≤

x 0

13. sin cos tan− − −= −( ) =

1 1 2 12

11

x x x

x= −

=−

=

− − −cot sec12

12

11 1

11x

x x xcosec

cos sin tan− − −= −( ) = −

1 1 2 1 21 1x x x

x=

=

=−

− − −cot sec12

1 121

1 1

1x

x x xcosec

tan sin cos cot sec− − − −=+

=+

=

=1 12

12

1

1

1

11x

x

x x x−− −+( ) = +

1 2 1 21 1x x

xcosec

14.

If x1, x2, ... , xn ∈ R, then tan tan . . . tan tan

. . .− − − −+ + + =− + − +

− + −1

11

21 1 1 3 5 7

2 4 61x x x

S S S SS S Sn ++

. . .

where Sk = Sum of the products of x1, x2, ... , xn taken k at a time.

| JUNE ‘17 23

WORK IT OUTVERY SHORT ANSWER TYPE

1. Find the value of cot π4

2 31−

−cot .

2. If tan− =1 43

θ , �nd the value of cos .

3. Find the principal value of sin sin−

1 23π .

4. For the principal values, evaluate the following:

cot ( ) tan ( ) sec− − −− + +

1 1 13 1 23

5. Evaluate : cos cos( )− − °( )1 680

SHORT ANSWER TYPE

6. Evaluate : (i) sin (cot–1 x) (ii) cos (tan–1 x)

7. Prove that sin sin cos .− − −+ =1 1 1817

35

3685

8. Solve for x : cos(tan–1 x) = sin sec .−

1 1312

9. Solve : sin cos− −= +1 16

x xπ

10. If in a ABC, A = tan–12 and B = tan–13, then

show that C is equal to π4

.

LONG ANSWER TYPE-I

11. Write each of the following in the simplest form:

(i) tan–1(sec x + tan x) (ii) sin tan2 11

1− −+

xx

12. Show that 2 11

11

1 12

2tan sin− −+−

+ −

+

=x

xxx

π

13. Evaluate the following :

(i) sin (2 sin–1 0.8) (ii) tan tan2 15 4

1−

π

14. Find the value of

tan tan cos cos sec sec− − −

+

+

1 1 156

136

95

π π π

15. If tan ,− + − −

+ + −

=1

2 2

2 2

1 1

1 1

x x

x xα then prove that

x2 = sin 2 a.

LONG ANSWER TYPE-II

16. If a, b, c > 0 such that a + b + c = abc, �nd the value of tan–1 a + tan–1 b + tan–1 c.

17. If cos cos ,− −+ =1 1xa

yb

α prove that

xa

xyab

yb

2

2

2

222− + =cos sin .α α

18. If a1, a2, a3, ..., an is an arithmetic progression with common di�erence d, then show that

tan tan tan− −+

++

1

1 2

1

2 31 1da a

da a

++

+ ++

−tan ...1

3 4 11 1da a

da an n

is equal to a a

a an

n

−+

1

11.

19. Solve :

(i) cos sin cos− −+

=1 11

30x

(ii) tan tan tan ( )− − −+−

+ −

= + −1 1 111

1 7xx

xx

π

20. Prove that

(i) sin cos tan− − −+ + =1 1 11213

45

6316

π

(ii) cos tan tan− − −+ =1 1 145

35

2711

SOLUTIONS

1. cot cot cot cotπ π4

2 34

3 12 3

1 12

= − −

×

− −

= −

=

⋅ +

−=

+

−=−cot cot

cot

cot

ππ

π443

443

1

43 4

43

1

43

171

2. Given, tan− =1 43

θ , where θ π π∈ −

2 2

,

\ tan .θ = 43

We know that cos > 0, when θ π π∈ −

2 2

, .

\ cossec tan

θθ θ

= =+

=+

=1 1

1

1

1 169

352

| JUNE ‘1724

3. Since, sin sin sin23 3 3π π π π= −

=

\ sin sin sin sin .− −

=

=1 12

3 3 3π π π

Hence, the principal value of sin–1 sin 23π

is π

3.

4. cot ( ) tan ( ) sec− − −− + +

1 1 13 1 23

=

+

+

− − −cot cot tan tan sec sec1 1 156 4 6π π π

= + + =56 4 6

54

π π π π

5. cos cos( ) cos (cos )− −− °{ } = °1 1680 680

= cos cos cos cos− −

= −

1 1349

4 29

π π π

= cos cos−

= = °1 2

929

40π π

6. (i) We have, sin (cot–1 x) = sin cot .−

11x

Now, cot sin− −=+

1 1

2

1

1x

x

Hence, sin (cot–1 x) = sin sin−

+

=+

12 2

1

1

1

1x x

(ii) cos (tan–1 x) = cos tan−

11x

Now, tan cos− −=+

1 1

2

1

1x

x

Hence, cos (tan–1 x) = cos cos−

+

=+

12 2

1

1

1

1x x

7. L.H.S. = sin sin− −+1 1817

35

= cos cos− −+1 11517

45

= cos− ×

− −

⋅ −

1

2 21517

45

1 1517

1 45

= cos cos− −− ⋅

= − ×{ }1 11217

64289

925

1217

817

35

= cos cos− −−{ } =

=1 112

172485

3685

R.H.S.

Hence, sin sin cos .− − −+ =1 1 1817

35

3685

8. Let tan–1 x = f. �en, tanφ = x1

\ cos cosφ φ=+

⇒ =+

−1

1

1

121

2x x

So, cos(tan–1x) = cos cos−

+

=

+

12 2

1

1

1

1x x

Also, let sec .− =1 1312

θ �en, sec θ = 1312

\ sin sinθ θ= ⇒ = −513

513

1

So, sin sec sin sin− −

=

=1 113

125

135

13

�us, 1

1

5132+

=x

⇒ 11

251692( )+

=x

⇒ 1 + x2 = 16925

16925

12⇒ = −x ⇒ x = ± 14425

Hence, x = ± 125

.

9. We have, sin cos− −= +1 16

x xπ

⇒ sin cos− −− =1 16

x x π

⇒ sin sin− −− −

=1 1

2 6x xπ π

⇒ 2 23

1sin− =x π ⇒ sin− =13

x π ⇒ x = 32

10. We have, A = tan–12, B = tan–13 We know that, A + B + C = ⇒ tan–1 2 + tan–1 3 + C =

⇒ π π+ +− ×

+ ∠ =−tan 1 2 3

1 2 3C

⇒ + tan–1(–1) + C = ⇒ π π π− + ∠ =4

C

⇒ 34π π+ ∠ =C ⇒ ∠ =C π

4

11. (i) sec tan sincos

cos

sinx x x

x

x

x+ = + =

− +

+

1 12

2

π

π

=+

+

+

= +

24 2

24 2 4 2

4 2

2sin

sin costan

π

π ππ

x

x xx

| JUNE ‘17 25

| JUNE ‘1726

\ tan–1 tan π π4 2 4 2

+

= +x x

(ii) Put x = cos

\ sin tan sin tan coscos

2 11

2 11

1 1− −−+

= −

+

xx

θθ

=

−sin tan sin /

cos /2 2 2

2 21

2

2θθ

=

= = −−sin tan tan sin22

11 2θ θ x

12. Given expression is 2 11

11

1 12

2tan sin− −+−

+ −

+

xx

xx

Putting x = tan , we get

2 11

11

1 12

2tan tantan

sin tantan

− −+−

+ −

+

θθ

θθ

= 2 4

14

21 1tantan tan

tan tansin (cos )− −

+

− ⋅

+

π θ

π θθ

= 24 2

21 1tan tan sin sin− −+

+ −

π θ π θ

= 24 2

22

22

2π θ π θ π θ π θ π+

+ − = + + − =

13. (i) sin (2 sin–10.8) = sin sin . .− − ( )

1 22 0 8 1 0 8

= sin[sin–1(2 × 0.8 × 0.6)] = sin[sin–1(0.96)] = 0.96

(ii) tan tan tan tan2 15 4

2 15

1 15

1 12

− −−

=

π

π4

= tan tan tan tan tan− − −−

= −

1 1 1512 4

512

=−

+ ×

= −

− −tan tan tan tan1 1

512

1

1 512

1

717

= − 7

17

14. tan tan cos cos sec sec− − −

+

+

1 1 156

136

95

π π π

⇒ tan tan cos cos

sec se

− −

+ +

+

1 1

1

62

6π π π π

cc 25

π π−

⇒ tan tan cos cos

sec sec

− −

+

+

1 1

1

6 6

5

π π

π

= − + + =π π π π6 6 5 5

.

15. We have, tan− + − −

+ + −

=1

2 2

2 2

1 1

1 1

x x

x xα

⇒ 1 1

1 1

2 2

2 2

+ − −

+ + −=x x

x xtan α

⇒ 1 1 1 1

1 1 1 1

2 2 2 2

2 2 2 2

+ − −( ) + + + −( )+ − −( ) − + + −( )

x x x x

x x x x =

+−

tantan

αα

11

⇒ 2 1

2 1

11

1

1

11

2

2

2

2

+

− −=

+−

⇒ −

+=

−+

x

x

x

x

tantan

tantan

αα

αα

⇒ 11

2

2−+

=−+

xx

cos sincos sin

α αα α

⇒ 11

1 21 2

22

22−

+=

−+

⇒ =xx

xsinsin

sinαα

α

16. It is given that a + b + c = abc

\ abcc

ac

bc

= + +1

⇒ ab ac

bc

= + +

1 ⇒ ab a b

c− = +1

⇒ ab – 1 > 0 a b c a bc

, , > ∴ + >

0 0 ⇒ ab > 1 Now, tan–1 a + tan–1 b + tan–1 c

= π + +−

+− −tan tan1 1

1a b

abc [ ab > 1]

= π + −−

+− −tan tan1 1

1abc c

abc

= π +− −

+− −tan

( )tan1 11

1c ab

abc

= + tan–1 (–c) + tan–1 c = – tan–1 c + tan–1 c =

| JUNE ‘17 27

17. We have cos cos− −+ =1 1xa

yb

α

⇒ cos− ⋅ − −

=12

2

2

21 1xa

yb

xa

y

⇒ xyab

xa

y

b

x y

a b− − − + =1

2

2

2

2

2 2

2 2 cos α

⇒ xyab

xa

y

b

x y

a b− = − − +cos α 1

2

2

2

2

2 2

2 2

Squaring both sides, we have

x y

a b

xyab

xa

y

b

x y

a b

2 2

2 22

2

2

2

2

2 2

2 22

1− + = − − +cos cosα α

⇒ xa

xyab

y

b

2

2

2

222

1− + = −cos cosα α

\ xa

xyab

y

b

2

2

2

222

− + =cos sinα α

18. Given, a1, a2, a3, a4, ... , an is an arithmetic progression, then d = a2 – a1 = a3 – a2 = ... = an –an – 1

\ tan tan tan− −+

++

1

1 2

1

2 31 1da a

da a

++

+ ++

− −

−tan ... tan1

3 4

1

11 1da a

da an n

= tan tan tan− −−+

+−

+

1 2 1

1 2

1 3 2

2 31 1a a

a aa a

a a

+−

+

+ +−

+

− − −

−tan ... tan1 4 3

3 4

1 1

11 1a a

a aa a

a an n

n n

= tan [tan–1 a2 – tan–1 a1 + tan–1 a3 – tan–1 a2 + ... + tan–1an– tan–1 an – 1] = tan[tan–1 an – tan–1 a1]

= tan tan .− −+

=

−+

1 1

1

1

11 1a a

a aa a

a an

n

n

n

19. (i) Here, cos sin cos cos− −+

= = ±

1 113

02

x π

\ sin cos− −+ = ±1 113 2

x π

i.e., cos sin− −= ± −1 12

13

x π

\ x = ± −

−cos sinπ2

13

1

x = −

−cos sinπ2

13

1 x = − −

−cos sinπ2

13

1

=

=−sin sin 1 1

313

= +

−cos sinπ2

13

1

= −

= −−sin sin 1 1

313

\ x = ± 13

(ii) We have,

tan tan tan ( )− − −+

+ −

= + −1 1 111

1 7xx

xx

π

+−

+ −

− +−

= + −− −tan tan ( )1 1

11

1

1 11

1 7

xx

xx

xx

xx

π

\ ( ) ( )( ) ( )( )

tan tan ( )x x xx x x x

+ + −− − + −

= + − −1 11 1 1

72

⇒ x2 + x + x2 – 2x + 1 = –7(x2 – x – x2 + 1) ⇒ 2x2 – 8x + 8 = 0 i.e., x2 – 4x + 4 = 0 ⇒ (x – 2)2 = 0 \ x = 2

20. (i) L.H.S. = sin cos tan− − −+ +1 1 11213

45

6316

= tan tan tan− − −+ +1 1 1125

34

6316

= ++

− ×

+− −π tan tan1 1

125

34

1 125

34

6316

= π + −

+− −tan tan1 163

166316

= π π− + =− −tan tan1 16316

6316

(ii) Let cos .− =1 45

θ �en cos θ = 45

.

\ tan coscos

tanθ θθ

θ= − = ⇒ = −1 34

34

21

Consequently, cos tan− −=1 145

34

\ cos tan tan tan− − − −+ = +1 1 1 145

35

34

35

= tan tan− −+

− ⋅

=1 1

34

35

1 34

35

2711

| JUNE ‘1728

Only One Option Correct Type

1. �e number of positive integral solutions of

tan cos sin− − −++

=1 12

1

1310

x y

y is

(a) 0 (b) 1 (c) 2 (d) > 2

2. If cos ,− −+

<1

2

211 3

xx

p then x belongs to the interval

(a) −

13

13

, (b) −

13

13

,

(c) 0 13

,

(d) none of these

3. cos cos− −

1 1715

p is equal to

(a) 1715

p (b) 1315

p

(c) 315p

(d) −1715

p

4. �e domain of the function

f x

xx( ) cos [log( )]=

+ −− −1 12

43 is

(a) [–6, 6] (b) [–5, 2) ∪ (2, 3)(c) (2, 3) (d) [–6, 2) ∪ (2, 3)

5. If q are f are the roots of the equation 8x2 + 22x + 5 = 0, then

(a) both sin–1 q and sin–1 f are real(b) both sec–1 q and sec–1 f are real(c) both tan–1 q and tan–1 f are real(d) none of these

6. If sin–1 x + sin–1 y + sin–1z = 32p , then the value of

25 (x + y + z) −+ +

2163 3 3( )x y z

must be

(a) 1 (b) 2(c) 3 (d) none of these

One or More Than One Option(s) Correct Type

7. If cos–1x + (sin–1y)2 = pp2

4and

(cos )(sin )− −1 1 2x y

= p2

16, then

(a) 0 4 1≤ ≤ +pp

(b) p = 2 is the integral value of p(c) p = 0, 1, 2 (integral values)(d) none of these

8. For 0 < f < p2

, if x = cos2

0

n

nf

=

∑ , y = sin2

0

n

nf

=

∑ ,

z = cos sin2 2

0

n n

nf f

=

∑ , then

(a) xyz = xz + y (b) xyz = xy + z(c) xyz = x + y + z (d) xyz = yz + x

9. For the equation 2x = tan(2 tan–1a) + 2 tan(tan–1a + tan–1a3), which of the following is invalid?

(a) a2x + 2a = x (b) a2 + 2ax + 1 = 0(c) a ≠ 0 (d) a ≠ –1, 1

10. tan tan− −−+

+ −+

1 1

1

1 2 1

1 2 1a x ya y x

a aa a + −

+

−tan 1 3 2

2 3 1a aa a

+ + −+

− −

−...... tan 1 1

1 1a aa a

n n

n n+ −tan 1 1

anis equal to

Total Marks : 80 Time Taken : 60 Min.

This specially designed column enables students to self analyse their extent of understanding of specified chapters. Give yourself four

marks for correct answer and deduct one mark for wrong answer.

Self check table given at the end will help you to check your readiness.

Class XII

Inverse Trigonometric Functions

| JUNE ‘17 29

| JUNE ‘1730

(a) tan–1 xy (b) tan–1 xy

(c) tan–1 yx

(d) none of these

11. �e value of q for which

q q q

q= −

+

− −tan ( tan ) sin sin

cos,1 2 12 1

23 2

5 4 2 is/are:

(a) np + tan–1(–2) (b) np, np p+4

(c) np + cot–1(–2) (d) none of these12. If x, y and z are in A.P. and tan–1 x, tan–1 y and tan–1 z

are also in A.P. then (a) x = y = z (b) 2x = 3y = 6z(c) 6x = 3y = 2z (d) 6x = 4y = 3z

13. In DABC, ∠ =C p2

and

sin sin sin− − −=

+

1 1 1x axc

bxc

where a, b and c are the sides of triangle, then the values of x is/are(a) 0 (b) 1 (c) 2 (d) –1

Comprehension Type

tan tan tan− −

−=

− −−

=

−+

= −( )∑ ∑1 1

11

1 11

11x x

x xx xr r

r rr

n

r rr

n

= tan tan ,− −− ∀ ∈1 10x x n Nn

On the basis of above information, answer the following questions:

14. �e value of cosec–1 5 + cosec–1 65 +

cosec–1 ( )325 + .... to ∞ is

(a) p (b) 34p (c) p

2 (d) p

415. �e sum to in�nite terms of the series

tan tan− −

− +

+− +

12 4

12 4

21 1 1

41 2 2

+− +

+−tan ...12 46

1 3 3is

(a) p4

(b) p2

(c) 34p (d) none of these

Matrix Match Type16. Match the following :

Column I Column II

P. If 2 tan–1(2x + 1) = cos–1 (–x), then x is

1. − 12

Q. If 2 cos–1x = sin–1 2 1 2x x−( ),then x is

2. 32

R. If tan tan− −−−

+ ++

1 112

12

xx

xx

= p4

, then isx

3. –1

4. 0

P Q R(a) 2 1 3(b) 3 2 1(c) 3 1 2(d) 4 2 1

Integer Answer Type

17. �e number of solutions for the equation

2 1 32

1 2 1 2sin ( ) cos ( )− −− + + − =x x x x p is

18. If tan q + tan p q3

+

+ tan − +

p q3 = a tan 3q,

then a is equal to

19. If sin–1x + sin–1y + sin–1z = 32p and f(1) = 2,

f(p + q) = f(p). f(q), ∀ p, q ∈ R, then

x f(1) + y f(2) + z f(3) −+ +

+ +( )

( ) ( ) ( )

x y zx y zf f f1 2 3 is equal to

20. If λ = tan 2 15 4

1tan ,− −

p then the value of –17 λ2 must be

Keys are published in this issue. Search now!

Check your score! If your score is> 90% EXCELLENT WORK ! You are well prepared to take the challenge of final exam.

90-75% GOOD WORK ! You can score good in the final exam.

74-60% SATISFACTORY ! You need to score more next time.

< 60% NOT SATISFACTORY! Revise thoroughly and strengthen your concepts.

No. of questions attempted ……

No. of questions correct ……

Marks scored in percentage ……

| JUNE ‘17 31

CATEGORY-I (Q. 1 to Q. 50)Only one answer is correct. Correct answer will fetch full marks 1. Incorrect answer or any combination of more than one answer will fetch –¼ marks. No answer will fetch 0 marks.

1. �e number of all numbers having 5 digits, with distinct digits is(a) 99999 (b) 9 × 9P4 (c) 10P5 (d) 9P4

2. �e greatest integer which divides (p + 1)(p + 2) (p + 3).....(p + q) for all p∈N and �xed q ∈N is(a) p! (b) q! (c) p (d) q

3. Let (1 + x + x2)9 = a0 + a1x + a2x2 + ..... + a18x18. �en(a) a0 + a2 + ..... + a18 = a1 + a3 + ... + a17(b) a0 + a2 + ..... + a18 is even(c) a0 + a2 + ..... + a18 is divisible by 9(d) a0 + a2 + ..... + a18 is divisible by 3 but not by 9

4. �e linear system of equations 8 3 5 05 8 3 03 5 8 0

x y zx y zx y z

− − =− + =+ − =

has(a) only zero solution(b) only �nite number of non-zero solutions(c) no non-zero solution(d) in�nitely many non-zero solutions

5. Let P be the set of all non-singular matrices of order 3 over R and Q be the set of all orthogonal matrices of order 3 over R. �en(a) P is proper subset of Q(b) Q is proper subset of P(c) Neither P is proper subset of Q nor Q is proper

subset of P(d) P ∩ Q = f, the void set

6. Let A = x x

x Bx

x+

+

= +

2 33 2

05 2, . �en all

solutions of the equation det (AB) = 0 is(a) 1, –1, 0, 2 (b) 1, 4, 0, –2(c) 1, –1, 4, 3 (d) –1, 4, 0, 3

7. �e value of det A, where

A = 1 0

11 1

coscos cos

cos

θθ θ

θ−

− −

lies

(a) in the closed interval [1, 2](b) in the closed interval [0, 1](c) in the open interval (0, 1)(d) in the open interval (1, 2)

8. Let f : R → R be such that f is injective and f(x)f(y) = f(x + y), ∀x, y ∈R. If f (x), f (y), f (z) are in G.P., then x, y, z are in(a) A.P. always(b) G.P. always(c) A.P. depending on the value of x, y, z(d) G.P. depending on the value of x, y, z

9. On the set R of real numbers we de�ne xPy if and only if xy ≥ 0. �en the relation P is (a) re�exive but not symmetric(b) symmetric but not re�exive(c) transitive but not re�exive(d) re�exive and symmetric but not transitive

10. On R, the relation be de�ned by 'x y holds if and only if x – y is zero or irrational'. �en(a) is re�exive and transitive but not symmetric.(b) is re�exive and symmetric but not transitive.(c) is symmetric and transitive but not re�exive.(d) is equivalence relation

By : Anil Kumar Gupta (akg Classes), Asansol (W.B.) Mob : 09832230099

| JUNE ‘1732

11. Mean of n observations x1, x2, ....., xn is x . If an observation xq is replaced by xq then the new mean is

(a) x – xq + xq (b) ( )n x x

nq− + ′1

(c) ( )n x x

nq− − ′1

(d) nx x x

nq q− + ′

12. �e probability that a non leap year selected at random will have 53 Sundays is(a) 0 (b) 1/7 (c) 2/7 (d) 3/7

13. �e equation sin x(sin x + cos x) = k has real solutions, where k is a real number. �en

(a) 01 2

2≤ ≤

+k (b) 2 3 2 3− ≤ ≤ +k

(c) 0 2 3≤ ≤ −k (d) 1 22

1 22

− ≤ ≤ +k

14. �e possible values of x, which satisfy the trigonometric equation

tan tan− −−−

+++

=1 112

12 4

xx

xx

π are

(a) ± 12

(b) ± 2 (c) ± 12

(d) ± 2

15. Transforming to parallel axes through a point (p, q), the equation 2x2 + 3xy + 4y2 + x + 18y + 25 = 0 becomes 2x2 + 3xy + 4y2 = 1. �en(a) p = –2, q = 3 (b) p = 2, q = –3(c) p = 3, q = –4 (d) p = –4, q = 3

16. Let A(2, –3) and B(–2, 1) be two angular points of ABC. If the centroid of the triangle moves on the

line 2x + 3y = 1, then the locus of the angular point C is given by(a) 2x + 3y = 9 (b) 2x – 3y = 9(c) 3x + 2y = 5 (d) 3x – 2y = 3

17. �e point P(3, 6) is �rst re�ected on the line y = x and then the image point Q is again re�ected on the line y = –x to get the image point Q . �en the circumcentre of the PQQ is(a) (6, 3) (b) (6, –3) (c) (3, –6) (d) (0, 0)

18. Let d1 and d2 be the lengths of the perpendiculars drawn from any point of the line 7x – 9y + 10 = 0 upon the lines 3x + 4y = 5 and 12x + 5y = 7 respectively. �en(a) d1 > d2 (b) d1 = d2 (c) d1 < d2 (d) d1 = 2d2

19. �e common chord of the circles x2 + y2 – 4x – 4y = 0 and 2x2 + 2y2 = 32 subtends at the origin an angle equal to

(a) π3

(b) π4

(c) π6

(d) π2

20. �e locus of the mid-points of the chords of the circle x2 + y2 + 2x – 2y – 2 = 0 which make an angle of 90° at the centre is(a) x2 + y2 – 2x – 2y = 0(b) x2 + y2 – 2x + 2y = 0(c) x2 + y2 + 2x – 2y = 0(d) x2 + y2 + 2x – 2y – 1 = 0

21. Let P be the foot of the perpendicular from focus

S of hyperbola xa

yb

2

2

2

2 1− = on the line bx – ay = 0

and let C be the centre of the hyperbola. �en the area of the rectangle whose sides are equal to that of SP and CP is(a) 2ab (b) ab

(c) ( )a b2 2

2+ (d)

ab

22. B is an extremity of the minor axis of an ellipse whose foci are S and S . If SBS is a right angle, then the eccentricity of the ellipse is

(a) 12

(b) 12

(c) 23

(d) 13

23. �e axis of the parabola x2 + 2xy + y2 – 5x + 5y – 5 = 0 is(a) x + y = 0 (b) x + y – 1 = 0

(c) x y− + =1 0 (d) x y− = 12

24. �e line segment joining the foci of the hyperbola x2 – y2 + 1 = 0 is one of the diameters of a circle. �e equation of the circle is(a) x2 + y2 = 4 (b) x y2 2 2+ =(c) x2 + y2 = 2 (d) x y2 2 2 2+ =

25. �e equation of the plane through (1, 2, –3) and (2, –2, 1) and parallel to X-axis is(a) y – z + 1 = 0 (b) y – z – 1 = 0(c) y + z – 1 = 0 (d) y + z + 1 = 0

26. �ree lines are drawn from the origin O with direction cosines proportional to (1, –1, 1), (2, –3, 0) and (1, 0, 3). �e three lines are(a) not coplanar (b) coplanar

| JUNE ‘17 33

(c) perpendicular to each other(d) coincident

27. Consider the non-constant di�erentiable function f of one variable which obeys the relation f xf y

( )( )

= f(x – y). If f (0) = p and f (5) = q, then f (–5) is

(a) pq

2 (b) q

p (c) p

q (d) q

28. If f(x) = log5 log3 x, then f (e) is equal to(a) e loge 5 (b) e loge3

(c) 15e elog

(d) 13e elog

29. Let F(x) = ex, G(x) = e–x and H(x) = G(F(x)), where

x is a real variable. �en dHdx

at x = 0 is

(a) 1 (b) –1 (c) − 1e

(d) –e

30. If f ″(0) = k, k ≠ 0, then the value of

lim ( ) ( ) ( )x

f x f x f xx→

− +0 2

2 3 2 4 is

(a) k (b) 2k (c) 3k (d) 4k

31. If y = emsin–1x, then (1 – x2) d ydx

x dydx

ky2

2 − − = 0, where k is equal to(a) m2 (b) 2 (c) –1 (d) –m2

32. �e chord of the curve y = x2 + 2ax + b, joining the points where x = a and x = , is parallel to the tangent to the curve at abscissa x =

(a) a b+2

(b) 23

a b+ (c) 23

α β+ (d) α β+2

33. Let f (x) = x13 + x11 + x9 + x7 + x5 + x3 + x + 19. �en f (x) = 0 has(a) 13 real roots(b) only one positive and only two negative real

roots(c) not more than one real root(d) has two positive and one negative real root

34. Let f(x) = x

xx

x

p

q(sin ),

0

02

0

if

, if

< <

=

π, (p, q ∈ R). �en

Lagrange's mean value theorem is applicable to f (x) in closed interval [0, x],(a) for all p, q (b) only when p > q(c) only when p < q (d) for no value of p, q

35. lim(sin ) tan

x

xx→0

2

(a) is 2 (b) is 1(c) is 0 (d) does not exist

36. cos(log )x dx∫ = F(x) + c, where c is an arbitrary constant. Here F(x) = (a) x[cos(log x) + sin(log x)](b) x[cos(log x) – sin(log x)]

(c) x2

[cos(log x) + sin(log x)]

(d) x2

[cos(log x) – sin(log x)]

37. xx x

dx2

4 21

3 1−

+ +(x > 0) is

(a) tan− +

+1 1x

xc (b) tan− −

+1 1x

xc

(c) loge

xx

xx

c+ −

+ ++

1 1

1 1 (d) loge

xx

xx

c− −

− ++

1 1

1 1

38. Let I = sin

.x

xdx

1 810

19

+∫ �en,

(a) I < 10–9 (b) I < 10–7

(c) I < 10–5 (d) I > 10–7

39. Let I1 = [ ] { } ,x dx I x dxn n

and 20 0

=∫ ∫ where [x] and

{x} are integral and fractional parts of x and n∈N – {1}. �en I1/I2 is equal to

(a) 11n −

(b) 1n

(c) n (d) n – 1

40. �e value of lim .....n

nn

nn n→∞ +

++

+ +

2 2 2 21 2

12

is

(a) nπ4

(b) π4

(c) π4n

(d) π2n

41. �e value of the integral e dxx2

0

1

∫(a) is less than 1(b) is greater than 1(c) is less than or equal to 1(d) lies in the closed interval [1, e]

| JUNE ‘1734

42. e dxx x− =∫ [ ]

0

100

(a) e100 1100

− (b) ee

100 11−

(c) 100(e – 1) (d) e −1100

43. Solution of (x + y)2 dydx

= a2 ('a' being a constant) is

(a) ( ) tan ,x ya

y ca

c+ = + is an arbitrary constant

(b) xy = a tan cx, c is an arbitrary constant

(c) xa

yc

c= tan , is an arbitrary constant

(d) xy = tan (x + c), c is an arbitrary constant

44. �e integrating factor of the �rst order di�erential equation

x2(x2 – 1) dydx

+ x(x2 + 1)y = x2 – 1 is

(a) ex (b) xx

− 1 (c) xx

+ 1 (d) 12x

45. In a G.P. series consisting of positive terms, each term is equal to the sum of next two terms. �en the common ratio of this G.P. series is

(a) 5 (b) 5 12− (c) 5

2 (d) 5 1

2+

46. If (log5 x)(logx 3x)(log3x y) = logx x3, then y equals(a) 125 (b) 25 (c) 5/3 (d) 243

47. �e expression ( )( )

11 2

+− −

ii

n

n equals

(a) –in + 1 (b) in + 1 (c) –2in + 1 (d) 1

48. Let z = x + iy, where x and y are real. �e points

(x, y) in the X-Y plane for which z iz i

+−

purely imaginary lie on(a) a straight line (b) an ellipse(c) a hyperbola (d) a circle

49. If p, q are odd integers, then the roots of the equation 2px2 + (2p + q)x + q = 0 are(a) rational (b) irrational(c) non-real (d) equal

50. Out of 7 consonants and 4 vowels, words are formed each having 3 consonants and 2 vowels. �e number of such words that can be formed is(a) 210 (b) 25200 (c) 2520 (d) 302400

CATEGORY-II (Q. 51 to Q. 65)Only one answer is correct. Correct answer will fetch full marks 2. Incorrect answer or any combination of more than one answer will fetch –½ marks. No answer will fetch 0 marks.

51. Let A = 1 1 10 1 10 0 1

�en for positive integer n, An is

(a) 100 0

2

2n n

n nn

(b)

1 12

0 10 0 1

n n n

n

+

(c) 100 0

2

2

2

n nn n

n

(d)

1 2 1

0 12

0 0 12

2

n nn n

n

−+

+

52. Let a, b, c be such that b(a + c) ≠ 0.

If a a ab b b

c c c

a b ca b c

a bn n

+ −− + −

− ++

+ + −− − +

− −+ +

1 11 11 1

1 1 11 1 1

1 12 1( ) ( ) (−−1)nc= 0

then the value of n is(a) any integer (b) zero(c) any even integer (d) any odd integer

53. On set A = {1, 2, 3}, relations R and S are given by R = {(1, 1), (2, 2), (3, 3), (1, 2), (2, 1)} S = {(1, 1), (2, 2), (3, 3), (1, 3), (3, 1)}. �en

(a) R ∪ S is an equivalence relation(b) R ∪ S is reflexive and transitive but not

symmetric(c) R ∪ S is reflexive and symmetric but not

transitive(d) R ∪ S is symmetric and transitive but not

re�exive54. If one of the diameters of the curve x2 + y2 – 4x – 6y

+ 9 = 0 is a chord of a circle with centre (1, 1), the radius of the circle is(a) 3 (b) 2 (c) 2 (d) 1

55. Let A(–1, 0) and B(2, 0) be two points. A point M moves in the plane in such a way that

MBA = 2 MAB. �en the point M moves along(a) a straight line (b) a parabola(c) an ellipse (d) a hyperbola

56. If f(x) = | |−∫ t dtx

,1

then for any x ≥ 0, f(x) is equal to

(a) 12

(1 – x2) (b) 1 – x2

(c) 12

(1 + x2) (d) 1 + x2

| JUNE ‘17 35

57. Let for all x > 0, f(x) = lim ,n

nn x→∞

−( )1

1 then

(a) f(x) + fx1

=1 (b) f(xy) = f(x) + f(y)

(c) f(xy) = xf(y) + yf(x) (d) f(xy) = xf(x) + yf(x)

58. Let I = ( cos )1 20

100

−∫ xπ

dx, then

(a) I = 0 (b) I = 200 2(c) I = π 2 (d) I = 100

59. �e area of the �gure bounded by the parabolas x = –2y2 and x = 1 – 3y2 is

(a) 43

square units (b) 23

square units

(c) 37

square units (d) 67

square units

60. Tangents are drawn to the ellipse x y2 2

9 51+ =

at the ends of both latusrectum. �e area of the quadrilateral so formed is

(a) 27 sq. units (b) 132

sq. units

(c) 154

sq. units (d) 45 sq. units

61. �e value of K in order that f(x) = sin x – cos x – Kx + 5 decreases for all positive real values of x is given by(a) K < 1 (b) K ≥ 1 (c) K > 2 (d) K < 2

62. For any vector x, the value of ( ) ( ) ( )� � � � � �x i x j x k× + × + ×2 2 2 is equal to(a) | |x 2 (b) 2| |x 2 (c) 3| |x 2 (d) 4| |x 2

63. If the sum of two unit vectors is a unit vector, then the magnitude of their di�erence is(a) 2 units (b) 2 units(c) 3 units (d) 5 units

64. Let a and be the roots of x2 + x + 1 = 0. If n be positive integer, then an + n is

(a) 2 23

cos nπ (b) 2 23

sin nπ

(c) 23

cos nπ (d) 23

sin nπ

65. For real x, the greatest value of x xx x

2

22 4

2 4 9+ ++ +

is

(a) 1 (b) –1 (c) 12

(d) 14

CATEGORY-III (Q. 66 to Q. 75)One or more answer(s) is (are) correct. Correct answer(s) will fetch full marks 2. Any combination containing one or more incorrect answer will fetch 0 marks. Also no answer will fetch 0 marks. If all correct answers are not marked and also no incorrect answer is marked then score = 2 × number of correct answers marked ÷ actual number of correct answers.

66. If a, b ∈ {1, 2, 3} and the equation ax2 + bx + 1 = 0 has real roots, then(a) a > b(b) a ≤ b(c) number of possible ordered pairs (a, b) is 3(d) a < b

67. If the tangent to y2 = 4ax at the point (at2, 2at) where |t| > 1 is a normal to x2 – y2 = a2 at the point (a sec , a tan ), then(a) t = –cosec (b) t = –sec (c) t = 2 tan (d) t = 2 cot

68. �e focus of the conic x2 – 6x + 4y + 1 = 0 is(a) (2, 3) (b) (3, 2) (c) (3, 1) (d) (1, 4)

69. Let f : R → R be twice continuously. Let f(0) = f(1) = f (0) = 0. �en(a) f (x) ≠ 0 for all x(b) f (c) ≠ 0 for some c ∈R(c) f (x) ≠ 0 if x ≠ 0(d) f (x) > 0 for all x

70. If f(x) = xn, n being a non-negative integer, then the values of n for which f (a + ) = f (a) + f ( ) for all a, > 0 is(a) 1 (b) 2 (c) 0 (d) 5

71. Let f be a non-constant continuous function for all x ≥ 0. Let f satisfy the relation f(x) f(a – x) = 1 for

some a ∈ R+. �en I = dxf x

a

10

+∫ ( ) is equal to

(a) a (b) a4

(c) a2

(d) f(a)

72. If the line ax + by + c = 0, ab ≠ 0, is a tangent to the curve xy = 1 – 2x, then(a) a > 0, b < 0 (b) a > 0, b > 0(c) a < 0, b > 0 (d) a < 0, b < 0

73. Two particles move in the same straight line starting at the same moment from the same point in the same direction. �e �rst moves with constant velocity u and the second starts from rest with constant acceleration f. �en

| JUNE ‘1736

(a) they will be at the greatest distance at the end

of time uf2

from the start

(b) they will be at the greatest distance at the end

of time uf

from the start

(c) their greatest distance is uf

2

2

(d) their greatest distance is uf

2

74. �e complex number z satisfying the equation |z – i| = |z + 1| = 1 is(a) 0 (b) 1 + i (c) –1 + i (d) 1 – i

75. On R, the set of real numbers, a relation is de�ned as 'a b if and only if 1 + ab > 0. �en(a) is an equivalence relation(b) is re�exive and transitive but not symmetric(c) is re�exive and symmetric but not transitive(d) is only symmetric

SOLUTIONS

1. (b) : 1st digit must be other than 0 (zero) i.e. 9 ways.\ Other 4 digits can be �lled in 9P4 ways\ Total no. of numbers having 5 (distinct) digits = 9 × 9P42. (b) : (p + 1)(p + 2)(p + 3).....(p + q) is a product of q consecutive positive integers\ It must be always divisible by q!3. (b) : (1 + x + x2)9 = a0 + a1x + a2x2 + ..... + a18x18

Putting x = 1 and –1, we get39 = a0 + a1 + a2 + ..... + a18 ...(i)1 = a0 – a1 + a2 – ..... + a18 ...(ii)Adding (i) & (ii), we get3 1

2

9 + = a0 + a2 + a4 + ..... + a18

⇒ a0 + a2 + a4 + ..... + a18 = 9842, which is even but not divisible by 3 or 9.

4. (d) : 8 3 55 8 33 5 8

0 3 50 8 30 5 8

− −−

−=

− −−

[C1 → C1 + C2 + C3] = 0\ Given system of equation has in�nitely many non-zero solutions.5. (b) : Every orthogonal matrix is non-singular but every non-singular matrix may or may not be orthogonal.\ Q is proper subset of P.

6. (b) : AB = x x

xx

x+

+

+

2 33 2

05 2

= x x x x

x x x

2 2

217 3 6

8 10 4 4+ ++ + +

det(AB) = x xxx x( )+++ +217 3

8 10 2 = x(x + 2)(x2 – 5x + 4) = x(x + 2)(x – 1)(x – 4)\ det(AB) = 0 ⇒ x = 1, 4, 0, –2

7. (a) : |A| =1 0

11 1

coscos cos

cos

θθ θ

θ−

− − = (1 + cos2 )

– cos (0) + 0

= 1 + cos2 ∈ [1, 2].8. (c) : f(x)⋅f(y) = f(x + y) ...(i)\ f(y)f(y) = f(y + y) ⇒ {f(y)}2 = f(2y)⇒ f(x)f(z) = f(2y) [ f(x), f(y), f(z) are in G.P.]⇒ f(x + z) = f(2y) [from (i)]⇒ x, y, z are in A.P. but will depend on the value of x, y, z.9. (d) : x2 ≥ 0 \ x⋅x ≥ 0 ⇒ xPx ⇒ Re�exive xy ≥ 0 ⇒ yx ≥ 0 ⇒ Symmetric (–5)(0) ≥ 0 & (0)(7) ≥ 0i.e., (–5, 0) ∈ P & (0, 7) ∈ PBut, (–5)(7) < 0 ⇒ (–5, 7) P\ P is not transitive.10. (b) : Here (x, y) ∈ if x – y is zero or irrational. x – x = 0 for all x ∈ R⇒ is re�exiveIf x – y is zero or irrational then y – x is also zero or irrational.⇒ is symmetric.Let (x, y) ∈ & (y, z) ∈ \ x – y = 0 or irrational & y – z = 0 or irrationalBut, their sum x – z may or may not be 0 or irrationale.g., 2 – 3 is irrational & 3 – 5 both are irrational but their sum 2 – 5 = – 3 is neither zero nor irrational⇒ is not transitive.11. (d) : Mean of n observations x1, x2, ....., xn is x .\ Sum of n observations = nxIf xq is replaced by xq then sum = nx x xq q− + ′

\ New mean = nx x x

nq q− + ′

12. (b) : A non-leap year has 52 weeks & 1 extra day

\ Prob. of 53 sundays = 17

| JUNE ‘17 37

13. (d) : We have, k = sin2 x + sinx cos x = 12

[(1 – cos 2x) + sin 2x]

= 12

12

2 12

2 12

+ ⋅ − ⋅

sin cosx x

= 12

12

2 45+ − °sin ( )x

–1 ≤ sin(2x – 45°) ≤ 1

\ 12

12

12

12

− ≤ ≤ +k

⇒ 1 22

1 22

− ≤ ≤ +k

14. (a) : tan tan tan− − −++

= − −

1 1 112

1 12

xx

xx

⇒ xx

xxxx

++

=− −

+ −−

12

1 12

1 12

⇒ xx x

++

= −−

12

12 3

⇒ 2x2 = 1 ⇒ x = ± 12

15. (b) : Putting x = x + p and y = y + q in given equation, it becomes2(x + p)2 + 3(x + p)(y + q) + 4(y + q)2 + (x + p) + 18(y + q) + 25 = 0On comparing with 2x2 + 3xy + 4y2 = 1, we get 4p + 3q + 1 = 0 ...(i) 3p + 8q + 18 = 0 ...(ii)and 2p2 + 3pq + 4q2 + p + 18q + 25 = – 1 ...(iii)On comparing (i) & (ii), p = 2, q = –3 by which (iii) is also satis�ed.

16. (a) : Centroid 2 23

3 13

− + − + +

h k,

i.e., h k3

23

, −

lies on 2x + 3y = 1

\ 23

3 23

1h k

+ −

=

⇒ 2h + 3k – 6 = 3⇒ 2x + 3y = 9 is the reqd. locus.17. (d) : P(3, 6) has re�ection on y = x as Q(6, 3)Again re�ection of Q(6, 3) on y = –x will be Q (–3, –6) Slope of PQ × slope of QQ = (–1)(1) = –1\ PQQ is a right angled with PQQ = 90°\ Circumcentre will be mid pt. of hypotenuse PQ i.e. (0, 0)18. (b) : Bisectors of 3x + 4y = 5 & 12x + 5y = 7

are 3 4 5

3 4

12 5 7

12 52 2 2 2

x y x y+ −

+= ± + −

+

A(2, –3)

B (–2, 1)

C (h, k)

⇒ 13(3x + 4y – 5) = ±5(12x + 5y – 7)⇒ 21x – 27y + 30 = 0 & 99x + 77y = 100⇒ 7x – 9y + 10 = 0 & 99x + 77y = 100 7x – 9y + 10 = 0 is one of the bisector.\ Perp. from any pt. on it will be equal to both given lines \ d1 = d2.19. (d) : Equation of common chord i.e., S1 – S2 = 0 will be⇒ (x2 + y2 – 16) – (x2 + y2 – 4x – 4y) = 0⇒ x + y = 4, which subtends 90° at (0, 0)20. (c) : Let M(h, k) be the mid point of chord of (x + 1)2 + (y – 1)2 = (2)2 ...(i)subtending 90° at centre C(–1, 1)\ 2CM2 = CB2

⇒ (h + 1)2 + (k – 1)2 = (2)

C(–1, 1)

BA M45°

45°2

(h, k)

⇒ x2 + y2 + 2x – 2y = 0 is the reqd. locus.

21. (b) : CS = ae, SP = b ae

b a

abe

b a

⋅ −

+=

+

02 2 2 2

\ CP = CS SP2 2−

= a e b a ea b

2 22 2 2

2 2−+

= a e

a b

2

2 2+\ Area of rectangle with sides SP and CP = SP⋅CP

= ab a ea b

ab⋅+

=2 2

2 2 [ b2 = a2(e2 – 1)]

22. (b) : CB = b, CS = ae SBS = 90°\ CB = CS⇒ b = ae ⇒ b2 = a2e2

⇒ a2(1 – e2) = a2e2

⇒ 2e2 = 1 \ e = 12

23. (a) : Parabola is (x + y)2 = 5(x – y + 1) whose axis is x + y = 0.24. (c) : Foci of hyperbola y2 – x2 = 1 are (0, ±be) i.e., (0, ± 2)\ End points of a diameter of reqd. circle are ( , ) ( , )0 2 0 2and − .\ Eqn. of reqd. circle is (x – 0)(x – 0) + ( )( )y y− +2 2 = 0or x2 + y2 = 2

25. (d) : Any plane through (1, 2, –3) is given by a(x – 1) + b(y – 2) + c(z + 3) = 0 ...(i)

P

aeC S(ae, 0)

bx –

ay =

0(0,0)

S

45°

B

C S

x2

a2 + y2

b2 = 1

| JUNE ‘1738

If (i) is || to x-axis then a = 0If (i) passes through (2, –2, 1) then b(–2 – 2) + c(1 + 3) = 0 ⇒ b = c\ (i) becomes b(y – 2) + b(z + 3) = 0⇒ y + z + 1 = 0

26. (b) : Here, 1 1 12 3 01 0 3

−− = 1(–9 – 0) + 1(6 – 0) + 1(0 + 3)

= –9 + 6 + 3 = 0

\ 3 lines with d.r.s (1, –1, 1), (2, –3, 0) & (1, 0, 3) are coplanar.27. (a) : Given, f(x) is non-constant & di�erentiable

s.t.f xf y

( )( )

= f(x – y) ...(i)

Let f(x) = amx satisfying (i). Now, f (x) = mamx

Given f (0) = p ⇒ ma0 = p ⇒ m = p ...(ii)Also, f (5) = q ⇒ ma5m = q

⇒ a5m = qm

qp

= ...(iii)

\ f (–5) = mam(–5) = p⋅a–5m = p pq

⋅ [using (iii)]

= pq

2

28. (c) : f(x) = log5log3x = loge(log3x)log5e = log5e loge(log3x)

f (x) = loglog

(log )53

31e

xd

dxx⋅

= ⋅ ⋅loglog

(log log )53

31e

xd

dxx ee

= loglog

log log log53

3 51 1 1e

xe

xe e

xx⋅ ⋅ ⋅ = ⋅

\ f (e) = log loglog5

1 15

e ee ee

e⋅ =

29. (c) : H(x) = G(F(x)) = G(ex) = e–ex

\ dHdx

e ddx

e e ee x e xx x= ⋅ − = ⋅ −− −( ) ( )

\ dHdx

e e eex

e

= − = − = −=

− −

0

0 10 1( )

30. (c) : limx

f x f x f xx→

− +0 2

2 3 2 4( ) ( ) ( ) 0

0form

= limx

f x f x f xx→

′ − ′ + ′0

2 6 2 4 42

( ) ( ) ( )

00

form

= limx

f x f x f x→

″ − ″ + ″0

2 12 2 16 42

( ) ( ) ( )

= f (0) – 6f (0) + 8f (0) = 3f (0) = 3k

31. (a) : We have, y em x=−sin 1

⇒ dydx

e mx

my

xm x= ⋅ ⋅

−=

−sin 1 1

1 12 2

⇒ ( )1 22

2 2−

=x dy

dxm y

⇒ ( )1 2 222

2

2− ⋅ ⋅ − ⋅

x dy

dxd ydx

x dydx

= m y dydx

2 2⋅ ⋅

⇒ ( )1 22

22−

− ⋅ =x d ydx

x dydx

m y ⇒ k = m2

32. (d) : Using mean value theorem, f (c) = f b f ab a

( ) ( )−−

⇒ 2c + 2a = f f( ) ( )β αβ α

−−

= ( ) ( )β β α αβ α

2 22 2+ + − + +−

a b a b = + a + 2a

\ c = α β+2

33. (c) : f(x) = x13 + x11 + x9 + x7 + x5 + x3 + x + 19 f (x) = 13x12 + 11x10 + 9x8 + 7x6 + 5x4 + 3x2 + 1 > 0 ∀ x ∈ R\ f(x) is a strictly increasing function. f(– ) = – , f( ) = , f(0) = 19\ f(x) = 0, will have only one real root34. (b) : LMVT is applicable to f(x) in [0, x]\ f(x) must be continuous in [0, x]

\ limx

f x f→ +

=0

0( ) ( ) ⇒ limx

p

qx

x→ +=

00

(sin )

⇒ limh

p

qh

h→

++

=0

00

0( ){sin( )}

⇒ limh

p q

qh

hh

=0

0sin

⇒ limh

p qh→

−=

0 10 \ L.H.L exists only if p > q.

35. (b) : Let A = limx

xx→0

2(sin ) tan

log A = limx

x x→0

2 tan log(sin )

= 20

limx

xx→

log(sin )cot

∞∞

form

= 2

1

0 2limcosecxx

x

x→

−sin

cos= −

→limx

x0

2sin = 0 ⇒ A = 1

36. (c) : Let I = cos(log )x dx∫

= cos(log ) ( ) sin(log ) ( )x x xx

x dx⋅ − − ⋅ ⋅∫ 1

| JUNE ‘17 39

= x x x dxcos(log ) sin(log )+ ∫ = x cos(log x) + sin(log x)(x) – cos(log ) ( )x

xx dx⋅∫ 1

⇒ 2I = x[cos(log x) + sin(log x)]

⇒ I = x2

[cos(log x) + sin(log x)] + c

37. (a) : x

xdx

x xx

22

2 22

1 1

1 3

+ +

∫ = 1 1

1 1

2

2

+

+

∫ xdx

xx

= dzz

z xx2 11

+= +∫ , where

= tan–1 z + c = tan− +

+1 1x

xc

38. (b) : I = sin xx

dx1 8

10

19

+∫

\ I = sin sinxx

dx xx

dx1 18

10

19

810

19

+≤

+∫ ∫

⇒ I ≤ | sin |xx

dx dxx1 18

10

19

810

19

+≤

+∫ ∫ | | ≤ sin x 1

⇒ I ≤ dxx

x8

7

10

19

10

19

7=

= 17

10 19 107

107 77

7( )− −−

−− < <

39. (d) : I2 = { } { } { }x dx x dx n x dxn n

0 0

1

0

1

∫ ∫ ∫= =⋅

[ {x} is periodic with period = 1]

= n x dx n

0

1

2∫ =

I1 + I2 = ([ ] { })x x dx x dx x nn n n

+ = =

=∫ ∫

0 0

2

0

2

2 2

\ I1 = n n2

2 2− \

II

n nn

n1

2

21= − = −

40. (b) : limn

nn

nn

nn n→∞ +

++

+ ++

2 2 2 2 2 21 2

.....

= limn n

n nnn

→∞+

+

+

+ +

+

1 1

1 1

1

1 2

1

12 2 2.....

= limn r

n

n rn

→∞ = +

∑1 1

12

1= dx

xx

1 420

11

01

+= =∫ −[tan ] π

41. (d) : 0 ≤ x ≤ 1 \ 1 ≤ ex2 ≤ e

⇒ dx e dx e dxx

0

1

0

1

0

12

∫ ∫ ∫≤ ≤ ⋅

⇒ 12

0

1

≤ ≤∫ e dx ex

42. (c) : e dx e dx e dxx x x x−∫ ∫ ∫= =[ ]( )

{ }( )

{ }

0

100 1

0

100 1

0

1

100

[ {x} is periodic with period 1]

= 1000

1

e dxx∫ = 100(e – 1)

43. (a) : Putting x + y = z ⇒ 1+ =dydx

dzdx

Given eq. becomes z dzdx

a2 21⋅ −

=

⇒ dzdx

az

= +12

2

⇒ zz a

dz dx2

2 2+= ∫∫ ⇒

( )z a az a

dz x2 2 2

2 2+ −

+=∫

⇒ z aa

za

− ⋅ −2 11 tan = x – c

⇒ x + y – a tan–1 x ya

x c+

= −

⇒ y c

ax y

a+

=+

−tan 1 ⇒ x ya

y ca

+ = +

tan ,

c is an arbitrary constant.

44. (b) : We have, dydx

x xx x

y xx x

+ +−

⋅ = −−

( )( ) ( )

2

2 2

2

2 211

11

⇒ dydx

xx x

yx

+ +−

⋅ =2

2 211

1( )

\ I.F. = e e e xx

xx x

dxx

xx

dxx

xe

2

2

211

1 1

1 11

+−

+

− −

∫ ∫

= = ⋅ = −( )log

45. (b) : Let G.P. be a, ar, ar2, .....A.T.Q., a = ar + ar2 ⇒ r2 + r – 1 = 0

⇒ r = − ± + = − ±1 1 42

1 52

Terms are positive \ r < 0 \ r = − +1 52

46. (a) : Using property of logarithm, log5y = 3logxx = 3\ y = 53 = 125

| JUNE ‘1740

47. (c) : 1+−

−i

ii

n

11 2( ) = ( ) ( )1

11 2

2

22+

+ −ii

i in

= 1 1 22

2 2 1− +{ } − = − +i i in

n( )

48. (d) : Let z iz i

ki+−

= (k ∈R)

By componendo & dividendo, we have 22

11

zi

kiki

= +−

⇒ z kiki

i z k

ki= +

−⇒ = +

+ −=1

11

11

2 2

2 2.

( ). | |

⇒ x2 + y2 = 1 which represents a circle49. (a) : 2px2 + 2px + qx + q = 0⇒ 2px(x + 1) + q(x + 1) = 0

\ x = –1, −qp2

, which are rational as p & q are odd

integers.50. (b) : 3 out of 7 consonants can be chosen in 7C3 ways and 12 out of 4 vowels can be chosen in 4C2 ways\ Total no. of words that can be formed = 7C3 × 4C2 × 5 = 25200

51. (b) : A2 = 1 1 10 1 10 0 1

1 1 10 1 10 0 1

1 2 30 1 20 0 1

=

Only (b) is satis�ed by putting n = 2

52. (d) : D1 = a a ab b b

c c c

a b ca b ca b c

+ −− + −

− +=

−+ + −− − +

1 11 11 1

1 1 11 1 1

[Interchanging rows & columns]

= ( )−+ + −− − +

−=

+ + −− − +

−1

1 1 11 1 1

1 1 11 1 12

a b ca b c

a b c

a b ca b c

a b c

D2 = a b ca b c

a b cn n n

+ + −− − +

− − −+ +

1 1 11 1 1

1 1 12 1( ) ( ) ( )

= ( )−+ + −− − +

−1

1 1 11 1 1n

a b ca b c

a b c\ D1 + D2 = 0 is possible only when n is any odd integer.53. (c) : R ∪ S = {(1, 1), (2, 2), (3, 3), (1, 2), (2, 1), (1, 3), (3, 1)}A = {1, 2, 3}(i) (1, 1), (2, 2), (3, 3) ∈ R ∪ S ⇒ Re�exive(ii) (a, b) ∈ R ∪ S

⇒ (b, a) ∈ R ∪ S ∀ a, b {1, 2, 3} ⇒ Symmetric(iii) (2, 1) & (1, 3)∈ R ∪ S but (2, 3) R ∪ S⇒ Not transitive54. (a) : Let radius of circle with centre (1, 1) be aIts eqn. is (x – 1)2 + (y – 1)2 = a2 ...(i)Given circle is x2 + y2 – 4x – 6y + 9 = 0 ...(ii)Eqn. of common chord (S1 – S2 = 0) is 2x + 4y = a2 + 7 ...(iii)If (iii) be a diameter of (ii) then centre (2, 3) will lie on (iii)⇒ 4 + 12 = a2 + 7 ⇒ a2 = 9⇒ Radius = 3 units

55. (d) : tan = kh +1

& tan 2 = kh2 −

⇒ kh

kh

kh

22

1

21

11

2 2−=

−= +

−+

tantan

θθ

= 2 11 2 2

k hh k

( )( )

++ −

⇒ (h + 1)2 – k2

= 2(2 – h)(h + 1)⇒ h2 + 2h + 1 – k2

= 4h + 4 –2h2 – 2h

⇒ 3h2 – k2 = 3 ⇒ h k2 2

1 31− =

\ M moves on a hyperbola

56. (c) : f(x) = | | = | | + | |− −∫ ∫∫t dt t dt t dtx x

1 01

0

= | | + | | = − +∫ ∫ ∫∫−−

x dx x dx x dx x dxx x

0 1

0

01

0

( )

= −

+

= − − + −

= +

x x x xx2

1

0 2

0

2 2

2 212

0 12

0 12

( )

57. (b) : f(x) = limn

nk

kn x x

k→∞ →−( ) = −

1

01 1lim , where n = 1

k = loge x

Now, f x fx

x xe e e( ) log log / log+

= + = =1 1 1 0

\ f(xy) = log log loge e exy x y= + = f(x) + f(y)

58. (b) : I = 2 2

0

100

sin x dxπ

hO B(2, 0)

M(h, k)

A(–1, 0)

k

| JUNE ‘17 41

= 2 2 1000

100

0

|sin | |sin |x dx x dx= ⋅∫ ∫π π

[ Period of |sin x|is ]

= 100 2 100 20

0sin cosx dx xπ

π∫ = −[ ]

= 100 2 1 1 200 2( )+ =

59. (a) :

O(–2, 0)

y1

y2

(1, 0)

x = –2 x = 0 x = 1Parabolas are y2 = − x

2 ...(i) & y2 = − −1

31( )x ...(ii)

On solving, − x2

= − −( )x 13

⇒ –2x + 2 = – 3x ⇒ x = –2

\ Reqd. area = 2 22

1

12

0

y dx y dx− −∫ ∫−

[y1, y2 are values of y from (i) & (ii) resp.]

= 2 13 2

2

1

2

0− − −

− −

∫ ∫x dx x dx

= 23

132

22 3

2

3 2

2

1 3 2( ) ( )/ /−

− −

x x−2

0

= − −[ ]+ −[ ]43 3

0 3 43 2

0 23 2 3 2/ /

= 4 83

43

− = square units

60. (a) : We have,

x y2 2

9 51+ = ...(i)

e = 1 59

23

− =

\ L ae ba

, ,2

2 53

Eqn. of tangent to (i) at L 2 53

,

is

x y9

25

53

1⋅ + ⋅ = ⇒ x y9 2 3

1/

+ =

S AC

LB

L

\ CA CB= =92

3,

\ Reqd. area of quad. = 4 × Area of CAB

= 4 12

92

3× ⋅ ⋅ = 27 sq. units.

61. (c) : f(x) = sin x – cos x – Kx + 5

⇒ f (x) = cos x + sin x – K = 24

sin x K+

−π

24

2sin x +

≤π

\ f(x) will be decreasing for all +ve real x if f (x) < 0

24

0sin x K+

− <π

⇒ K > 24

sin x +

π ⇒ K > 2

62. (b) : Let � � � �x ai b j ck= + +\ � � � �x i bk c j× = − + ( ) | |� � � �x i x i b c× = × = +2 2 2 2

Similarly, ( ) & ( )� � � �x j c a x k a b× = + × = +2 2 2 2 2 2

\ ( ) ( ) ( ) ( ) | |� � � � � � �x i x j x k a b c x× + × + × = + + =2 2 2 2 2 2 22 263. (c) : Let a b c, , be three unit vectors such that c a b= +\ c a b2 2= +( ) ⇒ 1 = 1 + 1 + 2a b⋅⇒ 2 a b⋅ = −1Now, | |a b a b− = + − ⋅ =2 1 1 2 3 ⇒ | |a b− = 3 units.64. (a) : Roots of x2 + x + 1 = 0 are & 2

Let a = = − + = +1 32

23

23

i icos sinπ π

and = 2 = − − = −

+ −

1 32

23

23

i icos sinπ π

= cos sin23

23

π π− i \ α β πn n n+ = 2 23

cos

65. (c) : Let y x xx x

= + ++ +

2

22 4

2 4 9⇒ (2y – 1)x2 + 2(2y – 1)x + (9y – 4) = 0For real x, D ≥ 0 ⇒ 4(2y – 1)2 – 4(2y – 1)(9y – 4) ≥ 0⇒ 4y2 – 4y + 1 – 18y2 + 17y – 4 ≥ 0 ⇒ 14y2 – 13y + 3 ≤ 0

⇒ (7y – 3) (2y – 1) ≤ 0 ⇒ 37

12

≤ ≤y

⇒ Greatest value of y i.e. x xx x

2

22 4

2 4 9+ ++ +

is 12

66. (c, d) : a, b ∈ {1, 2, 3} & ax2 + bx + 1 = 0 has real roots⇒ D ≥ 0 i.e., b a2 4≥ ...(i)

| JUNE ‘1742

\ Possible ordered pair (a, b) are (1, 2), (1, 3) & (2, 3) only67. (a, c) : Equation of tangent to y2 = 4ax at (at2, 2at) is y ⋅ 2at = 2a (x + at2)⇒ ty = x + at2 ...(i)

For x2–y2 = a2, dydx

xy

=

⇒ At (a sec , a tan ), dydx

aa

= =sectan sin

θθ θ

1

\ Eqn. of normal to x2 – y2 = a2 at (a sec , a tan ) isy – a tan = – sin (x – a sec )⇒ y = –x sin + 2a tan ...(ii) (i) & (ii) are identical

\ t ata

t1

12 2

2 2=

−= =

sin tan tanθ θ θ⇒ t t= − =cosec θ θ& tan268. (c) : Conic may be written as (x – 3)2 = –4(y – 2)X2 = 4AY ...(i)Here, X = x – 3, Y = y – 2 & 4A = –4 ...(ii)For focus, X = 0, Y = A ⇒ x – 3 = 0, y – 2 = –1⇒ x = 3, y = 1 \ Focus is (3, 1)69. (b)70. (b,c) : f(x) = xn ...(i) f (a + ) = f (a) + f ( ) ...(ii)When n = 2, f(x) = x2 , f (x) = 2x\ f (a + ) = 2(a + ) = 2a + 2 =f (a) + f ( )⇒ (ii) is satis�edWhen n = 0, f(x) = 1 ⇒ f (x) = 0 ⇒ (ii) is satis�ed.(ii) is not satis�ed if n = 1 or 5

71. (c) : I = dxf x

a

10 +∫ ( ) ...(i)

= dxf a x

a

10 + −∫ ( ) = f x f a x

f x f a x f a xdx

a ( ) ( )( ) ( ) ( )

−− + −∫0

I = f x

f xdx

a ( )( ) +∫ 10

...(ii)

Adding (i) & (ii), 2I = dx aa

=∫0 ⇒ I = a

272. (b,d) : ax + by + c = 0 ...(i)

Its slope = −ab

(ab ≠ 0)

Now, xy = 1 –2x ⇒ y + 2 = 1x

...(ii)

Di�erentiating (ii), we get, dydx

= − 12x

\ dydx

< 0

If (i) be a tangent to (ii) then − = − <ab x

1 02

⇒ ba

x= >2 0

\ a & b must be of same sign\ a > 0, b > 0 or a < 0, b < 0

73. (b,c) : u ff= 0

0Dist. between them at any instant is given by

x = u t t ft⋅ − ⋅ +{ }0 12

2

dxdt

u f t u ft d xdt

f= − ⋅ ⋅ = − ⇒ = − <12

2 02

2

For max. distance between them, dxdt

= 0 ⇒ t = uf

Max. distance = u uf

f uf

uf

uf

uf

⋅ − ⋅ = − =12 2 2

2

2

2 2 2

74. (a, c) : |z – i| = |z + 1| = 1⇒ x2 + (y – 1)2 = (x + 1)2 + y2 = 1 ...(i)⇒ –2y + 1 = 2x + 1 ⇒ y = –x ...(ii)When y = –x, we have x2 + 2x + 1 + x2 = 1⇒ 2x(x + 1) = 0 ⇒ x = 0, – 1when x = 0 ⇒ y = 0when x = –1 ⇒ y = 1\ z = 0, –1 + i75. (c) : (i) 1 + a⋅a = 1 + a2 > 0 ⇒ Re�exive(ii) If 1 + ab > 0 then 1 + ba > 0 ⇒ Symmetric

(iii) 1 1 12

32

0 1 12

1 12

1 12

0 12

1

+

= > ⇒

+ −( ) = > ⇒ −

,

,

ρ

ρ

But, 1 + (1)(–1) = 0 /> 0⇒ (1, –1) \ is not transitive.

• Fame Book House - Imphal Mob: 9774220848, 7085556697 • Jain Book Shop - Imphal Mob: 9856031157 • Job Centre - Imphal Ph: 0385-2440140; Mob: 9856700700, 9436204282• P.C. Jain And Co. Ph: 0385-2451756, 2225004; Mob: 9856084649

MANIPUR at

| JUNE ‘17 43

1. 1 1 1

1p q rp q r +

is equal to

(a) q – p (b) q + p (c) q (d) p (e) 0

2. Let A = 5 01 0

and B =

0 11 0−

.

If 4A + 5B – C = O, then C is

(a) 5 251 0−

(b)

20 51 0−

(c) 5 10 25

(d)

5 251 5−

(e) 0 55 25

3. If U =

12

12

12

12

, then U–1 is

(a) UT (b) U (c) I (d) 0 (e) U2

4. If A =−

0 1 01 0 00 0 1

, then A–1 is

(a) AT (b) A2 (c) A (d) I (e) O

5. If x y x yx z x z+ −+ +

=

2

0 01 1

, then the values of x, y

and z are respectively

(a) 0, 0, 1 (b) 1, 1, 0 (c) –1, 0, 0 (d) 0, 0, 0(e) 1, 1, 1

6. 78

10

50

231

510

+

is equal to

(a) 1627

(b)

2716

(c) 1516

(d) 1615

(e) 1616

7. If 1 2 41 3 51 4 a

is singular, then the value of a is

(a) a = –6 (b) a = 5(c) a = –5 (d) a = 6 (e) a = 0

8. If 1 2 30 4 50 0 1

111

=

xyz

, then (x, y, z) is equal to

(a) (1, 6, 6) (b) (1, –6, 1)(c) (1, 1, 6) (d) (6, –1, 1)(e) (–1, 6, 1)

9. If A =

1 50 2

, then

(a) A2 – 2A + 2I = O (b) A2 – 3A + 2I = O(c) A2 – 5A + 2I = O (d) 2A2 – A + I = O (e) A2 + 3A + 2I = O

10. If 2 1 1

0 0x y x yp q p q

+ +− +

=

, then (x, y, p, q) equals

(a) 0, 1, 0, 0 (b) 0, –1, 0, 0(c) 1, 0, 0, 0 (d) 0, 1, 0, 1(e) 1, 0, 1, 0

SOLVED PAPER 2017

Kerala PET

| JUNE ‘1744

11. �e value of 4 2 3 4 2 3+ − − is

(a) 1 (b) 2 (c) 4 (d) 3(e) 5

12. �e value of 82/3 –161/4 –91/2 is(a) –1 (b) –2 (c) –3 (d) –4 (e) –5

13. Let x = 2 be a root of y = 4x2 –14x+q = 0. �en y is equal to(a) (x – 2) (4x – 6) (b) (x – 2) (4x + 6)(c) (x – 2) (–4x – 6) (d) (x – 2) (–4x + 6)(e) (x – 2) (4x + 3)

14. If x1 and x2 are the roots of 3x2 – 2x – 6 = 0, then

x21 + x2

2 is equal to

(a) 509

(b) 409

(c) 309

(d) 209

(e) 109

15. Let x1 and x2 be the roots of the equation x2 + px –3 = 0. If x2

1 + x22 =10, then the value of p is equal to

(a) –4 or 4 (b) –3 or 3 (c) –2 or 2 (d) –1 or 1 (e) 0

16. If the product of roots of the equation mx2 + 6x + (2m – 1) = 0 is –1, then the value of m is

(a) 13

(b) 1 (c) 3 (d) –1

(e) –3

17. If f xx x x x x x x

( ) ,=+ +

−+ +

++

14 4

44 4

422 4 3 2 3 2

then f 12

is equal to

(a) 1 (b) 2 (c) –1 (d) 3 (e) 4

18. If x and y are the roots of the equation x2 + bx + 1 = 0,

then the value of 1 1x b y b+

++

is

(a) 1b

(b) b (c) 12b

(d) 2b

(e) 1

19. �e equations x5 + ax + 1 = 0 and x6 + ax2 + 1 = 0 have a common root. �en a is equal to(a) –4 (b) –2 (c) –3 (d) –1 (e) 0

20. �e roots of ax2 + x + 1 = 0, where a ≠ 0, are in the ratio 1 : 1. �en a is equal to

(a) 14

(b) 12

(c) 34

(d) 1

(e) 0

21. If z2 + z + 1 = 0 where z is a complex number, then

the value of zz

zz

zz

+

+ +

+ +

1 1 122

2

23

3

2

equals (a) 4 (b) 5 (c) 6 (d) 7 (e) 8

22. Let ∆ = − −

1 1 1

1 1

1

2 2

4

w w

w w

, where w ≠ 1 is a complex

number such that w3 = 1. �en equals(a) 3w + w2 (b) 3w2

(c) 3(w – w2) (d) –3w2

(e) 3w2 +1

23. If 3 9 12 9 1

10 9

i ii

i

−− = x + iy, then

(a) x = 1, y = 1 (b) x = 0, y = 1(c) x = 1, y = 0 (d) x = 0, y = 0 (e) x = –1, y = 0

24. If z = cos π π3 3

i sin , then z2 – z +1 is equal to

(a) 0 (b) 1 (c) –1 (d) π2

(e)

25. 1

12 12

112 12

+

+

+

cos sin

cos sin

π π

π π

i

i

72

is equal to

(a) 0 (b) – (c) 1 (d) 12

(e) − 12

26. If Ak kk k

k=

400 0

and det (A) = 256, then k equals

(a) 4 (b) 5 (c) 6 (d) 7 (e) 8

| JUNE ‘17 45

27. If A =

1 01 1

, then An + nI is equal to

(a) I (b) nA (c) I + nA (d) I – nA (e) nA – I

28. If |z| = 5 and w zz

= −+

55

, then Re(w) is equal to

(a) 0 (b) 125

(c) 25 (d) 1 (e) –1

29. If A =

1 11 1

, then A2017 is equal to

(a) 22015A (b) 22016A (c) 22014A (d) 22017A (e) 22020A

30. If a = ei , then 11

+−

aa

is equal to

(a) cot θ2

(b) tan

(c) icot θ2

(d) i tan θ2

(e) 2 tan 31. �ree numbers x, y and z are in arithmetic

progression. If x + y + z = – 3 and xyz = 8, then x2 + y2 + z2 is equal to(a) 9 (b) 10 (c) 21 (d) 20 (e) 1

32. �e 30th term of the arithmetic progression 10, 7, 4, .... is (a) –90 (b) –87 (c) –77 (d) –67 (e) –57

33. �e arithmetic mean of two numbers x and y is 3 and geometric mean is 1. �en x2 + y2 is equal to(a) 30 (b) 31 (c) 32 (d) 33 (e) 34

34. �e solution of 32x–1 = 811–x is

(a) 23

(b) 16

(c) 76

(d) 56

(e) 13

35. �e sixth term in the sequence is 3, 1, 13

, ... is

(a) 127

(b) 19

(c) 181

(d) 117

(e) 17

36. �ree numbers are in arithmetic progression. �eir sum is 21 and the product of the �rst number and the third number is 45. �en the product of these three numbers is(a) 315 (b) 90 (c) 180 (d) 270 (e) 450

37. If a+1, 2a + 1, 4a – 1 are in arithmetic progression, then the value of a is (a) 1 (b) 2 (c) 3 (d) 4 (e) 5

38. Two numbers x and y have arithmetic mean 9 and geometric mean 4. �en x and y are the roots of(a) x2 –18x –16 = 0 (b) x2 –18x +16 = 0(c) x2 +18x –16 = 0 (d) x2 +18x +16 = 0(e) x2 –17x +16 = 0

39. �ree unbiased coins are tossed. �e probability of getting at least 2 tails is

(a) 34

(b) 14

(c) 12

(d) 13

(e) 23

40. A single letter is selected from the word TRICKS. �e probability that it is either T or R is

(a) 136

(b) 14

(c) 12

(d) 23

(e) 13

41. From 4 red balls, 2 white balls and 4 black balls, four balls are selected. �e probability of getting 2 red balls is

(a) 721

(b) 821

(c) 921

(d) 1021

(e) 1121

42. In a class, 60% of the students know lesson I, 40% know lesson II and 20% know lesson I and lesson II. A student is selected at random. �e probability that the student does not know lesson I and lesson II is

(a) 0 (b) 45

(c) 35

(d) 15

(e) 25

43. Two distinct numbers x and y are chosen from 1, 2, 3, 4, 5. �e probability that the arithmetic mean of x and y is an integer is

| JUNE ‘1746

(a) 0 (b) 15

(c) 35

(d) 25

(e) 45

44. �e number of 3 3 matrices with entries –1 or +1 is(a) 24 (b) 25 (c) 26 (d) 27

(e) 29

45. Let S be the set of all 2 × 2 symmetric matrices whose entries are either zero or one. A matrix X is chosen from S. �e probability that the determinant of X is not zero is

(a) 13

(b) 12

(c) 34

(d) 14

(e) 29

46. �e number of words that can be formed by using all the letters of the word PROBLEM only once is(a) 5! (b) 6! (c) 7! (d) 8!

(e) 9!

47. �e number of diagonals in a hexagon is(a) 8 (b) 9 (c) 10 (d) 11

(e) 12

48. �e sum of odd integers from 1 to 2001 is(a) 10012 (b) 10002 (c) 10022 (d) 10032

(e) 9992

49. Two balls are selected from two black and two red balls. �e probability that the two balls will have no black ball is

(a) 17

(b) 15

(c) 14

(d) 13

(e) 16

50. If z = i9 + i19, then z is equal to(a) 0 + 0i (b) 1 + 0i (c) 0 + i (d) 1 + 2i(e) 1 + 3i

51. �e mean for the data 6, 7, 10, 12, 13, 4, 8, 12 is(a) 9 (b) 8 (c) 7 (d) 6

(e) 5

52. �e set of all real numbers satisfying the inequality x – 2 < 1 is(a) (3, ) (b) [3, ) (c) [–3, ) (d) (– , –3)(e) (– , 3)

53. If xx

−−

>33

0, then

(a) x ∈(–3, ) (b) x ∈(3, )(c) x ∈(2, ) (d) x ∈(1, )(e) x ∈(–1, )

54. �e mode of the data 8, 11, 9, 8, 11, 9, 7, 8, 7, 3, 2, 8 is(a) 11 (b) 9 (c) 8 (d) 3

(e) 7

55. If the mean of six numbers is 41, then the sum of these numbers is(a) 246 (b) 236 (c) 226 (d) 216

(e) 206

56. If f t dt x e xxx ( ) ( ),= + >∫ 20 0 then f(1) is equal to

(a) 1 + e (b) 2 + e (c) 3 + e (d) e

(e) 0

57. xx

dx+ =∫1

1 2/

(a) –x3/2 + x1/2 + c (b) x1/2

(c) 23

23 2 1 2x x c/ /+ + (d) x3/2 + x1/2 + c

(e) x3/2

58. In a �ight 50 people speak Hindi, 20 speak English and 10 speak both English and Hindi. �e number of people who speak at least one of the two languages is (a) 40 (b) 50 (c) 20 (d) 80

(e) 60

59. If f x xx

( ) ,= +−

11

then the value of f(f(x)) is equal to

(a) x (b) 0 (c) –x (d) 1

(e) 2

60. Two dice are thrown simultaneously. What is the probability of getting two numbers whose product is even?

(a) 34

(b) 14

(c) 12

(d) 23

(e) 116

61. limx

x xx→

+ − −0

2 2is equal to

(a) 12

(b) 2

(c) 0 (d) Does not exist

(e) 12 2

| JUNE ‘17 47

62. dxe ex x+ +−∫

2 is equal to

(a) 11e

cx ++ (b)

−+

+11e

cx

(c) 1

1++−e

cx (d) 1

1ecx− −

+

(e) 1

1ecx −

+

63. tan tanπ θ π θ4 2 4 2

+

+ −

is equal to

(a) sec (b) 2 sec (c) sec θ2

(d) sin

(e) cos

64. dxx x21

0

2+ +−∫ is equal to

(a) π4

(b) π2

(c) (d) 0

(e) –

65. sinsin cos

xx x+∫02

πdx is equal to

(a) 0 (b) – (c) 32π (d) π

2

(e) π4

66. If (x, y) is equidistant from (a + b, b – a) and (a – b, a + b), then(a) x + y = 0 (b) bx – ay = 0(c) ax – by = 0 (d) bx + ay = 0(e) ax + by = 0

67. If the points (1, 0), (0, 1) and (x, 8) are collinear, then the value of x is equal to (a) 5 (b) –6 (c) 6 (d) 7

(e) –7

68. �e minimum value of the function max{x, x2} is equal to

(a) 0 (b) 1 (c) 2 (d) 12

(e) 32

69. Let f(x + y) = f(x) f (y) for all x and y. If f(0) = 1, f(3) = 3 and f (0) = 11, then f (3) is equal to(a) 11 (b) 22 (c) 33 (d) 44

(e) 55

70. If f(9) = f (9) = 0, then lim ( )x

f xx→

−−9

33

is equal to

(a) 0 (b) f(0) (c) f (3) (d) f(9)

(e) 1

71. �e value of cos cosπ π4 4

+

+ −

x x is

(a) 2 2sin x (b) 2 sin x (c) 2 2cos x (d) 3 cos x(e) 2 cos x

72. Area of the triangle with vertices (–2, 2), (1, 5) and (6, –1) is

(a) 15 (b) 35

(c) 292

(d) 332

(e) 352

73. �e equation of the line passing through(–3, 5) and perpendicular to the line through the points (1, 0) and (–4, 1) is(a) 5x + y + 10 = 0 (b) 5x – y + 20 = 0(c) 5x – y – 10 = 0 (d) 5x + y + 20 = 0(e) 5y – x – 10 = 0

74. �e coe�cient of x5 in the expansion of (1 + x2)5 (1 + x)4 is(a) 30 (b) 60 (c) 40 (d) 10

(e) 4575. �e coe�cient of x4 in the expansion of (1 – 2x)5 is

equal to(a) 40 (b) 320 (c) –320 (d) –32

(e) 8076. �e equation 5x2 + y2 + y = 8 represents

(a) an ellipse (b) a parabola(c) a hyperbola (d) a circle (e) a straight line

77. �e centre of the ellipse 4x2 + y2 –8x + 4y – 8 = 0 is(a) (0, 2) (b) (2, –1) (c) (2, 1) (d) (1, 2)(e) (1, –2)

78. �e area bounded by the curves y = –x2 + 3 and y = 0 is(a) 3 1+ (b) 3 (c) 4 3 (d) 5 3

(e) 6 379. �e order of the di�erential equation

d ydx

d ydx

dydx

3

3

2 2

2

2 5

+

+

= 0 is

| JUNE ‘1748

(a) 3 (b) 4 (c) 1 (d) 5

(e) 6

80. If f(x) = 2 42

xx

+ , then f (2) is equal to

(a) 0 (b) –1 (c) 1 (d) 2

(e) –2

81. �e area of the circle x2 – 2x + y2 – 10y + k = 0 is 25 . �e value of k is equal to(a) –1 (b) 1 (c) 0 (d) 2

(e) 3

82. xx x

dx+ −∫ 40332016

2017 is equal to

(a) 14

(b) 32

(c) 20172

(d) 12

(e) 508

83. �e solution of dydx

y+ tan x = sec x, y(0) = 0 is

(a) y sec x = tan x (b) y tan x = sec x (c) tan x = y tan x (d) x sec x = tan y

(e) y cot x = sec x

84. If the vectors 2 2 6 2 6 2 3i j k i j k i j k+ + + + − +, ,λ are coplanar, then the value of is(a) –10 (b) 1 (c) 0 (d) 10

(e) 2

85. �e distance between (2, 1, 0) and 2x + y +2z + 5 = 0 is

(a) 10 (b) 103

(c) 109

(d) 5

(e) 1

86. �e equation of the hyperbola with vertices (0, ± 15) and foci (0, ± 20) is

(a) x y2 2

175 2251− = (b) x y2 2

625 1251− =

(c) y x2 2

225 1251− = (d) y x2 2

65 651− =

(e) y x2 2

225 1751− =

87. �e value of 15 6 3 6 15 211 4 6 6 36 4 216 1296

3 3+ + ⋅ ⋅ ⋅+ + + +( ) ( ) ( )

is equal to

(a) 297

(b) 719

(c) 617

(d) 2119

(e) 277

88. �e equation of the plane that passes through the points (1, 0, 2), (–1, 1, 2), (5, 0, 3) is(a) x + 2y – 4z + 7 = 0 (b) x + 2y – 3z + 7 = 0(c) x – 2y + 4z + 7 = 0 (d) 2y – 4z – 7 + x = 0(e) x + 2y + 3z + 7 = 0

89. �e vertex of the parabola y2 – 4y – x + 3 = 0 is(a) (–1, 3) (b) (–1, 2) (c) (2, –1) (d) (3, –1)(e) (1, 2)

90. If a b c, , are vectors such that a b c+ + = 0 and

a b c= = =7 5 3, , , then the angle between

c band is

(a) π3

(b) π6

(c) π4

(d)

(e) 0

91. Let f x x ax a x( ) ,= − + +2 9 12 13 2 2 where a > 0. �e minimum of f is attained at a point q and the maximum is attained at a point p. If p3 = q, then a is equal to(a) 1 (b) 3 (c) 2 (d) 2

(e) 12

92. For all real numbers x and y, it is known that the real valued function f satis�es f(x) + f (y) = f(x + y).

If f(1) = 7, then f rr ( )=∑ 1100

is equal to(a) 7 51 102 (b) 6 50 102(c) 7 50 102 (d) 6 25 102(e) 7 50 101

93. �e eccentricity of the ellipse

( )x y− + +

=12

34

116

2 2 is

(a) 12

(b) 1

2 2 (c) 1

2 (d) 1

4

(e) 14 2

94. max{ , }x x dx31

1−∫ is equal to

(a) 34

(b) 14

(c) 12

(d) 1

(e) 0

95. If x y∈ ∈02

02

, , ,π π

and sin x + cos y = 2, then

the value of x + y is equal to

| JUNE ‘17 49

(a) 2 (b) (c) π4

(d) π2

(e) 0

96. Let a, a + r and a + 2r be positive real numbers such that their product is 64. �en the minimum value of a + 2r is equal to(a) 4 (b) (c) 2 (d)

12

(e) 1

97. �e sum S = + + + +19

13 7

15 5

17 3

19! ! ! ! ! ! ! !

is equal to

(a) 28

10

! (b)

210

9

! (c) 2

10

7

! (d)

210

6

!

(e) 28

5

!

98. If f x

x x x

x xx

( ) ,=

2 3

21 2 30 2 6

then f (x) is equal to

(a) x3 + 6x2 (b) 6x3 (c) 3x (d) 6x2 (e) 0

99. xx

dx2

3 21+∫

( ) is equal to

(a) tan–1 (x2) + c (b) 23

1 3tan ( )− +x c

(c) 13

1 3tan ( )− +x c (d) 12

1 2tan ( )− +x c

(e) tan–1 (x3) + c

100. Let fn(x) be the nth derivative of f(x). �e least value of n so that fn = fn+1 , where f(x) = x2 + ex is

(a) 4 (b) 5 (c) 2 (d) 3 (e) 6

101. sin 765° is equal to

(a) 1 (b) 0 (c) 32

(d) 12

(e) 12

102. �e distance of the point (3, –5) from the line 3x – 4y – 26 = 0 is

(a) 37

(b) 25

(c) 75

(d) 35

(e) 1

103. �e di�erence between the maximum and minimum value of the function

f x t t dtx( ) ( )= + +∫ 20 1 on [2, 3] is

(a) 396

(b) 496

(c) 596

(d) 696

(e) 796

104. If a and b are the non zero distinct roots of x2 + ax + b = 0, then the minimum value of x2 + ax + b is

(a) 23

(b) 94

(c) −94

(d) −23

(e) 1105. If the straight line y = 4x + c touches the ellipse

x y2

24

1+ = then c is equal to

(a) 0 (b) ± 65 (c) ± 62 (d) ± 2 (e) ± 13

106. �e equations x – y = 2, 2x – 3y = – and 3x – 2y = –1 are consistent for

(a) = – 4 (b) = 1, 4(c) = 1, – 4 (d) = –1, 4(e) = –1

107. �e set {(x, y):|x|+|y|=1} in the xy plane represents(a) a square (b) a circle(c) an ellipse (d) a rectangle which is not a square(e) a rhombus which is not a square

108. The value of cos tan−

1 34

is

(a) 45

(b) 35

(c) 34

(d) 25

(e) 0109. Let A(6, –1), B(1, 3) and C(x, 8) be three points

such that AB = BC. �e values of x are(a) 3, 5 (b) –3, 5 (c) 3, –5 (d) 4, 5(e) –3, –5

110. In an experiment with 15 observations on x, the

following results were available x2 2830∑ = and

x∑ = 170 . One observation that was 20, was found

to be wrong and was replaced by the correct value 30. �en the corrected variance is

(a) 9.3 (b) 8.3 (c) 188.6 (d) 177.3 (e) 78

| JUNE ‘1750

111. �e angle between the pair of lines x y z− = − = +

−2

21

53

3and x y z+

−= − = −2

14

85

4 is

(a) cos−

1 219 38

(b) cos−

1 239 38

(c) cos−

1 249 38

(d) cos−

1 259 38

(e) cos−

1 269 38

112. Let a be a unit vector. If ( ) ( ) ,x a x a− ⋅ + = 12 then the magnitude of x is

(a) 8 (b) 9 (c) 10 (d) 13 (e) 12

113. �e area of the triangular region whose sides are y = 2x + 1, y = 3x + 1 and x = 4 is

(a) 5 (b) 6 (c) 7 (d) 8 (e) 9

114. If nCr–1 = 36, nCr = 84 and nCr+1 = 126, then the value of r is

(a) 9 (b) 3 (c) 4 (d) 5 (e) 6

115. Let f(x + y) = f(x) f(y) and f(x) = 1 + sin(3x)g(x), where g is di�erentiable. �en f (x) is equal to

(a) 3f(x) (b) g(0) (c) f(x)g(0) (d) 3g(x) (e) 3f(x)g(0)

116. �e roots of the equation x

xx

−−

−=

1 1 11 1 11 1 1

0 are

(a) 1, 2 (b) –1, 2 (c) –1, –2 (d) 1, –2 (e) 1, 1

117. If the 7th and 8th term of the binomial expansion

(2a –3b)n are equal, then 2 32 3a ba b

+−

is equal to

(a) 131

−+

nn

(b) n

n+−

113

(c) 613

−−

nn

(d) n

n−−

113

(e) 2 113

nn

−−

118. Standard deviation of �rst n odd natural numbers is

(a) n (b) ( )( )n n+ +2 13

(c) n2 13− (d) n

(e) 2n

119. Let S = {1, 2, 3, ..., 10}. �e number of subsets of S containing only odd numbers is

(a) 15 (b) 31 (c) 63 (d) 7 (e) 5

120. �e area of the parallelogram with vertices (0, 0), (7, 2) (5, 9) and (12, 11) is

(a) 50 (b) 54 (c) 51 (d) 52 (e) 53

SOLUTIONS

1. (a) : We have,

1 1 1

1p q rp q r +

= 1 1 1 1 1 0

01

p q rp q r

p qp q

+

= 0 + 1 1 0

01

p qp q

( R2 ~ R3 in 1st determinant)

= 1 1 0

01

p qp q

= 1(q – p)

2. (b) : Given, A = 5 01 0

and B =

0 11 0−

Now, 4A + 5B – C = O

⇒ C = 4A + 5B = 45 01 0

+ 5

0 11 0−

= 20 04 0

0 55 0

+

=

20 51 0−

3. (a) : Given, U = 1 2 1 2

1 2 1 2

/ /

/ /

Since, U–1 = 1

| |U adj(U)

| JUNE ‘17 51

Hence, adj (U) = 1 2 1 2

1 2 1 2

12

12

12

12

/ /

/ /

=−

T

|U| = 12

12

12

12

− − ⋅

= 12

+ 12

= 1

\ U–1 = 1 2 1 2

1 2 1 2

/ /

/ /−

= UT

4. (a) : We have, A = 0 1 01 0 00 0 1

Hence, adj(A) = 0 1 01 0 0

0 0 1

0 1 01 0 00 0 1

=−

T

Now, |A| = 0 + (1)(–1) = –1

\ A–1 = 0 1 01 0 0

0 0 1−

= AT

5. (a) : We have, x y x yx z x z+ −+ +

2

= 0 01 1

On comparing, we get x + y = 0 ...(i), x – y = 0 ...(ii)2x + z = 1 ...(iii), x + z = 1 ...(iv)On solving (iii) and (iv), we get x = 0, z = 1.Hence, from (i), we get y = 0\ x = 0, y = 0, z = 1.

6. (b) : We have, 7 1 58 0 0

231

+ 510

= 14 3 516 0 0

50

+ ++ +

+

= 2216

50

+

= 2716

7. (d) : Let A = 1 2 41 3 51 4 a

Since A is singular matrix \ |A| = 0

Hence, 1 2 41 3 51 4 a

= 0

⇒ 1(3a – 20) – 2(a – 5) + 4(4 – 3) = 0 ⇒ a = 6

8. (d) : We have, 1 2 30 4 50 0 1

xyz

= 111

⇒ x y z

y zz

+ −+

2 34 5 =

111

On comparing, we getx + 2y – 3z = 1 ...(i) 4y + 5z = 1 ...(ii) and z = 1 ...(iii)On solving (i), (ii) and (iii), we get z = 1, y = –1 and x = 6

9. (b) : We have, A = 1 50 2

A2 = A ⋅ A = 1 50 2

1 50 2

= 1 150 4

3A = 31 50 2

= 3 150 6

2I = 21 00 1

= 2 00 2

\ A2 – 3A + 2I = 1 150 4

– 3 150 6

+ 2 00 2

= 0 00 0

= O

10. (a) : We have, 2x y x yp q p q

+ +− +

= 1 10 0

On comparing, we get 2x + y = 1 ...(i), x + y = 1 ...(ii) p – q = 0 ...(iii), p + q = 0 ...(iv)On solving (i) and (ii), we get x = 0 and y = 1And, on solving (iii) and (iv), we get p = 0 and q = 0\ (x, y, p, q) = (0, 1, 0, 0)

11. (b) : Let | | | |4 2 3 4 2 3+ − − = xOn squaring both sides, we get

4 2 3 4 2 32

2+ − −( ) = x

⇒ (4 + 2 3) + (4 – 2 3) – 2( )4 2 3+ ( )4 2 3− = x2

⇒ 8 – 2 16 12− = x2 ⇒ 8 – 4 = x2

⇒ x2 = 4 ⇒ x = 2

Hence, | | | |4 2 3 4 2 3+ − − = ±2

| JUNE ‘1752

12. (a) : We have, 82/3 – 161/4 – 91/2

= ((2)3)2/3 – ((2)4)1/4 – ((3)2)1/2

= 4 – 2 – 3 = 4 – 5 = –113. (a) : Given, x = 2 is a root of y = 4x2 – 14x + q = 0 then, x = 2 satisfy the given equation i.e., y(2) = 0⇒ 4(2)2 – 14(2) + q = 0⇒ 16 – 28 + q = 0 ⇒ q = 12\ y = 4x2 – 14x + 12 = 0⇒ y = 4x2 – 6x – 8x + 12⇒ y = 2x(2x – 3) – 4(2x – 3)⇒ y = (2x – 4) (2x – 3) ⇒ y = (x – 2)(4x – 6)14. (b) : We have, 3x2 – 2x – 6 = 0\ Roots of given equation (x1, x2)

= − − ± +( )2 4 726

= 2 766

± = 2 2 196

±

= 1 193

±

Hence, x1 = 1 19

3+

and x2 = 1 193

Now, x12 + x2

2 = 1 193

1 193

2 2+

+ −

= 19

(1 + 19 + 2 19 + 1 + 19 – 2 19) = 409

15. (c) : Given, x1 and x2 are roots of x2 + px – 3 = 0.Also, x1

2 + x22 = 10

\ Sum of roots = (x1 + x2) = − p1

= –p

And, product of roots = x1x2 = −31

= – 3 \ (x1 + x2)2 = p2

⇒ x12 + x2

2 + 2x1x2 = p2 ⇒ 10 + 2(–3) = p2

⇒ p2 = 4 ⇒ p = 2Hence, the value of p is 2 or –2.16. (a) : Given, mx2 + 6x + (2m – 1) = 0 and product of roots is – 1.

⇒ ( )2 1mm

− = –1 ⇒ 2m – 1 = – m ⇒ 3m = 1 ⇒ m = 13

17. (e) : We have,

f(x) = 14 42x x+ +

– 44 44 3 2x x x+ +

+ 423 2x x+

= 1

2 2( )x + –

422 2x x( )+

+ 4

22x x( )+

\ f(1/2) = 1

12

22

+

– 4

12

12

22 2

+

+ 4

12

12

22

+

= 1

52

2

– 4

14

52

2

+ 4

14

52

= 425

– 6425

+ 325

= − +60 16025

= 10025

= 4

18. (b) : We have, x2 + bx + 1 = 0 and x, y are its roots.\ Sum of roots = (x + y) = – b ...(i)And product of roots = (xy) = 1 ...(ii)

Now, 1x b+

+ 1y b+

= y b x bx b y b

+ + ++ +( )( )

= ( )

( )

x y b

xy b x y b

+ ++ + +

22 =

( )− +− +

b bb b

21 2 2

(Using (i) and (ii)) = b19. (b) : Let y be the common root of x5 + ax + 1 = 0 and x6 + ax2 + 1 = 0�en, y5 + ay + 1 = 0 and y6 + ay2 + 1 = 0⇒ y5 + ay + 1 = y6 + ay2 + 1⇒ y5 – y6 + ay – ay2 = 0⇒ y5(1 – y) + ay(1 – y) = 0⇒ (y5 + ay) (1 – y) = 0 ⇒ y = 1Hence, the common root is 1.i.e., 1 + a + 1 = 0 ⇒ a = –220. (a) : Let x1 and x2 be the roots of ax2 + x + 1 = 0 then,

x1 + x2 = −1a

...(i) and x1x2 = 1a

...(ii)

Also, x1 : x2 = 1 : 1 ⇒ x1 = x2 ...(iii)

Using (iii) in (i), we get 2x1 = −1a

⇒ x1 = −12a

Using (iii) in (ii), we get x12 = 1

a \ 1

4 2a = 1

a

⇒ 4a = 1 ⇒ a = 14

21. (c) : We have, z2 + z + 1 = 0⇒ z = or 2

where , 2 are complex cube roots of unity.

Now, zz

+

1 2 + z

z2

2

21+

+ zz

33

21+

= ωω

+

1 2 + ω

ω2

2

21+

+ ω

ω3

3

21+

| JUNE ‘17 53

= ωω

2 21+

+ ωω

4

2

21+

+ (1 + 1)2

[using 3 = 1]

= −

ωω

2

+ −

ω

ω

2

2

2

+ 4 [using 1 + + 2 = 0]

= 1 + 1 + 4 = 6

22. (b) : We have, =

1 1 1

1 1

1

2 2

4

− − w w

w w

⇒ =

1 1 1

1 11

2 2− − w ww w

=

1 1 1

11

2w ww w

( 1 + w + w2 = 0)Applying C1 → C1 – C2 and C2 → C2 – C3 , we get

=

0 0 1

11 0

2 2− −−

w w w ww w

= 1(0 – (1 – w) (w – w2)) (on expanding along R1) = –(w – w2 – w2 + w3) = –(–1 – w2 – w2 – w2 + 1) = –(–3w2) = 3w2

23. (d) : We have, 3 9 12 9 1

10 9

i ii

i

−− = (x + iy)

⇒ 3i(9i2 + 9) + 9i(2i + 10) + (18 – 90i) = x + iy⇒ 3i(– 9 + 9) + 18i2 + 90i + 18 – 90i = x + iy⇒ – 18 + 18 = x + iy⇒ 0 + 0i = x + iyOn comparing, we get x = 0 and y = 0

24. (a) : We have, z = cos π3

– i sin π3

= 12

– 32

i =

1 32

− i

then, z2 – z + 1 = 1 3

2

2−

i –

1 32

i + 1

= 14

(1 – 3 – 2 3 i) + 12

+ 32

i

= − − + +12

32

12

32

i i = 0

25. (c) : Let z = cos sinπ π12 12

+

i

\ 1z

= cos sinπ π12 12

i

Now, from given expression, we have

1

12 12

112 12

+

+

+

cos sin

cos sin

π π

π π

i

i

72

= 11 1

72++

zz

= ( )( )1

1

72++

z zz

= (z)72 = cos sinπ π12 12

72

+

i

= cos sin7212

7212

π π

+

i

(Using De-Moivre's theorem) = cos 6 + i sin 6 = 1

26. (e) : We have, A = 400 0

k kk k

kNow, expanding along C1, we get |A| = 4(k2)But det(A) = 256 (Given)\ On comparing, we get 4k2 = 256 ⇒ k2 = 64 ⇒ k = ±8Hence, |k| = 8

27. (c) : We have, A = 1 01 1

\ A2 = 1 01 1

1 01 1

= 1 02 1

A3 = 1 02 1

1 01 1

= 1 03 1

\ An = 1 0

1n

Hence, An + nI = 1 0

1n

+ n1 00 1

= 1 0

1n

+ n

n0

0

= 1 0

1+

+

nn n

= 1 00 1

+ nn n

0

= 1 00 1

+ n1 01 1

= I + nA28. (a) : Let z = x + iy

\ |z| = 5 ⇒ x y2 2+ = 5 ⇒ x2 + y2 = 25

| JUNE ‘1754

Now, w = zz

−+

55

= x iyx iy

+ −+ +

55

= ( )( )x iyx iy

− ++ +

55

On rationalizing the denominator, we get

(( ) )(( ) )

( ) ( )

x iy x iy

x y

− + + −+ +

5 5

5 2 2 = x y yi

x y

2 2

2 225 10

5

− + ++ +( )

= 10

5 2 2yi

x y( )+ + [Using x2 + y2 = 25]

= 0 + 10

5 2 2yi

x y( )+ + \ R(w) = 0

29. (b) : We have, A = 1 11 1

\ A2 = A ⋅ A = 1 11 1

1 11 1

= 2 22 2

= 21 11 1

= 2A

A3 = A2 ⋅ A = 2 22 2

1 11 1

= 4 44 4

= 41 11 1

= 22 ⋅ A

Similarly An = 2n – 1 ⋅ A\ A2017 = 22017 – 1 ⋅ A = 22016 ⋅ A30. (c) : We have, a = eiq = cos + i sin (polar form)

\ 11

+−

aa

= 1

1+ +

− +cos sin

(cos sin )θ θθ θ

ii

= 2

22

2 2

22

22 2

2

2

cos sin cos

sin sin cos

θ θ θ

θ θ θ

+

i

i

= 2

2 2 2

22 2 2

cos cos sin

sin sin cos

θ θ θ

θ θ θ

+

i

i

= cotcos sin sin cos

sin cos

θθ θ θ θ

θ θ22 2 2 2

2 2

+

× +

i i

i

+

sin cosθ θ

2 2i

[Rationalizing the denominator]

= cotcos sin cos sin cos sin

sin cos

θθ θ θ θ θ θ

θ θ22 2 2 2 2 2

2 2

2 2

2 2

+ + −

+

i i

= i cot θ2

31. (c) : We have, x + y + z = – 3 ...(i)On squaring both sides, we get (x + y + z)2 = (–3)2

⇒ x2 + y2 + z2 + 2xy + 2yz + 2zx = 9 ...(ii)Also, x, y, z are in A.P. ⇒ 2y = x + z⇒ 2y = – 3 – y ⇒ y = – 1 ...(iii)So, xyz = 8 ⇒ xz = – 8 ...(iv)Now, using (iii) and (iv) in (ii), we get x2 + y2 + z2 – 2x – 2z – 16 = 9⇒ x2 + y2 + z2 – 2(x + z) – 16 = 9⇒ x2 + y2 + z2 + 4 – 16 = 9⇒ x2 + y2 + z2 = 9 + 12 = 21Hence, x2 + y2 + z2 = 2132. (c) : We have the series of an A.P. as 10, 7, 4....\ First term (a) = 10Common di�erence (d) = 7 – 10 = – 3\ a30 = a + (30 – 1)d = 10 + 29(–3) = 10 – 87 = – 7733. (e) : Given, arithmetic mean of x and y is 3

i.e., x y+2

= 3 ⇒ x + y = 6 ...(i)

and geometric mean of x and y is 1i.e., xy = 1 ⇒ xy = 1 ...(ii)Squaring (i) on both sides, we get (x + y)2 = (6)2 ⇒ x2 + y2 + 2xy = 36⇒ x2 + y2 + 2 = 36 (Using (ii))⇒ x2 + y2 = 3434. (d) : We have, 32x – 1 = 811 – x

⇒ (3)2x – 1 = ((3)4)1 – x ⇒ (3)2x – 1 = (3)4 – 4x

\ On comparing, we get, 2x – 1 = 4 – 4x

⇒ 6x = 5 or x = 56

35. (c) : Given series 3, 1, 13

, .... forms a G.P.

where �rst term (a) = 3 and common ratio (r) = 13

\ Sixth term, a6 = ar5

= (3)13

5

=

13

4

= 1

8136. (a) : Let three numbers in A.P. be a, b and c�en, according to question a + b + c = 21 ...(i) and ac = 45 ...(ii) a, b and c are in A.P. \ 2b = c + a ...(iii)

| JUNE ‘17 55

Substituting (iii) in (i), we get 3b = 21 ⇒ b = 7Hence, product of these three numbers = abc = 7(45) = 315 (Using (ii))37. (b) : Given, a + 1, 2a + 1, 4a – 1 are in A.P.\ 2(2a + 1) = 4a – 1 + a + 1⇒ 4a + 2 = 5a ⇒ a = 238. (b) : Given, arithmetic mean of x and y is 9

i.e., x y+2

= 9 ⇒ x + y = 18

Geometric mean of x and y is 4 i.e., xy = 4 ⇒ xy = 16Now, sum of roots (x + y) = 18Product of roots (xy) = 16\ Required quadratic equation is x2 – 18x + 16 = 039. (c) : Total number of outcomes = 8 i.e.,{HHH, HHT, HTH, THH, TTH, THT, HTT, TTT} Number of favourable outcomes = 4 i.e., {TTH, THT, HTT, TTT} = 4

\ P(getting atleast 2 tails) = 48

= 12

40. (e) : Number of letters in TRICKS = 6Number of favourable outcomes = {T, R} = 2

\ P(either T or R) = 26

= 13

41. (c) : Total number of outcomes = 10C42 red balls can be selected from 4 red balls in 4C2 waysAnd, remaining 2 balls can be selected from 2 white balls and 4 black balls in 6C2 ways.

\ Required probability = 4

26

210

4

C C

C

× = 3

7 or 9

21

42. (d) : Let A and B be two events of knowing lesson I and lesson II respectively.\ According to question,

P(A) = 60100

; P(B) = 40100

P(A ∩ B) = 20100

\ P(A ∪ B) = P(A) + P(B) – P(A ∩ B)

= 60100

+ 40100

– 20100

= 80100

Hence, required probability = P(AC ∩ BC) = P(A ∪ B)C = 1 – P(A ∪ B)

= 1 – 80100

= 20100

= 15

43. (d) : Two distinct numbers can be chosen from 1, 2, 3, 4, 5 in 5C2 ways.

Number of outcomes having arithmetic mean an integer i.e., {(1, 3), (1, 5) (2, 4), (3, 5)} = 4

\ Required probability = 4 4

1052C

= = 25

44. (e) : �ere are 9 elements in 3 × 3 matrices and each element can be �lled in two ways either – 1 or 1.\ Total possible matrices = 29

45. (b) : Total possible set of 2 × 2 symmetric matrices of entries either zero or one = 8Possbile set of matrices having determinant not zero = 4

\ Required probability = 48

= 12

46. (c) : Number of words that can be formed by using all 7 letters of word PROBLEM only once is 7!.47. (b) : Number of diagonals in n-sided polygon

= n n( )− 32

\ Number of diagonals in hexagon = 6 6 32

( )− = 9

48. (a) : We know sum of n odd numbers = n2

Number of odd terms from 1 to 2001 = 1001\ Sum of 1001 odd terms = (1001)2

49. (e) : Total number of ways of selecting two balls = 4C2Number of ways of selecting 2 balls in which no ball is black = 2C2

\ Required probability = 2

24

2

C

C =

16

50. (a) : We have, z = i9 + i19 = (i2)4 ⋅ i + (i2)9 ⋅ i = i + (–i) = 0 = 0 + 0i 51. (a) : Given data is 6, 7, 10, 12, 13, 4, 8, 12

\ Mean = 6 7 10 12 13 4 8 128

+ + + + + + + = 728

= 9

52. (e) : Given, x – 2 < 1 ⇒ x < 1 + 2 = 3Hence, set of all real numbers satisfying the inequality x – 2 < 1 is (– , 3)

53. (b) : Given, | |xx

−−

33

> 0

⇒ |x – 3| > 0 ⇒ x – 3 > 0 ⇒ x > 3\ x ∈ (3, )54. (c) : From the given data 8 has highest frequency.\ Mode of the given data is 8.55. (a) : Let the six numbers be x1, x2, x3, x4, x5, x6Given, mean of six numbers = 41

\ Mean = x x x x x x1 2 3 4 5 6

6+ + + + +

= 41

⇒ x1 + x2 + x3 + x4 + x5 + x6 = 246

| JUNE ‘1756

56. (b) : Given, f t dtx

( )0∫ = x2 + ex (x > 0)

⇒ f(x) = 2x + ex

\ f(1) = 2(1) + e = 2 + e

57. (c) : xx

dx+∫

11 2/ = x dx

xdx∫ ∫+ 1

= 23

x3/2 + 2x1/2 + c

58. (e) : Let H and E be the two events of people speaking Hindi and English respectively.\ n(H) = 50, n(E) = 20 n(H ∩ E) = 10\ n(H ∪ E) = n(H) + n(E) – n(H ∩ E) = 50 + 20 – 10 = 60i.e., number of people who speak atleast one of two languages is 60.

59. (a) : Given, f(x) = xx

+−

11

\ f(f(x)) = f xx

+−

11 =

xxxx

+−

+

+−

11

1

11

1

= ( )( )x xx x

+ + −+ − +

1 11 1

= 22x = x

60. (a) : Total number of outcomes = 6 × 6 = 36Favourable number of outcomes = 27

\ Required probability = 2736

= 34

61. (a) : We have, limx

x xx→

+ − −0

2 2

On rationalizing the numerator, we get

lim ( ) ( )( )

lim( )x x

x xx x x

xx x x→ →

+ − −+ + −

=+ + −0 0

2 22 2

22 2

=+ + −

=→

limx x x0

22 2

22 2

= 12

62. (b) : We have, dx

e ex x+ +−∫ 2 =

e dxe e

x

x x2 1 2+ +∫

= e dx

edtt

x

x( )+=∫ ∫1 2 2

[Put (ex + 1) = t ⇒ exdx = dt]

= −1t

+ c = −

+1

1ex + c

63. (b) : We have, tan π θ4 2

+

+ tanπ θ4 2

= tan tan

tan tan

π θ

π θ4 2

14 2

+

− +

tan tan

tan tan

π θ

π θ4 2

14 2

+

= 1

2

12

+

tan

tan

θ

θ + 1

2

12

+

tan

tan

θ

θ

= 1

21

2

12

2 2

2

+

+ −

tan tan

tan

θ θ

θ =

2 12

12

2

2

+

tan

tan

θ

θ

= 2 1cosθ

= 2 sec

64. (None of the options is correct) :

We have, dxx x2

1

0

2+ +−∫

= dx

x x21

0

2 14

14

+ + + −−∫ =

dx

x +

+−∫

12

74

21

0

= dx

x +

+

−∫

12

72

2 21

0 =

17 2

1 27 2

1

1

0

tan //

+x

= 27

1 27 2

1 27 2

1 1tan //

tan //

− −

− −

= 27

17

17

1 1tan tan− −+

= 47

17

1tan−

65. (e) : Let I = sinsin cos

xx x

dx+∫

0

...(i)

⇒ I = sin

sin cos

π

π π

π2

2 20

2 −

+ −

∫x dx

x x

⇒ I = cos

cos sin

x dx

x x+∫0

...(ii)

Adding (i) and (ii), we get

| JUNE ‘17 57

2I = sin cossin cos

x xx x

dx++

∫0

⇒ I = 12

10

2

⋅∫ dxπ

= 12 2

π

= π4

66. (b) : Given, (x, y) is equidistant from (a + b, b – a) and (a – b, a + b)\ Using distance formula, we have

( ( )) ( ( ))x a b y b a− + + − −2 2

= ( ( )) ( ( ))x a b y a b− − + − +2 2

On squaring both sides, we get (x – (a + b))2 + (y – (b – a))2 = (x – (a – b))2 + (y – (a + b))2

⇒ x2 + a2 + b2 + 2ab – 2ax – 2bx + y2 + b2 + a2

– 2ab – 2by + 2ay = x2 + a2 + b2 – 2ab + 2bx – 2ax + y2 + a2 + b2

+ 2ab – 2by – 2ay⇒ – 4bx + 4ay = 0 ⇒ ay – bx = 0 or bx – ay = 067. (e) : Given, (1, 0), (0, 1) and (x, 8) are collinear\ Area of formed by these points is zero

i.e., 12

|1(1 – 8) + 0(8 – 0) + x(0 – 1)| = 0

⇒ 12

|– 7 – x| = 0 ⇒ x = – 7

68. (a) : Graph of max{x, x2} is shown below

Y

1

y = x2

y = x2

y = x

0 X

Hence, min value of max{x, x2} is 0.69. (c) : Given, f(x + y) = f(x) f(y)

f (3) = lim( ) ( )

h

f h fh→

+ −0

3 3

= lim( ) ( ) ( )

h

f f h fh→

−0

3 3 = ff h

hh( ) lim

( )3

10→

= ff h f

hh( ) lim

( ) ( )3

00→

− [ f(0) = 1]

= ff h f

hh( ) lim

( ) ( )3

0 00→

+ − = f(3) f (0)

= 3 × 11 = 33 \ f (3) = 33

70. (a) : We have, f f x fxx

′ = −−→

( ) lim ( ) ( )9 999

⇒ =−

×−−→

09

339

lim ( ) ( )( )x

f xx

f xf x

⇒ =−

− +× + −

→0

33 3

9 939

lim( )

( )( )( )

( )x

f xx x

f xf x

⇒ =−

×

( ) −

− +( )

→ →0

33

3

3 39 9

2 2lim

( )( )

lim( ) ( )

( ) (x x

f xx

f x

f x x

− +

lim( ( ) ) ( )x f x x9

93 3

⇒ =+

×→

03

30lim

( )x

f xx

lim

( )( ( ) )( )x

f xx f→

++

−− +

9

33

99 3 9 3

⇒ =+

× +×

03

336

93 69

lim( )

x

f xx

⇒+

=→

lim( )

x

f xx9

33

0

71. (e) : cos π4

+

x + cos π4

x

= cos π4

cos x – sin π4

sin x + cos π4

cos x

+ sin π4

sin x

= 2 cos π4

cos x = 22

cos x = 2 cos x

72. (d) : Area of triangle

= 12

|(–2)(5 + 1) + 1(– 1 – 2) + 6(2 – 5)|

= 12

|(– 2)(6) – 3 – 18| = 12

|–12 – 3 – 18|

= 12

| – 33| = 332

sq. units

| JUNE ‘1758

73. (b) : Equation of line passing through (1, 0) and (–4, 1) is

y −−

01 0

= x −− −

14 1

⇒ y = x −−

15

or x + 5y – 1 = 0Now, equation of line perpendicular to x + 5y – = 0 is 5x – y + = 0 is for some constant Also, 5x – y + = 0 passes through (– 3, 5)\ 5(– 3) – 5 + = 0 ⇒ = 20Hence, 5x – y + 20 = 0 is the required equation of line74. (b) : Given expansion is (1 + x2)5 (1 + x)4

= [5C0(x2)0 + 5C1(x2)1 + 5C2(x2)2 + 5C3(x2)3 + 5C4(x2)4 +5C5(x2)5] [4C0x0 + 4C1x + 4C2x2 + 4C3x3 + 4C4x4]

= [5C0 + 5C1x2 + 5C2x4 + 5C3x6 + 5C4x8 + 5C5x10]

[4C0 + 4C1x + 4C2x2 + 4C3x3 + 4C4x4]\ Coe�cient of x5 = 5C1 ⋅ 4C3 + 5C2 ⋅ 4C1

= 20 + 40 = 60

75. (e) : Given expansion is (1 – 2x)5 = 5C0 – 5C1(2x) + 5C2(2x)2 – 5C3(2x)3

+ 5C4(2x)4 – 5C5(2x)5

\ Coe�cient of x4 = 5C4 . (2)4 = 8076. (a) : We have, 5x2 + y2 + y = 8

⇒ 5x2 + y2 + y – 8 + 14

– 14

= 0

⇒ ( 5 x)2 + (y + 1/2)2 – 334

= 0

⇒ ( 5 x)2 + (y + 1/2)2 = 332

2

which represents an ellipse.77. (e) : We have, 4x2 + y2 – 8x + 4y – 8 = 0⇒ 4(x2 – 2x + 1 – 1) + (y2 + 4y + 4 – 4) – 8 = 0⇒ 4(x – 1)2 – 4 + (y + 2)2 – 4 – 8 = 0

⇒ ( )/

x −11 4

2 + (y + 2)2 = 16

⇒ ( )x −14

2 + ( )y + 2

16

2 = 1

\ Centre (1, –2).

78. (c) :

y

(0, 3)

( 3 , 0)O

y = –x2 + 3

x

y

x( − 3 , 0)

Required area = 2 32

0

3( )− +∫ x dx

= 23

33

0

3− +

x x = 2 3 33

3 3− +

= 2(2 3 ) = 4 3

79. (a) : Order of di�erential equation

d y

dx

3

3

2

+ d y

dx

2

2

2

+

dydx

5

= 0 is 3.

80. (a) : We have, f(x) = 2x + 42x

\ f (x) = 2 ⋅ 12 x

+ 42

⋅ −

12 3 2x /

= 12x

– 23 2( ) /x

⇒ f (2) = 12

– 12

= 0

81. (b) : Let r be the radius of given circle.Given, area of circle x2 – 2x + y2 – 10y + k = 0 is 25i.e., r2 = 25 ⇒ r2 = 25 ...(i)Also, radius from the given equation is

r = ( ) ( )1 52 2+ − k ⇒ r2 = 26 – k

⇒ 25 = 26 – k [Using (i)]⇒ k = 1

82. (d) : Let I = x

x xdx

+ −∫ 40332016

2017 ...(i)

Also, I = 403340332016

2017 −− +∫

xx x

...(ii)

Adding (i) and (ii), we get

| JUNE ‘17 59

2I = x x

x xdx dx

+ −( )+ −( ) = ⋅∫ ∫

4033

40331

2016

2017

2016

2017

⇒ I = 12

[2017 – 2016] = 12

83. (a) : We have, dydx

+ y tan x = sec x, y(0) = 0

�is is a linear di�erential equation

\ I.F. = e x dxtan∫ = elog|sec x| = sec x

\ Solution is given by y ⋅ sec x = sec secx x dx⋅∫⇒ y ⋅ sec x = sec2 x dx∫ ⇒ y sec x = tan x + cNow, we have y(0) = 0

⇒ (0)·sec 0 = tan (0) + c ⇒ c = 0

\ Particular solution is, y sec x = tan x

84. (e) : Since the vectors 2 2 6i j k+ + , 2 6i j k+ +λ ,

2 3i j k− + are coplanar.

\ 2 2 62 62 3 1

λ−

= 0

⇒ 2( + 18) – 2(2 – 12) + 6(– 6 – 2 ) = 0⇒ – 10 + 20 = 0 ⇒ = 285. (b) : Distance of the point (2, 1, 0) from the plane 2x + y + 2z + 5 = 0 is given by

( ) ( ) ( )2 2 1 1 0 2 5

2 1 22 2 2

⋅ + ⋅ + ⋅ +

+ + =

4 1 53

+ + = 10

3

86. (e) : Coordinates of vertices of hyperbola (0, 15) = (0, b)and foci (0, 20) = (0, be)\ b = 15 and be = 20

⇒ e = 2015

= 43

Now, e2 = 1 + ab

2

2 ⇒ 169

= 1 + a2

225

⇒ a2 = 79

× 225 = 175

\ Equation of hyperbola y

bxa

2

2

2

2− = 1

i.e., y2

225 – x2

175 = 1

87. (e) : Given, 15 6 3 6 15 211 4 6 6 36 4 216 1296

3 3+ + ⋅ ⋅ ⋅+ + + +( ) ( ) ( )

= ++

( )( )15 61 6

3

4 =

( )( )217

3

4 = 277

88. (a) : �e equation of the plane passing through the points (1, 0, 2), (–1, 1, 2) and (5, 0, 3) is

x y z− − −− − − −

− − −

1 0 21 1 1 0 2 2

5 1 0 0 3 2 = 0 ⇒

x y z− −−

1 22 1 0

4 0 1 = 0

⇒ (x – 1) (1) – y(–2) + (z – 2)(–4)⇒ x – 1 + 2y – 4z + 8 = 0⇒ x + 2y – 4z + 7 = 089. (b) : Given equation is y2 – 4y – x + 3 = 0⇒ (y – 2)2 – x – 1 = 0 ⇒ (y – 2)2 = x + 1Now, shi�ing the origin to the point (–1, 2) without rotating the axes and denoting the new coordinates w.r.t. these axes by X and Y, we get y – 2 = Y and x + 1 = XNow, substituting (X = 0, Y = 0) y = 2, x = –1Hence, vertex w.r.t to old axes (–1, 2).

90. (a) : Given, a b c+ + = 0 ⇒ a = − +( )b c ⇒ | |a 2 = | ( ) |− +b c 2

⇒ | |a 2 = | |b 2 + | |c 2 + 2 | | | |b c cos ( is angle between b and c )

⇒ 49 = 25 + 9 + 30 cos

⇒ cos = 1530

= 12

⇒ = π3

91. (d) : Let f(x) = 2x3 – 9ax2 + 12a2x + 1\ f (x) = 6x2 – 18ax + 12a2

Now, f (x) = 0 ⇒ 6x2 – 18ax + 12a2 = 0⇒ (x – 2a) (x – a) = 0⇒ x = a or 2aAlso, f (x) = 12x – 18a\ f (a) = 12a – 18a = – 6a < 0 (a > 0)and f (2a) = 24a – 18a = 6a > 0Hence, p = a and q = 2aNow, p3 = q ⇒ a3 = 2a ⇒ a(a2 – 2) = 0

⇒ a = ± 292. (e) : We have, f(x) + f(y) = f(x + y) Put x = y = 1, we get f(1 + 1) = f(1) + f(1) ⇒ f(2) = 7 + 7 = 14 f(2 + 1) = f(2) + f(1) ⇒ f(3) = 14 + 7 = 21

| JUNE ‘1760

Continuing in the same way, we get f (4) = 28, f (5) = 35 and so on.

\ f rr

( )=∑

1

100 = f (1) + f (2) + f (3) +...+ f (100)

= 7 + 14 + 21 +.....+ 700Since, the series forms an A.P.

\ f rr

( )=∑

1

100= 100

2[7 + 700] = 50 × 707 = 50 × 7 × 101

93. (a) : Given, ( )x y− + +

12

34

2 2 = 1

16

or ( )/

x −11 8

2 +

( / )/

y + 3 41 16

2 = 1

\ Eccentricity (e) = 12

2− ba

= 1 1 161 8

− ( / )( / )

= 1 12

− = 12

94. (b) : We have, max{ , }x x dx3

1

1

−∫

= x dx3

1

0

−∫ + x dx

0

1

∫ = x4

1

0

4

+ x2

0

1

2

= 14

95. (d) : We have, sin x + cos y = 2It is possible only if sin x = 1 and cos y = 1

⇒ x = π2

and y = 0 \ x + y = π2

+ 0 = π2

96. (a) : Given numbers a, a + r, a + 2r are in A.P. Also their product = 64. �is is possible only when three numbers are equal to 4.i.e., a = a + r = a + 2r = 4\ Minimum value of a + 2r is 4.

97. (b) : We have, S = 19

13 7

15 5

17 3

19! ! ! ! ! ! ! !

+ + + +

= 19

2 2 93

9 8 7 65! ! !

+ ×8× + × × ×

= 19

2 120 2 72 20 3024120!

( )( ) ( )× + +

=

1 614410 12

××!

= 210

9

!

98. (d) : Given, f(x) =

x x x

x xx

2 3

21 2 30 2 6

= x(12x2 – 6x2) – x2(6x) + x3(2 – 0)= 6x3 – 6x3 + 2x3 = 2x3 \ f (x) = 6x2

99. (c) : Let I = xx

dx2

3 21+∫ ( )Put x3 = t ⇒ 3x2dx = dt

\ I = 13 1 2

dtt+∫ = 1

3[tan–1 t] + c = 1

3 tan–1(x3) + c

100. (d) : We have, f(x) = x2 + ex

\ f (x) = 2x + ex

f (x) = 2 + ex

f (x) = ex

Hence, the least value of n so that fn = fn + 1 is 3.101. (e) : sin 765° = sin(2 × 360° + 45°)

= sin 45° = 12

102. (d) : Distance of the point (3, –5) from the line 3x – 4y – 26 = 0 is

3 3 4 5 26

3 42 2

⋅ + − − −

+

( )( ) = 9 20 26

5+ − = 3

5

103. (c) : Given, f(x) = ( )t t dtx

2

01+ +∫ = t t t

x3 2

03 2

+ +

\ f(x) = x x x

3 2

3 2+ +

\ f(x) = x x x

3 2

3 2+ +

on [2, 3] gives

f(2) = 83

+ 42

+ 2 = 203

(minimum)

and f(3) = 273

+ 92

+ 3 = 332

(maximum)

\ Di�erence between the maximum and minimum

value is 332

– 203

= 596

104. (c) : Let, f(x) = x2 + ax + b = 0Since a and b are roots of f(x) \ a + b = –a and ab = b ⇒ a = 1\ 1 + b = – 1 ⇒ b = –2So, f(x) = x2 + x – 2Also, f (x) = 2x + 1. For maximum/minimum f (x) = 0

⇒ x = −12

Now, f (x) = 2 > 0

| JUNE ‘17 61

\ x = −12

is the minimum point.

\ Minimum value = f −

=1

2 −

+ −

12

12

2 – 2

= 14

– 12

– 2 = −94

105. (b) : We have, y = 4x + c and x2

4 + y2 = 1

y = 4x + c touches the given ellipse

\ x2

4 + (4x + c)2 = 1

⇒ x2 + 4(16x2 + c2 + 8xc) = 4⇒ x2 + 64x2 + 4c2 + 32xc – 4 = 0⇒ 65x2 + 32xc + 4c2 – 4 = 0Now, discriminant D = 0 ⇒ (32c)2 – 4(65)(4c2 – 4) = 0⇒ 1024c2 – 1040c2 + 1040 = 0⇒ 16c2 = 1040 ⇒ c2 = 65⇒ c = ± 65106. (d) : Given equations x – y = 2, 2x – 3y = – and 3x – 2y = – 1 are consistent

\ λ

λ− −−−

1 22 33 2 1

= 0

⇒ (–3 + 2 ) + 1(2 – 3 ) – 2(–4 + 9) = 0⇒ –3 + 2 2 + 2 – 3 + 8 – 18 = 0⇒ 2 – 3 – 4 = 0 ⇒ ( – 4)( + 1) = 0\ = –1, 4107. (a) : �e set {(x, y) : |x| + |y| = 1} in xy plane represents

x y x yx y x y

x y x yx y x y

+ = > >− = > <

− + = < >− − = < <

1 0 01 0 0

1 0 01 0 0

; ,; ,

; ,; ,

i.e.,

(0, 1)

(1, 0)

(0, –1)

(–1, 0)Ox

y

y

x

x – y = 1

x + y = 1–x + y

= 1

–x – y = 1

Hence, the given set represents a square.

108. (a) : Let tan–1 34

= ⇒ tan = 3

4

\ cos = 45

Now, cos tan−

1 34

= cos = 45

109. (b) : AB = BC\ By using distance formula, we have

( ) ( )1 6 3 12 2− + + = ( ) ( )x − + −1 8 32 2

On squaring both sides, we get (–5)2 + (4)2 = (x – 1)2 + (5)2

⇒ x2 + 1 – 2x = 16 ⇒ x2 – 2x – 15 = 0⇒ (x + 3)(x – 5) = 0 ⇒ x = –3, 5110. (e) : We have, n = 15, Incorrect x2 = 2830,Incorrect (x) = 170\ Correct (x) = (Incorrect x – Incorrect value)

+ Correct value = (170 – 20) + 30 = 180

\ Correct Mean = Correct Σx15

= 18015

= 12

Similarly, Correct x2 = Incorrect x2 – (Incorrect value)2

+ (Correct value)2

= 2830 – (20)2 + (30)2 = 2830 + 500 = 3330

\ Correct variance = Correct ( )Σxn

2 – (Correct mean)2

= 333015

– (12)2 = 222 – 144 = 78

111. (e) : Let be the angle between the lines

x − 22

= y −15

= z +−

33

and x +−

21

= y − 48

= z − 54

\ cos = | ( ) ( ) ( )( ) |

( ) ( ) ( ) ( ) ( ) ( )

2 1 5 8 3 4

2 5 3 1 8 42 2 2 2 2 2

− + + −

+ + − − + +

= | |− + −2 40 12

38 81 =

269 38

i.e., = cos–1 269 38

112. (d) : We have, ( ) ( )x a x a− ⋅ + = 12⇒ | | | |x a2 2− = 12Since, a be a unit vector \ | |a = 1⇒ | |x 2 – 1 = 12 ⇒ | |x 2 = 13 ⇒ | |x = 13113. (d) : Sides of triangular region are y = 2x + 1; y = 3x + 1 and x = 4.

| JUNE ‘1762

(4, 13)

(0, 1)

(–1/2, 0)

O

A

B(4, 9)

x = 4y = 2x + 1y = 3x

+ 1

X

Y

X

C

\ Area of shaded region i.e., ABC where A = (0, 1), B = (4, 9), C = (4, 13)

= 12

|0(9 – 13) + 4(13 – 1) + 4(1 – 9)| = 12

|4(12) – 32|

= 12

× 16 = 8 sq. units

114. (b) : Given, nCr – 1 = 36, nCr = 84, nCr + 1 = 126

Since, n

rn

r

C

Cn r

r−= − +

1

1

\ 8436

= n rr

− +1 ⇒ n rr

− +1 = 73

⇒ 7r = 3n – 3r + 3 ⇒ 10r = 3n + 3 ...(i)

Also, n

rn

r

C

C +1 = r

n r+−

1 ⇒ 84126

= rn r

+−

1 ⇒ rn r

+−

1 = 23

⇒ 3r + 3 = 2n – 2r ⇒ 5r = 2n – 3 ...(ii)Solving (i) & (ii), we get r = 3

115. (e) : f (x) = lim( ) ( )

h

f x h f xh→

+ −0

= lim( ) ( ) ( )

h

f x f h f xh→

−0

[ f(x + y) = f(x)f(y)]

= f xf h

hh( ) lim

( )→

−0

1 = f xh g hhh

( ) limsin ( )

+ −0

1 3 1

= f x g h hhh

( ) lim ( ) sin→0

3

= f x g hhh

( ) ( ) lim sin0 330→

× 3 = 3 f(x) g(0)

116. (b) : Given, x

xx

−−

1 1 11 1 11 1 1

= 0

⇒ (x – 1)[(x – 1)2 –1] – 1[x – 1 – 1] + 1[1 – x + 1] = 0⇒ (x – 1)[x2 + 1 – 2x – 1] – x + 2 + 2 – x = 0⇒ (x – 1) x(x – 2) – 2x + 4 = 0⇒ x(x – 1)(x – 2) – 2(x – 2) = 0⇒ (x – 2)[x2 – x – 2] = 0 ⇒ (x – 2)(x – 2)(x + 1) = 0⇒ x = 2, –1

117. (None of the options is correct) : Given, T7 = T8⇒ nC6(2a)n – 6 (–3b)6 = nC7(2a)n – 7 (–3b)7

⇒ n

n!

( )! !− 6 6(2a)n – 6 (–3b)6 =

nn

!( )! !− 7 7

(2a)n – 7 (–3b)7

⇒ 1

6( )n −(2a) = ( )−3

7b ⇒ 2

3ab

= − −( )n 67

Applying componendo & dividendo, we get

2 32 3

6 76 7

131

131

a ba b

nn

nn

nn

+−

= − +− −

= −− −

= −+

118. (c) : Mean of �rst n odd natural numbers

x nn

nn

n= + + + + − = =1 3 5 2 1 2.... ( )

Sum of square of �rst n odd natural numbers i.e.,

12 + 32 + 52 + ... + (2n – 1)2 = n n n( )( )2 1 2 13

+ −

\ Standard deviation = n n n

nn( )( )2 1 2 1

32+ − −

= n2 1

3−

119. (b) : Given, S = {1, 2, 3, ..., 10}. \ Set containing odd numbers of S = {1, 3, 5, 7, 9}\ Number of subsets of S containing only odd numbers = (2)5 – 1 = 32 – 1 = 31120. (e) : Let the vertices of parallelogram be A(0, 0), B(7, 2), C(5, 9) and D(12, 11)

C

B

D

A

(12, 11) (5, 9)

(0, 0) (7, 2)Area of || gm ABCD = area of ABC + area of ADC

Now, area of ABC = 12

|0(2 – 9) + 7(9 – 0) + 5(0 – 2)|

= 12

|63 – 10| = 532

\ Area of ADC = 12

|0(11 – 9) + 12(9 – 0) + 5(0 – 11)|

= 12

|108 – 55| = 532

\ Area of || gm ABCD = 532

532

+ = 53 sq. units

| JUNE ‘17 63

1. �e distance of the point (–2, 4, –5) from the line x y z+ = − = +3

34

58

6 is

(a) 3710

(b) 3710

(c) 3710

(d) 3710

2. If A is a square matrix of order 3 × 3, then |KA| is equal to(a) K|A| (b) K2|A| (c) 3K|A| (d) K3|A|

3. Equation of line passing through the point (1, 2) and perpendicular to the line y = 3x – 1 is(a) x – 3y = 0 (b) x + 3y = 0(c) x + 3y – 7 = 0 (d) x + 3y + 7 = 0

4. General solution of di�erential equation dydx

y y+ = ≠1 1( ) is

(a) log 11 −

= +y

x C (b) log|1 – y| = x + C

(c) log|1 + y| = x + C (d) log 11 −

= − +y

x C

5. �e value of C in mean value theorem for the function f(x) = x2 in [2, 4] is

(a) 2 (b) 4 (c) 72

(d) 3

6. �e value of lim coscosθ

θθ→

−−0

1 41 6 is

(a) 94

(b) 49

(c) 93

(d) 34

7. If y x xx x

dydx

= +−

−tan sin coscos sin

, then1 is equal to

(a) 0 (b) 12

(c) π4 (d) 1

8. If 11

1+−

=i

i

m, then the least positive integral

value of m is(a) 4 (b) 1 (c) 2 (d) 3

9. | |x dx+−∫ 25

5 is equal to

(a) 28 (b) 29 (c) 27 (d) 30

10. cos coscos cos

2 2xx

dx−−∫

θθ

is equal to

(a) 2(sinx + x cos ) + C (b) 2(sinx – x cos ) + C (c) 2(sinx + 2x cos ) + C (d) 2(sinx – 2x cos ) + C

11. �e area of the region bounded by the curve y = x2 and the line y = 16 is

(a) 2563

sq . units (b) 128

3sq . units

(c) 323

sq . units (d) 643

sq . units

12. If A and B are �nite sets and A B, then(a) n(A ∪ B) = n(B) (b) n(A ∩ B) = n(B)(c) n(A ∩ B) = f (d) n(A ∪ B) = n(A)

13. If a matrix A is both symmetric and skew symmetric, then(a) A is diagonal matrix (b) A is a zero matrix(c) A is scalar matrix (d) A is square matrix

14. If 3

13 24 1

xx

= then x is equal to

(a) 8 (b) 4 (c) ± 2 2 (d) 2

Karnataka CETSOLVED PAPER 2017

| JUNE ‘1764

15. �e integrating factor of the di�erential equation

x dydx

y x⋅ + =2 2 is (x ≠ 0)

(a) elog x (b) log |x|(c) x (d) x2

16. �e perpendicular distance of the point P(6, 7, 8) from XY-plane is(a) 7 (b) 6 (c) 8 (d) 5

17. �e shaded region in the �gure is the solution set of the inequations

(a) 5x + 4y ≤ 20, x ≤ 6, y ≤ 3, x ≥ 0, y ≥ 0 (b) 5x + 4y ≥ 20, x ≤ 6, y ≥ 3, x ≥ 0, y ≥ 0 (c) 5x + 4y ≥ 20, x ≤ 6, y ≤ 3, x ≥ 0, y ≥ 0 (d) 5x + 4y ≥ 20, x ≥ 6, y ≤ 3, x ≥ 0, y ≥ 0

18. If an LPP admits optimal solution at two consecutive vertices of a feasible region, then(a) the required optimal solution is at the midpoint

of the line joining two points. (b) the optimal solution occurs at every point on

the line joining these two points.(c) the LPP under consideration is not solvable.(d) the LPP under consideration must be

reconstructed.

19. 3 + 5 + 7 + .... to n terms is(a) n2 (b) n(n – 2)(c) n(n + 2) (d) (n + 1)2

20. If 21 30

01 2

5 61 8x

y

+

=

, then the value of

x and y are(a) x = 3, y = 3 (b) x = –3, y = 3 (c) x = 3, y = –3 (d) x = –3, y = –3

21. �e derivative of cos–1(2x2 – 1) w.r.t cos–1 x is

(a) 2 (b) 2x

(c) 1 – x2 (d) −

1

2 1 2x

22. A box has 100 pens of which 10 are defective. �e probability that out of a sample of 5 pens drawn one by one with replacement and atmost one is defective is

(a) 9

10 (b) 1

29

10

4

(c) 910

12

910

5 4

+

(d) 12

910

5

23. If y = log (log x), then d ydx

2

2 is equal to

(a) ( log )

log1

2+ x

x x (b)

− +( log )

( log )

12x

x x

(c) ( log )

( log )

12

+ x

x x (d)

− +( log )

log

12

x

x x

24. ( )

( )

x e

xdx

x+

+∫3

4 2 is equal to

(a) e

xC

x

( )++

4 (b)

ex

Cx

( )++

4 2

(c) e

xC

x

( )++

3 (d)

14 2( )x

C+

+

25. 12 2 2 2

0

2

a x b xdx

sin cos

/

+∫π

is equal to

(a) πa

b4 (b)

πba4

(c) π2ab

(d) πa

b2

26. Let f : R → R be de�ned by f(x) = x4, then(a) f is one-one but not onto (b) f is neither one-one nor onto(c) f is one-one and onto (d) f may be one-one and onto

27. �e point on the curve y2 = x where the tangent

makes an angle π4

with X-axis is

(a) (1, 1) (b) 14 2

, 1

(c) 12 4

, 1

(d) (4, 2)

28. �e total number of terms in the expansion of (x + a)47 – (x – a)47 a�er simpli�cation is

(a) 24 (b) 96 (c) 47 (d) 48

| JUNE ‘17 65

29. �e function f(x) = x2 + 2x – 5 is strictly increasing in the interval(a) [–1, ) (b) (– , –1)(c) (– , –1] (d) (–1, )

30. �e degree of the di�erential equation

12 2

2

2+

=dy

dxd ydx

is

(a) 1 (b) 4 (c) 2 (d) 3

31. Binary operation * on R – {–1} de�ned by

a b ab* =

+ 1 is

(a) * is associative and commutative (b) * is neither associative nor commutative(c) * is commutative but not associative (d) * is associative but not commutative

32. �e plane 2x – 3y + 6z – 11 = 0 makes an angle sin–1 (a) with X-axis. �e value of a is equal to

(a) 3

2 (b) 2

7 (c) 2

3 (d)

37

33. If coe�cient of variation is 60 and standard deviation is 24, then arithmetic mean is

(a) 207

(b) 720

(c) 140

(d) 40

34. �e contrapositive statement of the statement “If x is prime number, then x is odd” is(a) If x is not a prime number, then x is odd. (b) If x is not a prime number, then x is not odd.(c) If x is a prime number, then x is not odd. (d) If x is not odd, then x is not a prime number.

35. �e probability distribution of X is

X 0 1 2 3P(X) 0.3 k 2k 2k

�e value of k is(a) 0.7 (b) 0.3 (c) 1 (d) 0.14

36. x x dx2 2 5+ +∫ is equal to

(a) ( )x x x+ + +1 2 52

− + + + + +2 1 2 52log x x x C

(b) 12

1 2 52( )x x x+ + +

+ + + + + +2 1 2 52log x x x C

(c) ( )x x x+ + +1 2 52

+ + + + + +2 1 2 52log x x x C

(d) ( )x x x+ + +1 2 52

+ + + + + +1

21 2 52log x x x C

37. If nC12 = nC8 then n is equal to(a) 12 (b) 26 (c) 6 (d) 20

38. If yf x g x h x

l m na b c

dydx

=( ) ( ) ( )

, then is equal to

(a)

f x g x h xl m na b c

′ ′ ′( ) ( ) ( )

(b) l m n

f x g x h xa b c′ ′ ′( ) ( ) ( )

(c) f x l ag x m bh x n c

′′′

( )( )( )

(d) l m na b c

f x g x h x′ ′ ′( ) ( ) ( )

39. If tan tan− −+ =1 1 45

x y π , then cot–1x + cot–1y is equal to

(a) π5

(b) 35π

(c) 25π (d)

40. �e range of the function f x x( ) = −9 2 is(a) [0, 3] (b) (0, 3] (c) (0, 3) (d) [0, 3)

41. Two events A and B will be independent if (a) P(A ∩ B ) = (1 – P(A)) (1 – P(B))(b) A and B are mutually exclusive (c) P(A) + P(B) = 1 (d) P(A) = P(B)

42. �e eccentricity of the ellipse x y2 2

36 161+ = is

(a) 2 56

(b) 2 134

(c) 2 54

(d) 2 136

43. If a b& are unit vectors, then angle between a band for 3 a b− to be unit vector is(a) 45º (b) 30º (c) 90° (d) 60°

| JUNE ‘1766

44. If � � � � � � � �a i j k b i j k= + + = + +2 2 3λ and are orthogonal, then value of is

(a) 32 (b) 1 (c) − 5

2 (d) 0

45. �e value of cos245° – sin215° is

(a) 3 12 2

+ (b) 34

(c) 3 12 2

− (d) 3

246. �e range of sec–1x is

(a) [0, ] (b) [ , ]02

π π− { }(c)

π π2 2

, (d) −

π π2 2

,

47. If a b c, , are unit vectors such that a b c+ + = 0 , then the value of a b b c c a⋅ + ⋅ + ⋅ is equal to

(a) − 32

(b) 3 (c) 32

(d) 1

48. dxe xsin

/

/

+−∫ 12

2

π

π is equal to

(a) 1 (b) 0 (c) π2

(d) − π2

49. �e rate of change of volume of a sphere with respect to its surface area when the radius is 4 cm is(a) 2 cm3/cm2 (b) 4 cm3/cm2

(c) 8 cm3/cm2 (d) 6 cm3/cm2

50. tan

cot tan

/ 7

7 70

2 xx x

dx+∫

π is equal to

(a) π4

(b) π2

(c) π6

(d) π3

51. If |x –2| ≤ 1, then(a) x ∈ (1, 3) (b) x ∈ (–1, 3)(c) x ∈ [1, 3] (d) x ∈ [–1, 3)

52. [ ].

.x dx

0 2

3 5

∫ is equal to

(a) 3.5 (b) 4.5 (c) 3 (d) 453. �e area of triangle with vertices (K, 0), (4, 0), (0, 2)

is 4 square units, then value of K is(a) 8 (b) 0 or –8 (c) 0 (d) 0 or 8

54. If f x Kx xx

( ) ifif

= ≤>

2 23 2

is continuous at x = 2, then

the value of K is

(a) 43

(b) 34

(c) 3 (d) 4

55. If Ax x

x x=

− −

− −

11 1

1 1π

ππ

ππ

sin ( ) tan

sin cot ( )

Bx x

x x=

− −

− −

cos ( ) tan

sin tan ( )

1 1

1 1

ππ

ππ

then A – B is equal to

(a) 12

I (b) I (c) O (d) 2I

56. If f(x) = 8x3, g(x) = x1/3, then fog(x) is (a) 83x (b) 8x3

(c) 8x (d) (8x)1/3

57. Let ∆ ∆= =

Ax x

By y

Cz z

A B Cx y zzy zx xy

2

2

21

1

1

1

and then

(a) 1 = – (b) 1 = (c) 1 = 2 (d) 1 ≠

58. If sin , tan , thenx tt

y tt

dydx

=+

=−

21

212 2 is equal

to(a) 1 (b) –1 (c) 2 (d) 0

59. Re�ection of the point (a, , ) in XY plane is (a) (0, 0, ) (b) (a, , – )(c) (–a, – , ) (d) (a, , 0)

60. Area of the region bounded by the curve y = cos x, x = 0 and x = is(a) 2 sq. units (b) 3 sq. units(c) 4 sq. units (d) 1 sq. units

ANSWER KEYMPP-2 CLASS XI1. (c) 2. (a) 3. (d) 4. (a) 5. (b)

6. (c) 7. (b) 8. (a,c) 9 . (b,c) 10. (d)

11. (b,c) 12. (a,c,d) 13. (a) 14. (a) 15. (b)

16. (d) 17. (4) 18. (5) 19. (1) 20. (1)

| JUNE ‘17 67

SOLUTIONS

1. (c) : Given, point is A(–2, 4, –5),

Line (l) is x y z+ = − = + =33

45

86

λ (say)

Co-ordinates of B are (3 – 3, 5 + 4, 6 – 8)\ Direction ratios of AB are (3 – 1, 5 , 6 – 3)

Now, (3 – 1)(3) + (5 )(5) + (6 – 3)(6) = 0

⇒ λ = 310

∴ = − + −AB i j k� ��� � � �1

1032

65

∴ = = + +d AB| |� ��� 1

10094

3625

= + + = =1 225 144

100370100

3710

2. (d) : We have |KA| = Kn|A|, Here n = 3\ |KA| = K3|A|

3. (c) : Equation of required line is y – 2 = – 13

(x – 1)⇒ x + 3y – 7 = 0

4. (a) : Given, dydx

y+ = 1

�is is a linear di�erential equation.

∴ = =∫I.F. e edx x1

\ Solution is given by, ye e dx e Cx x x= ⋅ = +∫ 1 1⇒ ex(y – 1) = C1 ⇒ x + log|y – 1| = log C1 ⇒ –x – log|y – 1| = – logC1

⇒−

= +log 11y

x C [where – logC1 = C]

or log 11 −

= +y

x C

5. (d) : We have, f(x) = x2 in [2,4]\ According to mean value theorem,

We have, ′ = −−

f C f b f ab a

( ) ( ) ( )

[where a = 2 and b = 4]

∴ = −−

= − =2 4 24 2

4 22

122

2 2C f f( ) ( ) ( ) ( )

⇒ 2C = 6 ⇒ C = 3

6. (b) : We have, lim coscosθ

θθ→

−−0

1 41 6

=

→lim sin

sinθ

θθ0

2

223

= =( )( )23

49

2

2

7. (d) : We have, y x xx x

= +−

−tan sin coscos sin

1

= +

= +

= +− −tan tan

tantan tan1 11

1 4 4xx

x xπ π

∴ =dydx

1

8. (a) : Given, 11

1+−

=ii

m ⇒ im = i4 ⇒ m = 4

9. (b) : Let I x dx= +−∫ | |25

5

= − + + +

−−

∫∫ ( ) ( )x dx x dx2 22

5

5

2

= − +

+ +

x x x x2

5

2 2

2

5

22

22

= + = =9

2492

582

29

10. (a) : Let I xx

dx= −−∫

cos coscos cos

2 2θθ

= − − −−∫

( cos ) ( cos )cos cos

2 1 2 12 2xx

dxθθ

= −−

= +∫ ∫2 2

2 2(cos cos )cos cos

(cos cos )xx

dx x dxθθ

θ

= 2(sinx + x cos ) + C11. (a) : Required area = Area of shaded portion

| JUNE ‘1768

= ∫20

16ydy = ⋅ = =2 2

343

4 2563

3 2016 3y / [ ] sq. units

12. (a) : Given, A B ⇒ A ∩ B = A \ n(A ∪ B) = n(A) + n(B) – n(A ∩ B) = n(A) + n(B) –n(A) = n(B)

13. (b) : Given, A is symmetric ⇒ aij = aji ...(i) i ≠ jA is skew symmetric ⇒ aij = –aji ...(ii) and aii = 0Adding (i) and (ii) we get 2aij = 0 ⇒ aij = 0\ A is a zero matrix.

14. (c) : We have, 3

13 24 1

xx

=

⇒ 3 – x2 = 3 – 8 ⇒ = ⇒ = ±x x2 8 2 2

15. (d) : Given, x dydx

y x dydx

yx

x⋅ + = ⇒ + =2 22

∴ = = =∫

I.F. e e xxdx x

222log

16. (c) : Perpendicular distance of the point P(6,7,8) from XY-plane is 8.

17. (c) : Clearly, shaded region represents 5x + 4y > 20, x < 6, y < 3, x > 0, y > 0

18. (b) : �e optimal solution occurs at every point on the line joining these two points.

19. (c) : Given series is in A.P. with �rst term (a) = 3, common di�erence (d) = 2

∴ = × + − = +S n n n nn 22 3 1 2 2[ ( ) ] ( )

20. (a) : Given 2 6

1 2 25 61 8

++

=

yx

⇒ 2 + y = 5 and 2x + 2 = 8 ⇒ y = 3, x = 3

21. (a) : Let u = cos–1(2x2 – 1) and v = cos–1 x ⇒ cos v = x\ u = cos–1(2cos2v – 1) = cos–1(cos2v) = 2v

∴ =dudv

2

22. (c) : p q p n= = = − = =10100

110

1 910

5, ,

P X x Cx

x x( )= =

−5

5110

910

\ P(X ≤ 1) = P(X = 0) + P(X = 1)

=

+

50

0 55

1

1 4110

910

110

910

C C

23. (b) : We have, y = log (log x) ⇒ dydx x x

= 1log

⇒ = − ⋅ + ⋅

=− +d y

dx x xx

xx

x

x x

2

2 2 21 1 1

1

( log )log

( log )

( log )

24. (a) : Let I = ( )( )x ex

dxx+

+∫34 2 = + −

+∫( )

( )x e

xdx

x4 14 2

=+

−+

=

++∫

14

14 42x x

e dx ex

Cxx

( )

e f x f x dx e f x Cx x( ( ) ( )) ( )+ ′ = + ∫

25. (c) : Let Ia x b x

dx=+∫1

2 2 2 20

2

sin cos

=

+∫sectan

/ 2

2 2 20

2 xdxa x b

π

Put tan x = t ⇒ sec2 x dx = dt

∴ =+

=

+

∞ ∞

∫ ∫I dta t b

dt

a t ba

2 2 20 2 2

2

20

=

=−∞1

221

0aab

atb ab

tan π

26. (b) : Given f(x) = x4

Now, f(1) = f(–1) but 1 ≠ –1\ f is not one-one Also, co-domain of f is R and range of f is [0, )\ f is not onto

27. (b) : We have, y2 = x ⇒ 2yy = 1

⇒ ′ = = ⇒ =yy

y12 4

12

tan π

When y x y= = =

=12

12

14

22

,

28. (a) : Number of terms in (x + a)47 – (x – a)47

= + =47 12

24

[Number of terms in (a + b)n – (a – b)n is n + 1

2,

when n is odd number]

| JUNE ‘17 69

29. (d) : f is strictly increasing ⇒ f (x) > 0⇒ 2x + 2 > 0 ⇒ x > –1

30. (a) : Highest order derivative is d ydx

2

2 and its power is 1.

Hence, degree of di�erential equation is 1.

31. (b) : We have, a * b = a

b + 1

1 2 13

2 1 22

1* *= = =but

�us 1 * 2 ≠ 2 * 1 \ * is not commutative

Now, (1 * 2) * 3 = 13 * 3 = 1 3

41

12/ =

and ( )1 2 3 1 12

112

1

23* * *= =

+=

\ * is not associative [Infact, * is not a binary operation !!!]

32. (b) : Let f be the angle made by plane 2x – 3y + 6z – 11 = 0 and X-axis i.e., (1,0,0)

∴ = × − × + ×+ + + +

sin | |φ 2 1 3 0 6 04 9 36 1 0 0

⇒ =

=− −φ αsin sin ( )1 127

33. (d) : We have C.V. = σx

× 100

⇒ = × ⇒ =60 24 100 40x

x

34. (d) : �e contrapositive statement of the statement “If x is prime number, then x is odd” is “If x is not odd, then x is not a prime number.”

35. (d) : We know, P X( ) =∑ 1⇒ 0.3 + k + 2k + 2k = 1 ⇒ 5k = 0.7 ⇒ 0.14

36. (b) : Let I x x dx x dx= + + = + +∫ ∫2 2 22 5 1 2( )

= + + + + + + + + +( ) logx x x x x x C12

2 5 2 1 2 52 2

37. (d) : Given, nC12 = nC8 ⇒ 8 + 12 = n ⇒ n = 20 [ nCx = nCy ⇒ x = y or x + y = n]

38. (a, c, d)

39. (a) : Given, tan–1 x + tan–1 y = 45π

⇒ + = − =− −cot cot1 1 45 5

x y π π π

40. (a) : Let y = 9 2− x ⇒ y2 = 9 – x2

⇒ x2 = 9 – y2 ⇒ x = 9 2− y

Clearly, 9 – y2 > 0 ⇒ y2 < 9⇒ –3 < y < 3But y > 0. Hence 0 < y < 3.

41. (a) : If A and B are independent�en A and B are also independent⇒ P(A ∩ B ) = P(A )P(B ) = (1 – P(A))(1–P(B))

42. (a) : e a ba

= − = − =2 2 36 16

62 5

643. (b) : Given, | | | |a b= = 1

Now, | | | | | | ( )3 3 2 32 2 2a b a b a b− = + − ⋅

⇒ 1 = 3(1) + 1 − 2 3 | || | cosa b θ

⇒ 1 = 4 −2 3 1 1( )( )cosθ

⇒ = ⇒ = °cosθ θ32

30

44. (c) : For, a band to be orthogonal a b⋅ = 0⇒ (2)(1) + ( )(2) + (1)(3) = 0

⇒ 5 + 2 = 0 ⇒ = − 52

45. (b) : cos245° – sin215° = cos(45°+15°)cos(45°–15°)

= cos60° cos30° = ⋅ =12

32

34

46. (b) : Range of sec–1 x is [ , ]02

π π−

47. (a) : Here, a b c, , are unit vectors and a b c+ + = 0 ∴ + + =( )a b c 2 0

⇒ + + + ⋅ + ⋅ + ⋅ =| | | | | | ( )a b c a b b c c a2 2 2 2 0

⇒ + + + ⋅ + ⋅ + ⋅ =1 1 1 2 0( )a b b c c a

⇒ ⋅ + ⋅ + ⋅ = −a b b c c a 32

48. (c) : Let I dxe x=

+−∫ sin/

/

12

2

π

π ...(i)

⇒ =+

=+−

− −∫ ∫I dx

ee dx

ex

x

xsin/

/ sin

sin/

/

1 12

2

2

2

π

π

π

π ...(ii)

| JUNE ‘1770

Adding (i) and (ii), we get

2 112

2I e

edx

x

x= ++−

∫sin

sin/

/

π

π

⇒ = = = ⇒ =−−∫2 1

222

2

2I dx x I[ ] /

/

/

/

ππ

π

ππ π

49. (a) : dVdS

dV dtdS dt

r drdt

r drdt

r= = = = =//

/cm4

8 242

22

3 2π

πcm

50 . (a) : tan

cot tan

/ 7

7 70

2

4x

x xdx

+=∫

ππ

f x

f x f xdx( )

( )

/

+ −

=

∫ πππ

240

2

51. (c) : Here, |x –2| ≤ 1 ⇒ –1 ≤ x – 2 ≤ 1 ⇒ 1 ≤ x ≤ 3⇒ x ∈[1,3]

52. (b) : Here, [ ]..

. .x dx dx dx dx dx= + + +∫∫ ∫ ∫ ∫0 1 2 3

0 2

1

0 2

3 5

1

2

2

3

3

3 5

= 0 + [x]12 + [2x]2

3 + [3x]33.5 = 1 + 2(1) + 3(0.5) = 4.5

53. (d) : Given, area of triangle with vertices (K, 0),(4, 0), (0, 2) is 4 sq. units i.e.,

± = ⇒ − + = ±4 12

0 14 0 10 2 1

2 82

4K

K

⇒ K = 0 or 854. (b) : Given, f(x) is continuous

⇒ = =→ →− +lim ( ) lim ( ) ( )

x xf x f x f

2 22

⇒ = ⇒ =→

lim ( )x

Kx K2

2 3 4 3 ⇒ =K 34

55 . (None of the options is correct) :

If matrix B will be, Bx x

x x=

− −

− −

11 1

1 1π

ππ

ππ

cos tan

sin tan

�en, A B I− = 12

56. (c) : fog(x) = f(g(x)) = = =f x x x( ) ( )/ /1 3 1 3 38 8

57. (b) : We have, ∆ = =

Ax x

By y

Cz z

xyz

A xx

B yy

C zz

2

2

2

1

1

1

1

1

1

= = =A x yzB y xzC z xy

A B Cx y zzy zx xy

∆1

58. (a) : We have, x tt

t=+

=− −sin tan12

121

2

and tan tany tt

t=−

=− −12

121

2

∴ = ⇒ =y x dydx

1.

59. (b) : Re�ection of the point (a, , ) in XY plane is(a, ,– ) .

60. (a) : Given, curves are y = cos x ; x = 0 and x =

\ Required area = 20

2cos

/xdx

π∫

= [ ]2 02sin /x π = 2 sq. units

| JUNE ‘1772

JEE MAIN

1. If a1 = 11, a2 = 75, a3 = 20, a4 = 23 and an = an – 1 – an – 2+ an – 3 – an – 4, n 5, then a31 – a53 + a75 =(a) 53 (b) 58(c) 65 (d) 75

2. If xyz = 1, x + 1z

= 5, y + 1x

= 29, then z + 1y

=

(a) 13

(b) 14

(c) 2 (d) 34

3. In a triangle ABC, D 1 1 1 12

12

22

32r r r r

+ + +

=

(a) 2 tanA (b) 2 cotA(c) 4 tanA (d) 4 cotA

4. A straight line through the point (a, b) meets x-axisat A and y-axis at B. O is the origin. If (a, b) = (4, 1),then the minimum value of OA + OB is(a) 7 (b) 8 (c) 9 (d) 10

5. If x ≠ a, y ≠ b, z ≠ c anda y zx b zx y c

= 0, then

x ax a

y by b

z cz c

+−

+ +−

+ +−

=

(a) –1 (b) 0(c) 1 (d) 2

JEE ADVANCED

6. A bag contains 30 tokens numbered serially from0 to 30. �e number of ways of selecting 3 tokensfrom the bag, so that the sum of the numbers onthem is 30, is divisible by(a) 2 (b) 3 (c) 5 (d) 7

COMPREHENSION

Let f x x x x( ) [sin ] [cos ], [ , ]= + ∈− −1 1 0 1

7. �e number of points at which f(x) is notdi�erentiable is(a) 0 (b) 1 (c) 2 (d) 3

8. �e solution set of the equation f(x) = 0 is an interval of length

(a) 2 14

sin −

p (b) 2 14

sin +

p

(c) 2 14

cos −

p (d) 2 14

cos +

p

INTEGER TYPE

9. Ten boys and two girls are to be seated in a rowsuch that there are atleast 3 boys between thegirls. �e number of ways this can be done isλ · 12!, where λ =

MATRIX MATCH

10. �e curve f (x, y) = 0 lies in the �rst quadrant.�e tangent at a point on it meets the positivex and y axes at A and B and O is the origin

List-I List-IIP. f(x, y) = 4xy – 1 1. AB = 1Q. f(x, y) = x2 + y2 – 1 2. OA + OB = 1

R. f x y x y( , ) = + − 1 3. OA · OB = 1

S. f(x, y) = x2/3 + y2/3 – 1 4.1 1 12 2OA OB

+ =

P Q R S (a) 2 1 3 4(b) 1 2 4 3(c) 4 3 2 1 (d) 3 4 2 1

Maths Musing was started in January 2003 issue of The aim of Maths Musing is to augment the chances of bright students seeking admission into IITs with additional study material.

During the last 10 years there have been several changes in JEE pattern. To suit these changes Maths Musing also adopted the new pattern by changing the style of problems. Some of the Maths Musing problems have been adapted in JEE benefitting thousand of our readers. It is heartening that we receive solutions of Maths Musing problems from all over India.Maths Musing has been receiving tremendous response from candidates preparing for JEE and teachers coaching them. We do hope that students will continue to use Maths Musing to boost up their ranks in JEE Main and Advanced.

See Solution Set of Maths Musing 173 on page no. 85

| JUNE ‘17 73

1. Let x, y, z be real numbers such thatcos x + cos y + cos z = 0 andcos 3x + cos 3y + cos 3z = 0, thencos 2x cos 2y cos 2z(a) ≤ 0(b) ≥ 0(c) depends on x, y, z values(d) data insu�cient

2. Eliminate from the system : cos 2 = cos ( + a)and sin 2 = 2 sin ( + a)(a) (cos a)2/3 – (sin a)2/3 = 2/3

(b) (cos a)2/3 + (sin a)2/3 = 2/3

(c) (cos a)1/3 – (sin a)1/3 = 1/3

(d) (cos a)1/3 – (sin a)1/3 = 1/3

3. �e bisector of BAC intersects the circumcircleof ABC at D. If AB2 + AC2 = 2AD2 then angle ofintersection of AD and BC is(a) 30° (b) 45°(c) 60° (d) 90°

4. �e laws of points (x, y) satisfying the equationsx2 + y cos2 a = x sin a cos a andx cos 2a + y sin 2a = 0 lies on (a is a parameter)(a) circle (b) parabola(c) ellipse (d) hyperbola

5. In ABC, ABC = ACB = 40°. If P is a point inthe interior of the triangle such that PBC = 20°and PCB = 30° then(a) BP = BA (b) BP = 2BA

(c) BP = 12 BA (d) BP = 3BA

6. Let ABC be a triangle of area 12

, then minimum value of a2 + cosec A is(a) 2 (b) 3 (c) 4 (d) 5

7. Let a, , , be positive numbers such that for all x,sin ax + sin x = sin x + sin x, then + = (a) a (b) 2a (c) 3a (d) 4a

8. Let S be the set of all triangles ABC for which

5 1 1 1 3 6AP BQ CR AP BQ CR r

+ +

− =min. { , , }

,

where r is inradius and P, Q, R are points of tangency of incircle with sides AB, BC, CA respectively then all the triangles in the set S are(a) scalene (b) isosceles(c) equilateral (d) right angled

9. Let ABC be a triangle such that max. {A, B} = C + 30°

andRr

= +3 1 , R is circumradius, r is inradius

then ABC is(a) scalene (b) isosceles(c) equilateral (d) right angled

10. Let ABCDEFGHIJKL be a regular do-decagon thenABAF

AFAB

+ =

(a) 1 (b) 2 (c) 3 (d) 4

11. Let a, b, c, d ∈ [0, ] such that 2 cos a + 6 cos b + 7 cos c + 9 cos d = 0 and 2 sin a – 6 sin b + 7 sin c – 9 sin d = 0 then

cos ( )cos ( )

a db c

++

=

(a) 73

(b) 37

(c) 35

(d) 53

12. If sin x cos y + sin y cos z + sin z cos x = 32

, then

(a) sin x = cos 2y (b) sin x = sin y(c) sin x = cos y (d) sin x = cos z

20 CHALLENGINGPROBLEMS

For Entrance Exams

| JUNE ‘1774

13. In ABC, = sin A sin B + sin B sin C + sin C sin Aand (1 + sin A)(1 + sin B) (1 + sin C) = 2( + 1), then

ABC is(a) scalene (b) isosceles(c) equilateral (d) right angled

14. Let ABC be a triangle such thatsin2B + sin2C = 1 + 2 sinB sinC cosA, then ABC is(a) scalene (b) isosceles(c) equilateral (d) right angled

15. In ABC, cot cot cotA B C sr2

42

92

67

22 2

2

+ + =

where s → semiperimeter and r → inradius, thenthe ratio of the sides of the triangle is(a) 45 : 40 : 13 (b) 50 : 55 : 45(c) 55 : 65 : 70 (d) 60 : 65 : 80

16. Let n be a positive integer and for real numbers and akl . (k, l = 1, 2, 3, ... n) (k > l), we have

sinsin

cos ( )2

21

2 2nxx

a k l xkll k n

= + −≤ < ≤∑λ

(x ≠ m , m ∈I) then = (a) l (b) k (c) n (d) k – l

17. In ABC, A = 30° and an inscribed circle of �xedradius r is drawn. If ABC has least perimeter then

B =(a) 30° (b) 45° (c) 60° (d) 75°

18. If A1, A2, A3, A4 are the angles of a convex quad.then max. value of

sin sin sin sinA A A A1 2 3 4

2 2 2 2

+

+

+

is

(a) 2 (b) 2 2 (c) 3 2 (d) 4 2

19. If a, b, c, k are constants and a are variablessubject to the relation a tan a + b tan + c tan = kthen minimum value of tan2 a + tan2 + tan2 is

(a) k

a b c

2

2( )+ +(b)

ka b c

2

2 2 2+ +

(c) ( )a b c

k+ + 2

2(d)

a b ck

2 2 2

2

+ +

20. For x ∈ R, the minimum value of|sin x + cos x + tan x + sec x + cosec x + cot x| is(a) 2 2 (b) 2 2 – 1

(c) 2 2 + 1 (d) 2 1− BP

SOLUTIONS

1. (a) : Using cos 3x = 4 cos3x – 3 cos x.We have, cos3 0x =∑using identity a abc a a ab3 23− = −∑ ∑ ∑∑ ( )( )We have, abc = cos x cos y cos z = 0Let cos z = 0. So, cos x = – cos y andcos 2x = cos 2y and cos 2z = –1So, cos 2x cos 2y cos 2z = – cos2 2x ≤ 0.

2. (b) : Expanding the given equations, we have(cos cos a – sin sin a) = (cos2 – sin2 )and sin cos a + cos sin a = sin cos ⇒ cos a = cos3 and sin a = sin3 Hence, (cos a)2/3 + (sin a)2/3 = 2/3.

3. (b) : Let CAB = a, ABC = , BCA = thenusing sine rule, we have

ABsin γ

= AC ADsin sin ( / )β α β

=+2

So, AB2 + AC2 = 2AD2 becomes sin2 + sin2 = 2 sin2 (a/2 + )

Simplifying, cos ( – )[1 + cos ( + )] = 0i.e., cos ( – ) = 0. So, a + 2 = /2

So, AEC = α β π2 4

45+ = = °

4. (b) : Simplifying the �rst equation, we havex sin 2a – y cos 2a = 2x2 + yand the other given equation isx cos 2a + y sin 2a = 0Solving, we have

sin 2a = x x y

x yy x yx y

( )cos

( )22

22

2 2

2

2 2

++

=− +

+and α

Hence, sin2 2a + cos2 2a = 1 gives

( )2 2

2 2

x yx y

++

2

= 1 i.e., 4x2 + 4y – 1 = 0, Parabola.

5. (a) : Let us assume that AB = AC = 1 unit thenBC = 2 cos 40° and BPC = 130°Applying sine rule in BPC, we have

BPsin ( )30°

= BCsin ( )130°

⇒ BP =BC ⋅ °

°=

° ⋅ °°

=sin

coscos sin

cos30

402 40 30

401

i.e., BP = AB.

| JUNE ‘17 75

6. (d) : Given, area = 12

⇒ 12

bc sin A = 12

⇒ cosec A = bc and [bc ≥ 1]Now, a2 + cosec A = a2 + bc = b2 + c2 – 2bc cos A + bc

[cosine rule]

= b c bc A bc2 2 22 1+ − − +sin

= b c b c bc A bc2 2 2 2 22+ − − +( sin )

= b bc c b c2 2 2 22 1+ + − −

≥ 3 2 12 2bc b c− −

Let x = bc (≥ 1) then y = 3 2 12x x− − gives5x2 – 6xy + y2 + 4 = 0. As x ∈ R, D ≥ 0 gives(–6y)2 – 20(y2 + 4) ≥ 0 i.e., y ≥ 5 .

7. (b) : Di�erentiating the given identity three times,we have, a3 cos ax + 3 cos x = 3 cos x + 3 cos xAlso, acos ax + cos x = cos x + cos xIn particular for x = 0, we have

a + = + and a3 + 3 = 3 + 3

i.e., (a + )3 = ( + )3

⇒ a = on simpli�cation.So, (a – )(a – ) = a2 – a( + ) +

= a2 – a(a + ) + a = 0⇒ = a and = a i.e., + = 2a

8. (b) : Let us assume that min. {AP, BQ, CR} = APand let tan (A/2) = x, tan (B/2) = y, tan (C/2) = z.

So, AP = rx

, BQ = ry

and CR = rz

Now, the relation in question becomes 2x + 5y + 5z = 6and in any , we know that xy + yz + zx = 1.Now, eliminating (x) from these two equations, we have 5y2 + 5z2 + 8yz – 6y – 6z + 2 = 0i.e., (3y – 1)2 + (3z – 1)2 = 4(y – z)2

or, 5a2 + 5 2 + 8a = 0 [where 3y – 1 = a,3z – 1 = ]

⇒ a = 0 = for real solutions.

i.e., y = z = 13

and so, x = 43

i.e., isosceles triangle.9. (d) : Let max. {A, B} = A then A.T.Q, A – C = 30°

and using the identity

r = 42 2 2

R A B Csin . sin . sin

We have, r r A B C= +

4 3 1

2 2 2sin sin sin

i.e., 3 14− = sin cos cosB A C A C

2 2 2−

−+

i.e.,3 14−

= sin cos cosB B2

302

1802

°

−° −

i.e., sin sin .2

2 26 2

43 14

0B B−

+

+

=

Solving, sin (B/2) = 6 24

22

−or

i.e., B2

= 15° or 45°.

But B = 90° is not possible. Hence, B = 30° and A = 90°, C = 60°, i.e., [Right angled ]

10. (d) : Let R be the circumradius then

AB = 212

R sin π

and AF = 2 512

R sin π

So, the required quantity is 22 5

2 52

RR

RR

sinsin

sinsin

θθ

θθ

+

where θ π=

12

= sin sin

sin sin

2 2 55

θ θθ θ+

= 1 2 1 10

4 6− + −

−cos coscos cos

θ θθ θ

= 4 (on simpli�cation)11. (a) : Rearranging the two equations, we have

2sin a – 9 sin d = 6 sin b – 7 sin cand 2 cos a + 9 cos d = – 6 cos b – 7 cos cSquaring and adding both the equations, we have85 + 36 cos (a + d) = 85 + 84 cos (b + c)

i.e.,cos ( )cos ( )

a db c

++

= =8436

73

12. (c) : �e given equation can be rewritten as,(sin x – cos y)2 + (sin y – cos z)2 + (sin z – cos x)2 = 0i.e., sin x = cos y

13. (d) : Equating the two values and simplifying,we have(1 – sin A)(1 – sin B) (1 – sin C) = 0i.e., sin A = 1 or sin B = 1 or sin C = 1i.e., ABC is right angled triangle.

| JUNE ‘1776

14. (d) : Using cosine rule, we have a2 + 2bc cos A = b2 + c2

And using sine rule, here, we have sin2 B + sin2 C = sin2 A + 2 sin B sin C cos ANow comparing this with equation given in question, we have, sin2 A = 1 i.e., A = 90°

15. (a) : In any triangle, we have

cot cot cot cot .cot .cotA B C A B C sr2 2 2 2 2 2

+ + = =

So, the given relation in question becomes,

( ) cot cot cot6 3 22

22

32

2 22 2

+ +

+

+

22A B C

= 62

62

62

2

cot cot cotA B C+ +

i.e., Equality in Cauchy-Schwarz inequality.

So, cot ( / ) cot ( / ) cot ( / )A B C2

62 2

33 2

2= =

i.e., cot ( / ) , cot ( / ) ( / )A B C2 7 2 74

2 79

= = =and cot

⇒ sin , sin , sinA B C= = =725

5665

126130

i.e., Sides of the triangle are 26, 80 and 90.

16. (c) : Using the two identities, we have

S1 = cos ( )sin cos ( )

sin2

1

1mx

nx n xxm

n=

⋅ +

=∑

and S2 = sin ( )sin sin ( )

sin2

1

1mx

nx n xxm

n=

⋅ +

=∑

S S12

22+ =

sinsin

nxx

2

But S S12

22+ = (cos 2x + cos 4x + ... + cos nx)2

+ (sin 2x + sin 4x + ... + sin nx)2

= n k x l x kx lx+ +∑2 2 2 2 2(cos cos sin sin )1 ≤ l < k ≤ n

i.e., S S12

22+ = n k l x

l k n+ −

≤ < ≤∑ cos ( )2

1

i.e., = n

17. (d) : In any triangle, we have cot ( / )A sr

2 =∑

So, perimeter 2s = 2 2r A. cot ( / )∑

Since A, r are �xed, s is min. when cot ( / )A 2∑ is min.i.e., cot (B/2) + cot (C/2) is min.

i.e.,cos ( / )

sin ( / ) sin ( / )A

B C2

2 2 is min.

or, sin (B/2) . sin (C/2) is max.

i.e.,12 2 2

cos sinB C A−

is max.

i.e., cosB C−

2

= 1 for max. i.e., B = C.

\ ABC is an isosceles with B = C = 75°.18. (b) : A1 + A2 + A3 + A4 = 2 ,

So, A A A A1 2 3 4

2 2 2 2+ + + =

Now, f (x) = sin x is a concave function.

So, sin sin sin sinA A A A1 2 3 4

2 2 2 2

+

+

+

is max.

When A A A A1 2 3 4

2 2 2 2= = = = each = π

4[Result of Jensen's inequality]

So, required max. value is 4 ⋅ sin ( /4) = 2 2 .19. (b) : Use the identity

( tan tan )b cγ β−∑ 2

= (a2 + b2 + c2)(tan2 a + tan2 + tan2 )– (a tan a + b tan + c tan )2

We have, RHS ≥ 0⇒ ( ) ( tan )a k2 2 2 0∑ ∑⋅ − ≥α

i.e., tanmin

22

2α∑( ) = kaΣ

20. (b) : Let E = |sin x + cos x + tan x + sec x + cosec x+ cot x|

Putting a = sin x, b = cos x, c = a + bWe have on simpli�cation, the given expression

E = cc

cc

+−

= − +−

+21

1 21

1

where c ∈ −[ , ]2 2

Using AM ≥ GM on (c – 1) and 21c −

, we have

Emin = | |− +2 2 1 i.e., 2 2 1−

| JUNE ‘17 77

1. �e sum of all the non-real roots of(x2 + x – 2) (x2 + x – 3) = 12 is(a) 1 (b) –1(c) 6 (d) –2

2. Statement-1 : �e equation sinx + xcosx = 0 has atleast one root in the interval (0, ).Statement-2 : Between any two roots of f(x) = 0, there exists atleast one root of f (x) = 0.(a) Statement-1 is true, Statement-2 is true; Statement-2

is a correct explanation for Statement-1.(b) Statement-1 is true, Statement-2 is true; Statement-2

is not correct explanation for Statement-1.(c) Statement-1 is true, Statement-2 is false.(d) Statement-1 is false, Statement-2 is true.

3. If LCM of p, q is r2t4s2, where r, s, t are primenumbers and p, q are positive integers . �en the number of ordered pairs (p, q) is(a) 252 (b) 254(c) 225 (d) 224

4. Let f(x) = max{x, x3} x ∈ R the set of points wheref(x) is not di�erentiable is(a) {–1, 1} (b) {–1, 0}(c) {0, 1} (d) {–1, 0, 1}

5. Statement-1: �e sum of the �rst 30 terms of thesequence 1, 2, 4, 7, 11, 16, 22, 29, 37, 46 ... is 4520.Statement-2: �e successive di�erences of the terms of the sequence form an A.P.

(a) Statement-1 is true, Statement-2 is true; Statement-2 is a correct explanation for Statement-1.

(b) Statement-1 is true, Statement-2 is true; Statement-2 is not correct explanation for Statement-1.

(c) Statement-1 is true, Statement-2 is false.(d) Statement-1 is false, Statement-2 is true.

6. tan−

=

=∑ 12

1

12rr

n

(a) tan–1n (b) tan−+

11

nn

(c) tan−+

12

nn

(d) tan− ++

1 12

nn

7. �e statement p → (q → p) is equivalent to(a) p → (p → q) (b) p → (p q)(c) p → (p q) (d) p → (p q)

8. �e remainder le� out when 82n – (62)2n + 1 isdivisible by 9 (where n ∈ N)(a) 0 (b) 2 (c) 7 (d) 8

9. Consider all functions that can be de�ned from the set A = {1, 2, 3} to the set B = {1, 2, 3, 4, 5}. A function f(x) is selected at random from these functions. �e probability that, selected function satis�es f(i) ≤ f(j) for i < j, is equal to

(a) 625

(b) 1225

(c) 25

(d) none of these

Math Archives, as the title itself suggests, is a collection of various challenging problems related to the topics of IIT-JEE Syllabus. This section is basically aimed at providing an extra insight and knowledge to the candidates preparing for IIT-JEE. In every issue of MT, challenging problems are offered with detailed solution. The readers’ comments and suggestions regarding the problems and solutions offered are always welcome.

BESTPROBLEMS10

B y : b . : 0 9 3 3 4 8 7 0 0 2 1

10. [cot ]x dx0

π

∫ =

(where [⋅] denotes the greatest integer function)

(a) π2

(b) 1 (c) –1 (d) − π2

SOLUTIONS

1. (b) : Put x2 + x = y then, we have y2 – 5y – 6 = 0⇒ (y – 6) (y + 1) = 0⇒ x2 + x – 6 = 0 or x2 + x + 1= 0⇒ x = –3, 2 or x = , 2

Sum of non-real roots = + 2 = –12. (c) : Take f(x) = xsinx, which is continuous in[0, ] and di�erentiable in (0, ). Also f(0) = f( ) = 0By Rolle’s theorem, there exists at least one root of f (x) = 0 ⇒ xcosx + sinx = 03. (c) : No. of ordered pairs

= (2(2) + 1)(2(4) + 1)(2(2) + 1)= 5 × 9 × 5 = 225

4. (d) :–1 0 1

5. (d) : a2 – a1 = 1, a3 – a2 = 2, a4 – a3 = 3, ...

an n n n

n = +−( ) = − +1

12

22

2

\ Sum = ⋅ ⋅ − ⋅ + ⋅

=12

30 31 616

30 312

2 30 4525

6. (b) : 12

24

2 1 2 1

1 4 12 2 2r rr r

r= =

+( ) − −( )+ −( )

Sum = (tan–1(2r + 1) – tan–1 (2r – 1))

= tan–1(2n + 1) – tan–1(1) =+

−tan 11

nn

7. (b) : p → (q → p) ~ p (q → p) ~ p (~ q p) p → (p q)

8. (b) : (1 + 63)n – (63 – 1)2n + 1

Remainder is 2.9. (d) : Total Function = 53

10. (d) : Let = ⇒ = −∫∫[cot ] [cot( )]x dx x dxπππ

00

∴ + = − + − =− ∉

∫� � ∵( ) [ ] [ ]1

10

0dx x x

x zx z

π ifif

∴ = − π2

| JUNE ‘1780

1. a1 , ... , ak, ak + 1, ..., an are positive numbers (k < n).Suppose that the values of ak + 1, ...., an are �xed. How should one choose the values of a1,...., an in

order to minimize aa

i

ji j i j, , ≠∑ ?

2. Let m be a positive integer. De�ne the sequencea0 , a1 , a2, .... by a0 = 0, a1 = m and an + 1 = m2an – an – 1 for n = 1, 2, 3, .... . Prove that an ordered pair (a, b) of non-negative integers, with a ≤ b, gives a solution to the equation

a bab

m2 2

21

++

=

if and only if (a, b) is of the form (an , an + 1) for some n ≥ 0.

3. In a ABC, C = 2 B. P is a point in theinterior of ABC satisfying that AP = AC andPB = PC. Show that AP trisects A.

4. Determine all the possible values of the sum of thedigits of the perfect squares.

5. ABCD is a convex quadrilateral and O is theintersection of its diagonals. Let L, M, N be themid-points of DB, BC, CA respectively. Supposethat AL, OM, DN are concurrent. Show thateither AD BC or [ABCD] = 2[OBC].

SOLUTIONS

1. To minimize the given rational function, choose

aa a

a a

iik n

k n

=+ +

+ +

= ⋅ =+

+

1

1

1 2

1 21 1

1 2...

...( ) , , ,

/

/A H ....,k

where A is the arithmetic mean and H is the harmonic mean of ak + 1 , ..., an. To prove this, we will be forgiven if we change notation : let xi = ai , i = 1, 2, ..., k and br = ak + r , r = 1, ..., m with k + m = n and denote the given rational function F(x1, ..., xk). �en we have F(x1, ..., xk) = X + Y + B, where

Xxx

xx

i

j

j

ii j k= +

≤ < ≤1,

Yxb

bxi k

i

r

r

ir m= ∑ +

≤ ≤ ≤ ≤1 1,

Bbb

bb

r

s

s

rr s m= +

≤ < ≤1.

Note that B is �xed and Y can be improved to

Yb

x bxrr m

i ir m ii k

= ∑

+ ∑

≤ ≤ ≤ ≤≤ ≤

1 11 11

= +

∑ m x m

xiii HA

where A is the arithmetic mean and H is the harmonic mean of the br .

Now we recall that the simple function α βxx

+(with a, , x all positive) assumes its minimum

when α βxx

= ; that is x = β α/ . Thus each

of the terms in Y (and so Y itself ) assumes its minimum when we choose, for i = 1, 2, ..., k,

x mmi = =A

HAH

( / ), as asserted.

But there is more. It is also known that each term in X, (and so X itself) assumes its minimum when xi = xj, with 1 ≤ i < j ≤ k. �us choosing all xi = AH minimizes both X and Y and, since B

| JUNE ‘17 81

is �xed, minimizes F(x1, ..., xk) as claimed.2. Let us �rst prove by induction that

a aa a

mn n

n n

21

2

1

21

+⋅ +

=+

+ for all n ≥ 0.

Proof : Base case (n = 0) : a aa a

m m02

12

0 1

22

100 1

+⋅ +

= ++

= .

Now, let us assume that it is true for n = k, k ≥ 0.

�en, a aa a

mk k

k k

21

2

1

21

+⋅ +

=+

+

a a m a a mk k k k2

12 2

12+ = ⋅ ⋅ ++ +

a m a m a a ak k k k k+ + ++ − ⋅ ⋅ +12 4

12 2

122

= m2 + m4a2k + 1 – m2 · ak . ak + 1

a2k + 1 + (m2ak + 1 – ak)2 = m2 + m2ak + 1(m2ak + 1 – ak)

a a m m a ak k k k+ + + ++ = + ⋅ ⋅12

22 2 2

1 2

Therefore, a aa a

mk k

k k

+ +

+ +

+⋅ +

=12

22

1 2

21

, proving the

induction. Hence (an, an+1) is a solution to a bab

m2 2

21

++

= for all n ≥ 0.

Now, consider the equation a bab

2 2

1++

= m2 and suppose

(a, b) = (x, y) is a solution with 0 ≤ x ≤ y. �en

x yxy

m2 2

21

++

= ...(1)

If x = 0 then it is easily seen that y = m, so (x, y) = (a0, a1). Since we are given x ≥ 0, suppose now that x > 0.Let us show that y ≤ m2x.Proof by contradiction : Assume that y > m2x. �en y = m2x + k where k ≥ 1.Substituting into (1) we get

x m x kx m x k

m2 2 2

22

1+ +

+ +=( )

( )( )

x2 + m4x2 + 2m2xk + k2 = m4x2 + m2kx + m2

(x2 + k2) + m2(kx – 1) = 0.No w, m 2( k x – 1 ) ≥ 0 s i n c e k x ≥ 1 a n d x2 + k2 ≥ x2 + 1 ≥ 1 so (x2 + k2) + m2(kx – 1) ≠ 0. �us we have a contradiction, so y ≤ m2x if x > 0.Now substitute y = m2x – x1, where 0 ≤ x1 < m2x, into (1).

We havex m x xx m x x

m2 2

12

21

2

1+ −

− +=

( )( )

x2 + m4x2 – 2m2x · x1 + x12 = m4x2 –m2x · x1 + m2

x2 + x12 = m2(x · x1 + 1)

x xx x

m2

12

1

21

+⋅ +

= ....(2)

If x1 = 0, then x2 = m2. Hence x = m and(x1, x) = (0, m) = (a0, a1). But y = m2x – x1 = a2, so (x, y) = (a1, a2). �us suppose x1 > 0.Let us now show that x1 < x.Proof by contradiction: Assume x1 ≥ x.Then m2x – y ≥ x, since y = m2x – x1, and

x yxy

x y x2 2

1++

− ≥ , since (x, y) is a solution to

a bab

m2 2

21

++

= .

So x3 + xy2 ≥ x2y + xy2 + x + y. Hence x3 ≥ x2y + x + y, which is a contradiction since y ≥ x > 0.With the same proof that y ≤ m2x, we have x ≤ m2x1. So the substitution x = m2x1 – x2 with x2 ≥ 0 is valid.Subst itut ing x = m2x1 – x2 into (2) g ives

x xx x

m12

22

1 2

21

+⋅ +

= .

If x2 ≠ 0, then we continue with the substitution

x m xi x ii= −

+ +1

22(*) until we get

x x

x xmj j

j j

21

2

1

21

+

⋅ +=+

+

and xj + 1 = 0. (�e sequence xi is decreasing, non-negative and integer.)So, if xj + 1 = 0, then x2

j = m2 so xj = m and(xj + 1, xj) = (0, m) = (a0, a1).�en (xj, xj –1) = (a1, a2) since xj –1 = m2xj – xj + 1 (from (*)).Continuing, we have (x1, x) = (an – 1, an) for some n. �en (x, y) = (an, an + 1).

Hence a bab

m2 2

21

++

= has solutions (a, b) if and

only if (a, b) = (an, an + 1) for some n.

| JUNE ‘1782

3. Let PAC and BAP be 2a and respectively.�en, since C = 2 B, we deduce fromA + B + C = 180° that2a + + 3B = 180°. ...(1)�e angles at the base of theisosceles triangle PAC are each 90° – a. Also BPC isisosceles, having base anglesC – (90° – a) = 2B + a – 90°, and so BPA = 180° – ( PBA + BAP)= 180° – [B – (2B + a – 90°) + 180° – 2a – 3B]= 4B + 3a – 90°As usual, let a, b and c denote the lengths of thesides BC, AC and AB. By the Law of Cosines,applied to BPA, where PA = b and PB = PC= 2b sin a,c2 = b2 + (2b sin a)2 – 2⋅b⋅2b sin a⋅cos (4B + 3a

– 90°),so thatc2 = b2 [1 + 4 sin2 a – 4 sina sin(4B + 3a)] ...(2)We now use the fact that C = 2 B is equivalent to the condition c2 = b(b + a).

Since a = 2⋅ PC ⋅ cos(2B + a – 90°) = 4b sin a sin (2B + a), we have

c2 = b2 [1 + 4 sin a sin(2B + a)] ...(3)�erefore, from (2) and (3), we getb2 [1 + 4 sin2 a – 4 sin a sin (4B + 3a)]

= b2 [1 + 4 sin a sin (2B + a)],which simpli�es to sin a – sin (4B + 3a) = sin (2B + a).Since sin a – sin (4B + 3a) = –2 cos(2B + 2a) sin(2B + a), this equation may be rewritten as sin(2B + a). [1 + 2 cos (2B + 2a)] = 0Since, from (1), 2B + a < 180°, we must have 1 + 2 cos(2B + 2a) = 0, giving cos(2B + 2a) = – 1/2; that is,2B + 2a = 120° ...(4)Since, again from (1), 2B + 2a < 180°Finally, we may eliminate B between (1) and (4) to obtain a = . �e result follows.

4. �e squares can only be 0, 1, 4 or 7 mod 9.�us the sum of the digits of a perfect square cannotbe 2, 3, 5, 6 or 8 mod 9, since the number itselfwould then be 2, 3, 5, 6 or 8 mod 9.We shall show that the sum of the digits of aperfect square can take every value of the form0, 1, 4 or 7 mod 9.(10m – 1)2 = 102m – 2 ⋅ 10m + 1

= ≥− −

9 9 9 8 0 01 11 1

... ... , .m m

m��� �

A

B C

Pa a

�e sum of the digits is 9m, giving all the values greater than or equal to 9 congruent to 0 mod 9(10m – 2)2 = 102m – 4 ⋅ 10m + 4

= ≥− −

9 9 9 6 0 0 4 11 1

... ... , .m m

m��� �

�e sum of the digits is 9m + 1, which gives all values greater than or equal to 10 congruent to 1 mod 9.(10m – 3)2 = 102m – 6 ⋅ 10m + 9

= ≥− −

9 9 9 4 0 0 9 11 1

... ... , .m m

m��� ���

�e sum of the digits is 9m + 4, which takes every value greater than or equal to 13 which is congru-ent to 4 mod 9(10m – 5)2 = 102m – 10m+1 + 25

=− −

9 9 0 0 0 2 51 1

... ... .m m������

�e sum of the digits is 9(m – 1) + 7 = 9m – 2, from which we get every value greater than or equal to 7 congruent to 7 mod 9.We have taken care of all the integers apart from 0, 1, 4, which are the sums of the digits of 02, 12 and 22 respectively.

5. Let O be the origin of a coordinate system whereA, B, C, D are represented by (a, 0), (0, b), (c, 0),(0, d) with a, b positive and c, d negative. �us Lis the point

02 2 2 2

0, ( ) , , , ( ) ,b d M c b N a c+( ) ( ) +( )is is and

AL : (b + d)x + 2ay – a(b + d) = 0OM : bx – cy = 0DN : 2dx + (a + c) y – d(a + c) = 0.�ese lines are concurrent if and only if

b cb d a a b d

d a c d a c

−+ − +

+ − +=

02

20( )

( ).

�is equation reduces (a�er some manipulation) to (ab – cd) [(a – c) (b – d) + 2bc] = 0.Consequently, either(a) ab = cd, in which case AD BC, or

(b) 12

2 12

( )( )sin sina c b d bc− − = −( )α α

(where a = AOB), in which case [ABCD] = 2 [OBC].

| JUNE ‘1784

1. For r = 0, 1, ........., 10, let Ar, Br, Cr denote respectively, the coe�cients of xr in the expansion of (1 + x)10,

(1 + x)20, (1 + x)30. �en r =∑

1

10 Ar (B10Br – C10 Ar) =

(Ram Krishan, West Bengal)Ans. (1 + x)10 = A0 + A1x + A2x2 + ........... + A10x10 (x + 1)20 = B0x20 + B1x19 + B2x18 + ............+ B20. Considering the coe�cient of x20 in the product, we getA0B0 + A1B1 + A2B2 + .......... + A10 B10 = coe�cient of x20 in the expansion of (1 + x)10(x + 1)20 = (1 + x)30 which is C20

\ A B C Cr rr

= ==∑ 20 10

0

10

But n n n n

nn

n0 1 222 2 2 2

+

+

+ +

=

........

\ A02 + A1

2 + A22 + .............. + A Bn

210

2010

=

=

A Brr

210

0

10=

=∑ . Now, A B B C Ar r r

r( )10 10

1

10−

=∑

= −==∑∑B A B C Ar r r

rr10 10

2

1

10

1

10

= B10 (C10 – 1) – C10 (B10 – 1) = C10 – B10.

2. If

4 4 1

4 4 1

4 4 1

112

2

2

2

a a

b b

c c

fff

( )( )( )

=

3 3

3 3

3 3

2

2

2

a a

b b

c c

+

+

+

, f (x) is a

quadratic function and its maximum value occurs at a point V. A is a point of intersection of y = f (x) with x-axis and point B is such that the chord AB subtends a right angle at V. Find the area enclosed by y = f (x) and the chord AB. (Suresh Prasad, Jharkhand)

Ans. �e given equation implies, (4f (–1) – 3) x2 + (4f (1) – 3) x + f (2) = 0 is satis�ed by 3 roots a, b, c

⇒ It is an identity \ f (–1) = 34 , f (1) =

34 , f (2) = 0

If f (a) = a x2 + x + , then = 0, a = − 14 , = 1

⇒ f (x) = − + = −x x2 2

41 4

4�e maximum point, V = (0, 1) ; A(– 2, 0). Taking B = (2t, 1 – t2)

AVB = π2 ⇒ 1

2 2. −

t = – 1 ⇒ t = 4 \ B = (8, – 15)

Equation of AB is 3x + 2y + 6 = 0

�e area required is 44

3 22

2

2

8 − + +

∫ x x( )dx = 41.67

3. Let ABC be an equilateral triangle inscribed in thecircle x2 + y2 = a2. Suppose perpendiculars from A,

B, C to the major axis meet the ellipse xa

y

b

2

2

2

21+ =

(a > b) respectively at P, Q, R so that P, Q, R lie on the same side of the major axis as A, B, C respectively.Prove that the normals to the ellipse at P, Q, R areconcurrent. (Priyanshu Sharma, Bihar)

Ans. A, B, C are the vertices of an equilateral triangle.A (a cos , a sin ),

B a acos , sin ,θ π θ π+

+

23

23

C a acos , sinθ π θ π+

+

43

43

Hence, P (a cos , b sin ),

Q a bcos , sin ,θ π θ π+

+

23

23

R a bcos , sinθ π θ π+

+

43

43

�e normals to the ellipse at P, Q, R are respectively,

L1 ax sin – by cos – a b2 2

2− sin 2 = 0;

L2 ax sin θ π+

23

– by cos θ π+

23

– a b2 2

2−

sin2 θ π+

23

= 0; L3 ax sin θ π+

43

– by cos θ π+

43

– a b2 2

22 4

3− +sin ( )θ π = 0

Since, sin + sin θ π+

23

+ sin θ π+

43

= 0

cos + cos θ π+

23

+ cos θ π+

43

= 0

sin 2 + sin 2 θ π+

23

+ sin 2 θ π+

43 = 0

Hence L1, L2, L3 are concurrent.

Do you have a question that you just can’t get answered?Use the vast expertise of our MTG team to get to the bottom of the question. From the serious to the silly, the controversial to the trivial, the team will tackle the questions, easy and tough.The best questions and their solutions will be printed in this column each month.

Y UASKWE ANSWER

| JUNE ‘17 85

SOLUTION SET-173

1. (c) : Anil Balu ProbabilityR, R R, R 1/33B, B B, B 1/33R, B R, B 10/33

\ Required probability = + = =233

1033

1233

411

2. (b) : Let (h, k) be the mid point of the chord ofcircle x2 + y2 = a2, then the equation of the chord be S1 = T

⇒ xh + yk – a2 = h2 + k2 – a2

⇒ h2 + k2 = xh + yk⇒ h2 + k2 = ah + bk (As it passes through(a, b))⇒ x2 + y2 = ax + by (By changing the locus of

h, k → x, y)

3. (b) : Let I = dx

x xI x

xdx

cos sinsec

tan

/ /

6 60

2 6

60

2

1+⇒ =

+∫ ∫π π

⇒ I = ( tan )

tan

/ 11

2 3

60

2 ++∫

xx

dxπ

⇒ I =1 3 1

1

6 2 2

60

2 + + ++∫

tan tan ( tan )tan

/ x x xx

dxπ

I = 13

1

2 2

3 20

2

++∫

tan sec(tan )

/ x x dxx

π

Put tan3 x = t ⇒ dt = 3tan2x sec2x dx

⇒ I = dx dtt

++∫ ∫

0

2

20 1

π/

= π π π π2 2 2

1

0+ = + =−

tan ( )t

4. (d) : Consider Cr =24r

(1 – x)24 = C0 – C1x + C2x2 – .....+ C16x16 + .....(1 – x)–1 = 1 + x + x2 + .....+ x16 + .....Considering the coe�cient of x16 in the product of these two,C0 – C1 + C2 – .....+ C16 = coe�. of x16 in (1 – x)23

=

=

2316

237

5. (a) : Let the A.P.s. be a, a + a, a + 2a,... andb, b + , b + 2 ,...⇒ ab = a1b1 = 120, (a + a)(b + ) = a2b2 = 143,(a + 2a)(b + 2 ) = a3b3 = 154⇒ ab = 120, a + ba = 29, a = –6\ a8b8 = (a + 7a)(b + 7 ) = ab + 7(a + ba) + 49a = 120 + 7 × 29 – 49 × 6 = 29

6. (b, c) : f (x) = loge [ ]x x3 6 1+ +

\ f (–x)= loge [ ]− + +x x3 6 1\ f (x) + f (–x) = loge1 = 0

f (–x) = – f (x)Hence f(x) is an odd function. \ (b) is correct.

Again, f (x) = 1

13 6

2 13 6

25

6x xx x

x+ ++

+

= 1

13 1

13 6

26 3

6x xx x x

x+ +

+ +

+

=+

>3

10

2

6

x

x\ f (x) is an increasing function, so (c) is correct.

7. (b) : DBC = a ⇒ BD = 20, EBD = aBy sine rule for EBD, we getsin sin3

20 8α α=

A

B

3� 2�

E 8 D 20 C

⇒ − =3 4 52

2sin α

⇒ = =sin , cos2 18

2 34

α α .

8. (d) : AE + 28 = AB cot a and AE + 8 = AB cot 2aOn subtracting, we get 20 = AB (cot a – cot 2a)

\ AB = 20 sin 2a = 20 1 916

5 7− =

\ AE AB+ = = =8 2 5 7 37

15cot α ⋅ ⇒ AE = 7.

9. (8) : a + b + c = 0 ⇒ a, b, c are the roots ofx3 + qx + r = 0 ...(i)

ab = q, abc = –r, a + b + c = 0⇒ a3 = 3abc \ abc = 1, r = –1, a2 = –2qFrom (i), we get a5 + q a3 + r a2 = 0 ⇒ 10 + 3q + 2q = 0 ⇒ q = –2From (i), x3 – 2x – 1 = 0.\ a3 – 2 a – = 0 ⇒ a4 = 2 a2 + a

a4 = 2(–2q) + 0 = (–4) –2) = 8. 10. (b) : P. 232 = – 1 mod 53, 2322 = – 1 mod 53, 2323 = – 23 mod 53 = 30 mod 53Q. X = x + 2, Y = y + 1, Z = z, U = u – 1⇒ X + Y + Z + U = 5

\ Number of solutions = 83

56

=

R. Coe�cient of x2y in (1 + x + 2y)5 is 52 2

2 60!

! !.

⋅=

S. r rrr=

∑ − = + + + =1

10 11 1 2 10 55( ) ... .